Feat: DM pour les TST1

This commit is contained in:
Bertrand Benjamin 2021-02-04 11:18:14 +01:00
parent c2b2687bf0
commit ef6c26a51c
45 changed files with 8752 additions and 0 deletions

View File

@ -0,0 +1,203 @@
\documentclass[a5paper,10pt]{article}
\usepackage{myXsim}
\usepackage{tasks}
% Title Page
\title{DM2 \hfill AIOUAZ Ahmed}
\tribe{TST}
\date{\hfillÀ render pour le Mercredi 24 février}
\xsimsetup{
solution/print = false
}
\begin{document}
\maketitle
\begin{exercise}[subtitle={Loi binomiale}]
Trois personnes s'apprêtent à passer le portique de sécurité. On suppose que pour chaque personne la probabilité que le portique sonne est égale à $0.8$.
Soit $X$ la variable aléatoire donnant le nombre de personnes faisant sonner le portique, parmi les 3 personnes de ce groupe.
\begin{enumerate}
\item Tracer l'arbre représentant le situation.
\item Justifier que $X$ suit une loi binomiale dont on précisera les paramètres.
\item Quelle est la probabilité qu'une seule personne fasse sonner le portique?
\item Calculer puis interpréter les probabilités suivantes
\[
P(X = 0) \qquad \qquad P(X \geq 2)
\]
\item Calculer l'espérance de $X$ et interpréter le résultat.
\end{enumerate}
\end{exercise}
\begin{solution}
\begin{enumerate}
\item
\begin{tikzpicture}[sloped]
\node {.}
child {node {$0$}
child {node {$0$}
child {node {$0$}
edge from parent
node[above] {0.2}
}
child {node {$1$}
edge from parent
node[above] {0.8}
}
edge from parent
node[above] {0.2}
}
child[missing] {}
child {node {$1$}
child {node {$0$}
edge from parent
node[above] {0.2}
}
child {node {$1$}
edge from parent
node[above] {0.8}
}
edge from parent
node[above] {0.2}
}
edge from parent
node[above] {0.2}
}
child[missing] {}
child[missing] {}
child[missing] {}
child { node {$1$}
child {node {$0$}
child {node {$0$}
edge from parent
node[above] {0.2}
}
child {node {$1$}
edge from parent
node[above] {0.8}
}
edge from parent
node[above] {0.2}
}
child[missing] {}
child {node {$1$}
child {node {$0$}
edge from parent
node[above] {0.2}
}
child {node {$1$}
edge from parent
node[above] {0.8}
}
edge from parent
node[above] {0.2}
}
edge from parent
node[above] {0.8}
} ;
\end{tikzpicture}
\item Chaque personne a 2 possibilités (1: fait sonner ou 2: ne fait pas sonner) et l'on fait passer 3 personnes ce qui correspond à une répétition identique et aléatoire. On peut donc modéliser la situation par une loi binomiale.
\[
X \sim \mathcal{B}(3; 0.76)
\]
\item Probabilité qu'une seule personne fasse sonner le portique. On voit qu'il y a 3 branches qui correspondent à cette situation dont
\[
P(X = 1) = 3 \times 0.8^1 \times 0.2^2 \approx 0.096
\]
\item
\[
P(X = 0) = 0.2^3 \approx 0.008
\]
\[
P(X \geq 2) = P(X = 2) + P(X = 3) = 3 \times 0.8^2 \times 0.2^1 + 0.8^3 \approx 0.896
\]
\item Il faut d'abord tracer le tableau résumant la loi de probabilité:
\begin{center}
\begin{tabular}{|c|*{4}{c|}}
\hline
Valeur & 0 & 1 & 2 & 3 \\
\hline
Probabilité & $0.008$ & $0.096$ & $0.384$ &$0.512$ \\
\hline
\end{tabular}
\end{center}
On peut alors calculer l'espérance
\[
E[X] = 0 \times 0.008 + 1 \times 0.096 + 2 \times 0.384 + 3 \times 0.512 = 2.4
\]
On peut donc estimer qu'il y aura en moyenne $2.4$ personnes qui feront sonner le portique sur les 3 personnes.
\end{enumerate}
\end{solution}
\begin{exercise}[subtitle={Équation puissance}]
Résoudre les équations et inéquations suivantes
\begin{multicols}{2}
\begin{enumerate}
\item $10^x = 27$
\item $20^x = 45$
\item $0.55^x \leq 27$
\item $3 \times 0.31^x = 37$
\end{enumerate}
\end{multicols}
\end{exercise}
\begin{solution}
Les solutions ci-dessous ne sont pas justifiée car l'ordinateur ne sait pas faire. Par contre, vous vous devez savoir justifier vos réponses!
\begin{enumerate}
\item $x = \log(27)$
\item $x = \frac{\log(45)}{\log(20)}$
\item Il faut faire attention quand on divise par un log car ce dernier peut être négatif ce qui est le cas ici. Il faut donc pense à changer le sens de l'inégalité.
$x \geq \frac{\log(27)}{\log(0.55)}$
\item Il faut penser à faire la division à par $3$ avant d'utiliser le log car sinon, on ne peut pas utiliser la formule $\log(a^n) = n\times \log(a)$.
$x = \frac{\log(12.33)}{\log(0.31)}$
\end{enumerate}
\end{solution}
\begin{exercise}[subtitle={Étude de fonctions}]
Soit $f(x) = - 10x^3 + 720x^2 - 11400x - 9$ une fonction définie sur $\R$.
\begin{enumerate}
\item Calculer $f'(x)$ la dérivée de $f(x)$.
\item Calculer $f'(38)$ et $f'(10)$.
\item En déduire une forme factorisée de $f'(x)$.
\item Étudier le signe de $f'(x)$ et en déduire les variations de $f(x)$.
\item Est-ce que la fonction $f(x)$ admet un maximum ou un minimum? Si oui, calculer sa valeur.
\end{enumerate}
\end{exercise}
\begin{solution}
\begin{enumerate}
\item Dérivée de $f(x)$: $f'(x) = - 30x^2 + 1440x - 11400$
\item
\begin{align*}
f'(38) &= - 30 \times 38^{2} + 1440 \times 38 - 11400\\&= - 30 \times 1444 + 54720 - 11400\\&= - 43320 + 43320\\&= 0
\end{align*}
\begin{align*}
f'(10) &= - 30 \times 10^{2} + 1440 \times 10 - 11400\\&= - 30 \times 100 + 14400 - 11400\\&= - 3000 + 3000\\&= 0
\end{align*}
Donc $x = 38$ et $x=10$ sont des racines de $f'(x) = - 30x^2 + 1440x - 11400$.
\item On en déduit la forme factorisée suivante
\[
f'(x) = -30 (x - 38)(x-10)
\]
\item Pas de correction disponible
\item À causes des branches extérieurs, la fonction $f(x)$ n'a pas de maximum ou de minimum.
\end{enumerate}
\end{solution}
%\printsolutionstype{exercise}
\end{document}
%%% Local Variables:
%%% mode: latex
%%% TeX-master: "master"
%%% End:

View File

@ -0,0 +1,203 @@
\documentclass[a5paper,10pt]{article}
\usepackage{myXsim}
\usepackage{tasks}
% Title Page
\title{DM2 \hfill BAHBAH Zakaria}
\tribe{TST}
\date{\hfillÀ render pour le Mercredi 24 février}
\xsimsetup{
solution/print = false
}
\begin{document}
\maketitle
\begin{exercise}[subtitle={Loi binomiale}]
Trois personnes s'apprêtent à passer le portique de sécurité. On suppose que pour chaque personne la probabilité que le portique sonne est égale à $0.25$.
Soit $X$ la variable aléatoire donnant le nombre de personnes faisant sonner le portique, parmi les 3 personnes de ce groupe.
\begin{enumerate}
\item Tracer l'arbre représentant le situation.
\item Justifier que $X$ suit une loi binomiale dont on précisera les paramètres.
\item Quelle est la probabilité qu'une seule personne fasse sonner le portique?
\item Calculer puis interpréter les probabilités suivantes
\[
P(X = 0) \qquad \qquad P(X \geq 2)
\]
\item Calculer l'espérance de $X$ et interpréter le résultat.
\end{enumerate}
\end{exercise}
\begin{solution}
\begin{enumerate}
\item
\begin{tikzpicture}[sloped]
\node {.}
child {node {$0$}
child {node {$0$}
child {node {$0$}
edge from parent
node[above] {0.75}
}
child {node {$1$}
edge from parent
node[above] {0.25}
}
edge from parent
node[above] {0.75}
}
child[missing] {}
child {node {$1$}
child {node {$0$}
edge from parent
node[above] {0.75}
}
child {node {$1$}
edge from parent
node[above] {0.25}
}
edge from parent
node[above] {0.75}
}
edge from parent
node[above] {0.75}
}
child[missing] {}
child[missing] {}
child[missing] {}
child { node {$1$}
child {node {$0$}
child {node {$0$}
edge from parent
node[above] {0.75}
}
child {node {$1$}
edge from parent
node[above] {0.25}
}
edge from parent
node[above] {0.75}
}
child[missing] {}
child {node {$1$}
child {node {$0$}
edge from parent
node[above] {0.75}
}
child {node {$1$}
edge from parent
node[above] {0.25}
}
edge from parent
node[above] {0.75}
}
edge from parent
node[above] {0.25}
} ;
\end{tikzpicture}
\item Chaque personne a 2 possibilités (1: fait sonner ou 2: ne fait pas sonner) et l'on fait passer 3 personnes ce qui correspond à une répétition identique et aléatoire. On peut donc modéliser la situation par une loi binomiale.
\[
X \sim \mathcal{B}(3; 0.76)
\]
\item Probabilité qu'une seule personne fasse sonner le portique. On voit qu'il y a 3 branches qui correspondent à cette situation dont
\[
P(X = 1) = 3 \times 0.25^1 \times 0.75^2 \approx 0.422
\]
\item
\[
P(X = 0) = 0.75^3 \approx 0.422
\]
\[
P(X \geq 2) = P(X = 2) + P(X = 3) = 3 \times 0.25^2 \times 0.75^1 + 0.25^3 \approx 0.157
\]
\item Il faut d'abord tracer le tableau résumant la loi de probabilité:
\begin{center}
\begin{tabular}{|c|*{4}{c|}}
\hline
Valeur & 0 & 1 & 2 & 3 \\
\hline
Probabilité & $0.422$ & $0.422$ & $0.141$ &$0.016$ \\
\hline
\end{tabular}
\end{center}
On peut alors calculer l'espérance
\[
E[X] = 0 \times 0.422 + 1 \times 0.422 + 2 \times 0.141 + 3 \times 0.016 = 0.75
\]
On peut donc estimer qu'il y aura en moyenne $0.75$ personnes qui feront sonner le portique sur les 3 personnes.
\end{enumerate}
\end{solution}
\begin{exercise}[subtitle={Équation puissance}]
Résoudre les équations et inéquations suivantes
\begin{multicols}{2}
\begin{enumerate}
\item $10^x = 6$
\item $4^x = 46$
\item $0.74^x \leq 39$
\item $4 \times 0.52^x = 19$
\end{enumerate}
\end{multicols}
\end{exercise}
\begin{solution}
Les solutions ci-dessous ne sont pas justifiée car l'ordinateur ne sait pas faire. Par contre, vous vous devez savoir justifier vos réponses!
\begin{enumerate}
\item $x = \log(6)$
\item $x = \frac{\log(46)}{\log(4)}$
\item Il faut faire attention quand on divise par un log car ce dernier peut être négatif ce qui est le cas ici. Il faut donc pense à changer le sens de l'inégalité.
$x \geq \frac{\log(39)}{\log(0.74)}$
\item Il faut penser à faire la division à par $4$ avant d'utiliser le log car sinon, on ne peut pas utiliser la formule $\log(a^n) = n\times \log(a)$.
$x = \frac{\log(4.75)}{\log(0.52)}$
\end{enumerate}
\end{solution}
\begin{exercise}[subtitle={Étude de fonctions}]
Soit $f(x) = 10x^3 - 600x^2 - 15000x + 35$ une fonction définie sur $\R$.
\begin{enumerate}
\item Calculer $f'(x)$ la dérivée de $f(x)$.
\item Calculer $f'(50)$ et $f'(-10)$.
\item En déduire une forme factorisée de $f'(x)$.
\item Étudier le signe de $f'(x)$ et en déduire les variations de $f(x)$.
\item Est-ce que la fonction $f(x)$ admet un maximum ou un minimum? Si oui, calculer sa valeur.
\end{enumerate}
\end{exercise}
\begin{solution}
\begin{enumerate}
\item Dérivée de $f(x)$: $f'(x) = 30x^2 - 1200x - 15000$
\item
\begin{align*}
f'(50) &= 30 \times 50^{2} - 1200 \times 50 - 15000\\&= 30 \times 2500 - 60000 - 15000\\&= 75000 - 75000\\&= 0
\end{align*}
\begin{align*}
f'(-10) &= 30 \times - 10^{2} - 1200(- 10) - 15000\\&= 30 \times 100 + 12000 - 15000\\&= 3000 - 3000\\&= 0
\end{align*}
Donc $x = 50$ et $x=-10$ sont des racines de $f'(x) = 30x^2 - 1200x - 15000$.
\item On en déduit la forme factorisée suivante
\[
f'(x) = 30 (x - 50)(x--10)
\]
\item Pas de correction disponible
\item À causes des branches extérieurs, la fonction $f(x)$ n'a pas de maximum ou de minimum.
\end{enumerate}
\end{solution}
%\printsolutionstype{exercise}
\end{document}
%%% Local Variables:
%%% mode: latex
%%% TeX-master: "master"
%%% End:

View File

@ -0,0 +1,203 @@
\documentclass[a5paper,10pt]{article}
\usepackage{myXsim}
\usepackage{tasks}
% Title Page
\title{DM2 \hfill BALLOFFET Kenza}
\tribe{TST}
\date{\hfillÀ render pour le Mercredi 24 février}
\xsimsetup{
solution/print = false
}
\begin{document}
\maketitle
\begin{exercise}[subtitle={Loi binomiale}]
Trois personnes s'apprêtent à passer le portique de sécurité. On suppose que pour chaque personne la probabilité que le portique sonne est égale à $0.83$.
Soit $X$ la variable aléatoire donnant le nombre de personnes faisant sonner le portique, parmi les 3 personnes de ce groupe.
\begin{enumerate}
\item Tracer l'arbre représentant le situation.
\item Justifier que $X$ suit une loi binomiale dont on précisera les paramètres.
\item Quelle est la probabilité qu'une seule personne fasse sonner le portique?
\item Calculer puis interpréter les probabilités suivantes
\[
P(X = 0) \qquad \qquad P(X \geq 2)
\]
\item Calculer l'espérance de $X$ et interpréter le résultat.
\end{enumerate}
\end{exercise}
\begin{solution}
\begin{enumerate}
\item
\begin{tikzpicture}[sloped]
\node {.}
child {node {$0$}
child {node {$0$}
child {node {$0$}
edge from parent
node[above] {0.17}
}
child {node {$1$}
edge from parent
node[above] {0.83}
}
edge from parent
node[above] {0.17}
}
child[missing] {}
child {node {$1$}
child {node {$0$}
edge from parent
node[above] {0.17}
}
child {node {$1$}
edge from parent
node[above] {0.83}
}
edge from parent
node[above] {0.17}
}
edge from parent
node[above] {0.17}
}
child[missing] {}
child[missing] {}
child[missing] {}
child { node {$1$}
child {node {$0$}
child {node {$0$}
edge from parent
node[above] {0.17}
}
child {node {$1$}
edge from parent
node[above] {0.83}
}
edge from parent
node[above] {0.17}
}
child[missing] {}
child {node {$1$}
child {node {$0$}
edge from parent
node[above] {0.17}
}
child {node {$1$}
edge from parent
node[above] {0.83}
}
edge from parent
node[above] {0.17}
}
edge from parent
node[above] {0.83}
} ;
\end{tikzpicture}
\item Chaque personne a 2 possibilités (1: fait sonner ou 2: ne fait pas sonner) et l'on fait passer 3 personnes ce qui correspond à une répétition identique et aléatoire. On peut donc modéliser la situation par une loi binomiale.
\[
X \sim \mathcal{B}(3; 0.76)
\]
\item Probabilité qu'une seule personne fasse sonner le portique. On voit qu'il y a 3 branches qui correspondent à cette situation dont
\[
P(X = 1) = 3 \times 0.83^1 \times 0.17^2 \approx 0.072
\]
\item
\[
P(X = 0) = 0.17^3 \approx 0.005
\]
\[
P(X \geq 2) = P(X = 2) + P(X = 3) = 3 \times 0.83^2 \times 0.17^1 + 0.83^3 \approx 0.923
\]
\item Il faut d'abord tracer le tableau résumant la loi de probabilité:
\begin{center}
\begin{tabular}{|c|*{4}{c|}}
\hline
Valeur & 0 & 1 & 2 & 3 \\
\hline
Probabilité & $0.005$ & $0.072$ & $0.351$ &$0.572$ \\
\hline
\end{tabular}
\end{center}
On peut alors calculer l'espérance
\[
E[X] = 0 \times 0.005 + 1 \times 0.072 + 2 \times 0.351 + 3 \times 0.572 = 2.49
\]
On peut donc estimer qu'il y aura en moyenne $2.49$ personnes qui feront sonner le portique sur les 3 personnes.
\end{enumerate}
\end{solution}
\begin{exercise}[subtitle={Équation puissance}]
Résoudre les équations et inéquations suivantes
\begin{multicols}{2}
\begin{enumerate}
\item $10^x = 15$
\item $6^x = 15$
\item $0.63^x \leq 2$
\item $8 \times 0.02^x = 23$
\end{enumerate}
\end{multicols}
\end{exercise}
\begin{solution}
Les solutions ci-dessous ne sont pas justifiée car l'ordinateur ne sait pas faire. Par contre, vous vous devez savoir justifier vos réponses!
\begin{enumerate}
\item $x = \log(15)$
\item $x = \frac{\log(15)}{\log(6)}$
\item Il faut faire attention quand on divise par un log car ce dernier peut être négatif ce qui est le cas ici. Il faut donc pense à changer le sens de l'inégalité.
$x \geq \frac{\log(2)}{\log(0.63)}$
\item Il faut penser à faire la division à par $8$ avant d'utiliser le log car sinon, on ne peut pas utiliser la formule $\log(a^n) = n\times \log(a)$.
$x = \frac{\log(2.88)}{\log(0.02)}$
\end{enumerate}
\end{solution}
\begin{exercise}[subtitle={Étude de fonctions}]
Soit $f(x) = 3x^3 + 18x^2 - 1485x - 39$ une fonction définie sur $\R$.
\begin{enumerate}
\item Calculer $f'(x)$ la dérivée de $f(x)$.
\item Calculer $f'(11)$ et $f'(-15)$.
\item En déduire une forme factorisée de $f'(x)$.
\item Étudier le signe de $f'(x)$ et en déduire les variations de $f(x)$.
\item Est-ce que la fonction $f(x)$ admet un maximum ou un minimum? Si oui, calculer sa valeur.
\end{enumerate}
\end{exercise}
\begin{solution}
\begin{enumerate}
\item Dérivée de $f(x)$: $f'(x) = 9x^2 + 36x - 1485$
\item
\begin{align*}
f'(11) &= 9 \times 11^{2} + 36 \times 11 - 1485\\&= 9 \times 121 + 396 - 1485\\&= 1089 - 1089\\&= 0
\end{align*}
\begin{align*}
f'(-15) &= 9 \times - 15^{2} + 36(- 15) - 1485\\&= 9 \times 225 - 540 - 1485\\&= 2025 - 2025\\&= 0
\end{align*}
Donc $x = 11$ et $x=-15$ sont des racines de $f'(x) = 9x^2 + 36x - 1485$.
\item On en déduit la forme factorisée suivante
\[
f'(x) = 9 (x - 11)(x--15)
\]
\item Pas de correction disponible
\item À causes des branches extérieurs, la fonction $f(x)$ n'a pas de maximum ou de minimum.
\end{enumerate}
\end{solution}
%\printsolutionstype{exercise}
\end{document}
%%% Local Variables:
%%% mode: latex
%%% TeX-master: "master"
%%% End:

View File

@ -0,0 +1,203 @@
\documentclass[a5paper,10pt]{article}
\usepackage{myXsim}
\usepackage{tasks}
% Title Page
\title{DM2 \hfill BENHATTAL Chakir}
\tribe{TST}
\date{\hfillÀ render pour le Mercredi 24 février}
\xsimsetup{
solution/print = false
}
\begin{document}
\maketitle
\begin{exercise}[subtitle={Loi binomiale}]
Trois personnes s'apprêtent à passer le portique de sécurité. On suppose que pour chaque personne la probabilité que le portique sonne est égale à $0.16$.
Soit $X$ la variable aléatoire donnant le nombre de personnes faisant sonner le portique, parmi les 3 personnes de ce groupe.
\begin{enumerate}
\item Tracer l'arbre représentant le situation.
\item Justifier que $X$ suit une loi binomiale dont on précisera les paramètres.
\item Quelle est la probabilité qu'une seule personne fasse sonner le portique?
\item Calculer puis interpréter les probabilités suivantes
\[
P(X = 0) \qquad \qquad P(X \geq 2)
\]
\item Calculer l'espérance de $X$ et interpréter le résultat.
\end{enumerate}
\end{exercise}
\begin{solution}
\begin{enumerate}
\item
\begin{tikzpicture}[sloped]
\node {.}
child {node {$0$}
child {node {$0$}
child {node {$0$}
edge from parent
node[above] {0.84}
}
child {node {$1$}
edge from parent
node[above] {0.16}
}
edge from parent
node[above] {0.84}
}
child[missing] {}
child {node {$1$}
child {node {$0$}
edge from parent
node[above] {0.84}
}
child {node {$1$}
edge from parent
node[above] {0.16}
}
edge from parent
node[above] {0.84}
}
edge from parent
node[above] {0.84}
}
child[missing] {}
child[missing] {}
child[missing] {}
child { node {$1$}
child {node {$0$}
child {node {$0$}
edge from parent
node[above] {0.84}
}
child {node {$1$}
edge from parent
node[above] {0.16}
}
edge from parent
node[above] {0.84}
}
child[missing] {}
child {node {$1$}
child {node {$0$}
edge from parent
node[above] {0.84}
}
child {node {$1$}
edge from parent
node[above] {0.16}
}
edge from parent
node[above] {0.84}
}
edge from parent
node[above] {0.16}
} ;
\end{tikzpicture}
\item Chaque personne a 2 possibilités (1: fait sonner ou 2: ne fait pas sonner) et l'on fait passer 3 personnes ce qui correspond à une répétition identique et aléatoire. On peut donc modéliser la situation par une loi binomiale.
\[
X \sim \mathcal{B}(3; 0.76)
\]
\item Probabilité qu'une seule personne fasse sonner le portique. On voit qu'il y a 3 branches qui correspondent à cette situation dont
\[
P(X = 1) = 3 \times 0.16^1 \times 0.84^2 \approx 0.339
\]
\item
\[
P(X = 0) = 0.84^3 \approx 0.593
\]
\[
P(X \geq 2) = P(X = 2) + P(X = 3) = 3 \times 0.16^2 \times 0.84^1 + 0.16^3 \approx 0.069
\]
\item Il faut d'abord tracer le tableau résumant la loi de probabilité:
\begin{center}
\begin{tabular}{|c|*{4}{c|}}
\hline
Valeur & 0 & 1 & 2 & 3 \\
\hline
Probabilité & $0.593$ & $0.339$ & $0.065$ &$0.004$ \\
\hline
\end{tabular}
\end{center}
On peut alors calculer l'espérance
\[
E[X] = 0 \times 0.593 + 1 \times 0.339 + 2 \times 0.065 + 3 \times 0.004 = 0.48
\]
On peut donc estimer qu'il y aura en moyenne $0.48$ personnes qui feront sonner le portique sur les 3 personnes.
\end{enumerate}
\end{solution}
\begin{exercise}[subtitle={Équation puissance}]
Résoudre les équations et inéquations suivantes
\begin{multicols}{2}
\begin{enumerate}
\item $10^x = 31$
\item $11^x = 39$
\item $0.65^x \leq 46$
\item $2 \times 0.75^x = 11$
\end{enumerate}
\end{multicols}
\end{exercise}
\begin{solution}
Les solutions ci-dessous ne sont pas justifiée car l'ordinateur ne sait pas faire. Par contre, vous vous devez savoir justifier vos réponses!
\begin{enumerate}
\item $x = \log(31)$
\item $x = \frac{\log(39)}{\log(11)}$
\item Il faut faire attention quand on divise par un log car ce dernier peut être négatif ce qui est le cas ici. Il faut donc pense à changer le sens de l'inégalité.
$x \geq \frac{\log(46)}{\log(0.65)}$
\item Il faut penser à faire la division à par $2$ avant d'utiliser le log car sinon, on ne peut pas utiliser la formule $\log(a^n) = n\times \log(a)$.
$x = \frac{\log(5.5)}{\log(0.75)}$
\end{enumerate}
\end{solution}
\begin{exercise}[subtitle={Étude de fonctions}]
Soit $f(x) = - 4x^3 + 354x^2 - 9120x + 5$ une fonction définie sur $\R$.
\begin{enumerate}
\item Calculer $f'(x)$ la dérivée de $f(x)$.
\item Calculer $f'(40)$ et $f'(19)$.
\item En déduire une forme factorisée de $f'(x)$.
\item Étudier le signe de $f'(x)$ et en déduire les variations de $f(x)$.
\item Est-ce que la fonction $f(x)$ admet un maximum ou un minimum? Si oui, calculer sa valeur.
\end{enumerate}
\end{exercise}
\begin{solution}
\begin{enumerate}
\item Dérivée de $f(x)$: $f'(x) = - 12x^2 + 708x - 9120$
\item
\begin{align*}
f'(40) &= - 12 \times 40^{2} + 708 \times 40 - 9120\\&= - 12 \times 1600 + 28320 - 9120\\&= - 19200 + 19200\\&= 0
\end{align*}
\begin{align*}
f'(19) &= - 12 \times 19^{2} + 708 \times 19 - 9120\\&= - 12 \times 361 + 13452 - 9120\\&= - 4332 + 4332\\&= 0
\end{align*}
Donc $x = 40$ et $x=19$ sont des racines de $f'(x) = - 12x^2 + 708x - 9120$.
\item On en déduit la forme factorisée suivante
\[
f'(x) = -12 (x - 40)(x-19)
\]
\item Pas de correction disponible
\item À causes des branches extérieurs, la fonction $f(x)$ n'a pas de maximum ou de minimum.
\end{enumerate}
\end{solution}
%\printsolutionstype{exercise}
\end{document}
%%% Local Variables:
%%% mode: latex
%%% TeX-master: "master"
%%% End:

View File

@ -0,0 +1,203 @@
\documentclass[a5paper,10pt]{article}
\usepackage{myXsim}
\usepackage{tasks}
% Title Page
\title{DM2 \hfill CLAIN Avinash}
\tribe{TST}
\date{\hfillÀ render pour le Mercredi 24 février}
\xsimsetup{
solution/print = false
}
\begin{document}
\maketitle
\begin{exercise}[subtitle={Loi binomiale}]
Trois personnes s'apprêtent à passer le portique de sécurité. On suppose que pour chaque personne la probabilité que le portique sonne est égale à $0.29$.
Soit $X$ la variable aléatoire donnant le nombre de personnes faisant sonner le portique, parmi les 3 personnes de ce groupe.
\begin{enumerate}
\item Tracer l'arbre représentant le situation.
\item Justifier que $X$ suit une loi binomiale dont on précisera les paramètres.
\item Quelle est la probabilité qu'une seule personne fasse sonner le portique?
\item Calculer puis interpréter les probabilités suivantes
\[
P(X = 0) \qquad \qquad P(X \geq 2)
\]
\item Calculer l'espérance de $X$ et interpréter le résultat.
\end{enumerate}
\end{exercise}
\begin{solution}
\begin{enumerate}
\item
\begin{tikzpicture}[sloped]
\node {.}
child {node {$0$}
child {node {$0$}
child {node {$0$}
edge from parent
node[above] {0.71}
}
child {node {$1$}
edge from parent
node[above] {0.29}
}
edge from parent
node[above] {0.71}
}
child[missing] {}
child {node {$1$}
child {node {$0$}
edge from parent
node[above] {0.71}
}
child {node {$1$}
edge from parent
node[above] {0.29}
}
edge from parent
node[above] {0.71}
}
edge from parent
node[above] {0.71}
}
child[missing] {}
child[missing] {}
child[missing] {}
child { node {$1$}
child {node {$0$}
child {node {$0$}
edge from parent
node[above] {0.71}
}
child {node {$1$}
edge from parent
node[above] {0.29}
}
edge from parent
node[above] {0.71}
}
child[missing] {}
child {node {$1$}
child {node {$0$}
edge from parent
node[above] {0.71}
}
child {node {$1$}
edge from parent
node[above] {0.29}
}
edge from parent
node[above] {0.71}
}
edge from parent
node[above] {0.29}
} ;
\end{tikzpicture}
\item Chaque personne a 2 possibilités (1: fait sonner ou 2: ne fait pas sonner) et l'on fait passer 3 personnes ce qui correspond à une répétition identique et aléatoire. On peut donc modéliser la situation par une loi binomiale.
\[
X \sim \mathcal{B}(3; 0.76)
\]
\item Probabilité qu'une seule personne fasse sonner le portique. On voit qu'il y a 3 branches qui correspondent à cette situation dont
\[
P(X = 1) = 3 \times 0.29^1 \times 0.71^2 \approx 0.439
\]
\item
\[
P(X = 0) = 0.71^3 \approx 0.358
\]
\[
P(X \geq 2) = P(X = 2) + P(X = 3) = 3 \times 0.29^2 \times 0.71^1 + 0.29^3 \approx 0.203
\]
\item Il faut d'abord tracer le tableau résumant la loi de probabilité:
\begin{center}
\begin{tabular}{|c|*{4}{c|}}
\hline
Valeur & 0 & 1 & 2 & 3 \\
\hline
Probabilité & $0.358$ & $0.439$ & $0.179$ &$0.024$ \\
\hline
\end{tabular}
\end{center}
On peut alors calculer l'espérance
\[
E[X] = 0 \times 0.358 + 1 \times 0.439 + 2 \times 0.179 + 3 \times 0.024 = 0.87
\]
On peut donc estimer qu'il y aura en moyenne $0.87$ personnes qui feront sonner le portique sur les 3 personnes.
\end{enumerate}
\end{solution}
\begin{exercise}[subtitle={Équation puissance}]
Résoudre les équations et inéquations suivantes
\begin{multicols}{2}
\begin{enumerate}
\item $10^x = 8$
\item $10^x = 44$
\item $0.23^x \leq 48$
\item $3 \times 0.81^x = 10$
\end{enumerate}
\end{multicols}
\end{exercise}
\begin{solution}
Les solutions ci-dessous ne sont pas justifiée car l'ordinateur ne sait pas faire. Par contre, vous vous devez savoir justifier vos réponses!
\begin{enumerate}
\item $x = \log(8)$
\item $x = \frac{\log(44)}{\log(10)}$
\item Il faut faire attention quand on divise par un log car ce dernier peut être négatif ce qui est le cas ici. Il faut donc pense à changer le sens de l'inégalité.
$x \geq \frac{\log(48)}{\log(0.23)}$
\item Il faut penser à faire la division à par $3$ avant d'utiliser le log car sinon, on ne peut pas utiliser la formule $\log(a^n) = n\times \log(a)$.
$x = \frac{\log(3.33)}{\log(0.81)}$
\end{enumerate}
\end{solution}
\begin{exercise}[subtitle={Étude de fonctions}]
Soit $f(x) = 9x^3 - 837x^2 + 19872x - 40$ une fonction définie sur $\R$.
\begin{enumerate}
\item Calculer $f'(x)$ la dérivée de $f(x)$.
\item Calculer $f'(46)$ et $f'(16)$.
\item En déduire une forme factorisée de $f'(x)$.
\item Étudier le signe de $f'(x)$ et en déduire les variations de $f(x)$.
\item Est-ce que la fonction $f(x)$ admet un maximum ou un minimum? Si oui, calculer sa valeur.
\end{enumerate}
\end{exercise}
\begin{solution}
\begin{enumerate}
\item Dérivée de $f(x)$: $f'(x) = 27x^2 - 1674x + 19872$
\item
\begin{align*}
f'(46) &= 27 \times 46^{2} - 1674 \times 46 + 19872\\&= 27 \times 2116 - 77004 + 19872\\&= 57132 - 57132\\&= 0
\end{align*}
\begin{align*}
f'(16) &= 27 \times 16^{2} - 1674 \times 16 + 19872\\&= 27 \times 256 - 26784 + 19872\\&= 6912 - 6912\\&= 0
\end{align*}
Donc $x = 46$ et $x=16$ sont des racines de $f'(x) = 27x^2 - 1674x + 19872$.
\item On en déduit la forme factorisée suivante
\[
f'(x) = 27 (x - 46)(x-16)
\]
\item Pas de correction disponible
\item À causes des branches extérieurs, la fonction $f(x)$ n'a pas de maximum ou de minimum.
\end{enumerate}
\end{solution}
%\printsolutionstype{exercise}
\end{document}
%%% Local Variables:
%%% mode: latex
%%% TeX-master: "master"
%%% End:

View File

@ -0,0 +1,203 @@
\documentclass[a5paper,10pt]{article}
\usepackage{myXsim}
\usepackage{tasks}
% Title Page
\title{DM2 \hfill COLASSI Alexis}
\tribe{TST}
\date{\hfillÀ render pour le Mercredi 24 février}
\xsimsetup{
solution/print = false
}
\begin{document}
\maketitle
\begin{exercise}[subtitle={Loi binomiale}]
Trois personnes s'apprêtent à passer le portique de sécurité. On suppose que pour chaque personne la probabilité que le portique sonne est égale à $0.0$.
Soit $X$ la variable aléatoire donnant le nombre de personnes faisant sonner le portique, parmi les 3 personnes de ce groupe.
\begin{enumerate}
\item Tracer l'arbre représentant le situation.
\item Justifier que $X$ suit une loi binomiale dont on précisera les paramètres.
\item Quelle est la probabilité qu'une seule personne fasse sonner le portique?
\item Calculer puis interpréter les probabilités suivantes
\[
P(X = 0) \qquad \qquad P(X \geq 2)
\]
\item Calculer l'espérance de $X$ et interpréter le résultat.
\end{enumerate}
\end{exercise}
\begin{solution}
\begin{enumerate}
\item
\begin{tikzpicture}[sloped]
\node {.}
child {node {$0$}
child {node {$0$}
child {node {$0$}
edge from parent
node[above] {1.0}
}
child {node {$1$}
edge from parent
node[above] {0.0}
}
edge from parent
node[above] {1.0}
}
child[missing] {}
child {node {$1$}
child {node {$0$}
edge from parent
node[above] {1.0}
}
child {node {$1$}
edge from parent
node[above] {0.0}
}
edge from parent
node[above] {1.0}
}
edge from parent
node[above] {1.0}
}
child[missing] {}
child[missing] {}
child[missing] {}
child { node {$1$}
child {node {$0$}
child {node {$0$}
edge from parent
node[above] {1.0}
}
child {node {$1$}
edge from parent
node[above] {0.0}
}
edge from parent
node[above] {1.0}
}
child[missing] {}
child {node {$1$}
child {node {$0$}
edge from parent
node[above] {1.0}
}
child {node {$1$}
edge from parent
node[above] {0.0}
}
edge from parent
node[above] {1.0}
}
edge from parent
node[above] {0.0}
} ;
\end{tikzpicture}
\item Chaque personne a 2 possibilités (1: fait sonner ou 2: ne fait pas sonner) et l'on fait passer 3 personnes ce qui correspond à une répétition identique et aléatoire. On peut donc modéliser la situation par une loi binomiale.
\[
X \sim \mathcal{B}(3; 0.76)
\]
\item Probabilité qu'une seule personne fasse sonner le portique. On voit qu'il y a 3 branches qui correspondent à cette situation dont
\[
P(X = 1) = 3 \times 0.0^1 \times 1.0^2 \approx 0.0
\]
\item
\[
P(X = 0) = 1.0^3 \approx 1.0
\]
\[
P(X \geq 2) = P(X = 2) + P(X = 3) = 3 \times 0.0^2 \times 1.0^1 + 0.0^3 \approx 0.0
\]
\item Il faut d'abord tracer le tableau résumant la loi de probabilité:
\begin{center}
\begin{tabular}{|c|*{4}{c|}}
\hline
Valeur & 0 & 1 & 2 & 3 \\
\hline
Probabilité & $1.0$ & $0.0$ & $0.0$ &$0.0$ \\
\hline
\end{tabular}
\end{center}
On peut alors calculer l'espérance
\[
E[X] = 0 \times 1.0 + 1 \times 0.0 + 2 \times 0.0 + 3 \times 0.0 = 0.0
\]
On peut donc estimer qu'il y aura en moyenne $0.0$ personnes qui feront sonner le portique sur les 3 personnes.
\end{enumerate}
\end{solution}
\begin{exercise}[subtitle={Équation puissance}]
Résoudre les équations et inéquations suivantes
\begin{multicols}{2}
\begin{enumerate}
\item $10^x = 18$
\item $16^x = 40$
\item $0.19^x \leq 17$
\item $6 \times 0.07^x = 45$
\end{enumerate}
\end{multicols}
\end{exercise}
\begin{solution}
Les solutions ci-dessous ne sont pas justifiée car l'ordinateur ne sait pas faire. Par contre, vous vous devez savoir justifier vos réponses!
\begin{enumerate}
\item $x = \log(18)$
\item $x = \frac{\log(40)}{\log(16)}$
\item Il faut faire attention quand on divise par un log car ce dernier peut être négatif ce qui est le cas ici. Il faut donc pense à changer le sens de l'inégalité.
$x \geq \frac{\log(17)}{\log(0.19)}$
\item Il faut penser à faire la division à par $6$ avant d'utiliser le log car sinon, on ne peut pas utiliser la formule $\log(a^n) = n\times \log(a)$.
$x = \frac{\log(7.5)}{\log(0.07)}$
\end{enumerate}
\end{solution}
\begin{exercise}[subtitle={Étude de fonctions}]
Soit $f(x) = 6x^3 - 27x^2 - 972x + 2$ une fonction définie sur $\R$.
\begin{enumerate}
\item Calculer $f'(x)$ la dérivée de $f(x)$.
\item Calculer $f'(9)$ et $f'(-6)$.
\item En déduire une forme factorisée de $f'(x)$.
\item Étudier le signe de $f'(x)$ et en déduire les variations de $f(x)$.
\item Est-ce que la fonction $f(x)$ admet un maximum ou un minimum? Si oui, calculer sa valeur.
\end{enumerate}
\end{exercise}
\begin{solution}
\begin{enumerate}
\item Dérivée de $f(x)$: $f'(x) = 18x^2 - 54x - 972$
\item
\begin{align*}
f'(9) &= 18 \times 9^{2} - 54 \times 9 - 972\\&= 18 \times 81 - 486 - 972\\&= 1458 - 1458\\&= 0
\end{align*}
\begin{align*}
f'(-6) &= 18 \times - 6^{2} - 54(- 6) - 972\\&= 18 \times 36 + 324 - 972\\&= 648 - 648\\&= 0
\end{align*}
Donc $x = 9$ et $x=-6$ sont des racines de $f'(x) = 18x^2 - 54x - 972$.
\item On en déduit la forme factorisée suivante
\[
f'(x) = 18 (x - 9)(x--6)
\]
\item Pas de correction disponible
\item À causes des branches extérieurs, la fonction $f(x)$ n'a pas de maximum ou de minimum.
\end{enumerate}
\end{solution}
%\printsolutionstype{exercise}
\end{document}
%%% Local Variables:
%%% mode: latex
%%% TeX-master: "master"
%%% End:

View File

@ -0,0 +1,203 @@
\documentclass[a5paper,10pt]{article}
\usepackage{myXsim}
\usepackage{tasks}
% Title Page
\title{DM2 \hfill COUBAT Alexis}
\tribe{TST}
\date{\hfillÀ render pour le Mercredi 24 février}
\xsimsetup{
solution/print = false
}
\begin{document}
\maketitle
\begin{exercise}[subtitle={Loi binomiale}]
Trois personnes s'apprêtent à passer le portique de sécurité. On suppose que pour chaque personne la probabilité que le portique sonne est égale à $0.1$.
Soit $X$ la variable aléatoire donnant le nombre de personnes faisant sonner le portique, parmi les 3 personnes de ce groupe.
\begin{enumerate}
\item Tracer l'arbre représentant le situation.
\item Justifier que $X$ suit une loi binomiale dont on précisera les paramètres.
\item Quelle est la probabilité qu'une seule personne fasse sonner le portique?
\item Calculer puis interpréter les probabilités suivantes
\[
P(X = 0) \qquad \qquad P(X \geq 2)
\]
\item Calculer l'espérance de $X$ et interpréter le résultat.
\end{enumerate}
\end{exercise}
\begin{solution}
\begin{enumerate}
\item
\begin{tikzpicture}[sloped]
\node {.}
child {node {$0$}
child {node {$0$}
child {node {$0$}
edge from parent
node[above] {0.9}
}
child {node {$1$}
edge from parent
node[above] {0.1}
}
edge from parent
node[above] {0.9}
}
child[missing] {}
child {node {$1$}
child {node {$0$}
edge from parent
node[above] {0.9}
}
child {node {$1$}
edge from parent
node[above] {0.1}
}
edge from parent
node[above] {0.9}
}
edge from parent
node[above] {0.9}
}
child[missing] {}
child[missing] {}
child[missing] {}
child { node {$1$}
child {node {$0$}
child {node {$0$}
edge from parent
node[above] {0.9}
}
child {node {$1$}
edge from parent
node[above] {0.1}
}
edge from parent
node[above] {0.9}
}
child[missing] {}
child {node {$1$}
child {node {$0$}
edge from parent
node[above] {0.9}
}
child {node {$1$}
edge from parent
node[above] {0.1}
}
edge from parent
node[above] {0.9}
}
edge from parent
node[above] {0.1}
} ;
\end{tikzpicture}
\item Chaque personne a 2 possibilités (1: fait sonner ou 2: ne fait pas sonner) et l'on fait passer 3 personnes ce qui correspond à une répétition identique et aléatoire. On peut donc modéliser la situation par une loi binomiale.
\[
X \sim \mathcal{B}(3; 0.76)
\]
\item Probabilité qu'une seule personne fasse sonner le portique. On voit qu'il y a 3 branches qui correspondent à cette situation dont
\[
P(X = 1) = 3 \times 0.1^1 \times 0.9^2 \approx 0.243
\]
\item
\[
P(X = 0) = 0.9^3 \approx 0.729
\]
\[
P(X \geq 2) = P(X = 2) + P(X = 3) = 3 \times 0.1^2 \times 0.9^1 + 0.1^3 \approx 0.028
\]
\item Il faut d'abord tracer le tableau résumant la loi de probabilité:
\begin{center}
\begin{tabular}{|c|*{4}{c|}}
\hline
Valeur & 0 & 1 & 2 & 3 \\
\hline
Probabilité & $0.729$ & $0.243$ & $0.027$ &$0.001$ \\
\hline
\end{tabular}
\end{center}
On peut alors calculer l'espérance
\[
E[X] = 0 \times 0.729 + 1 \times 0.243 + 2 \times 0.027 + 3 \times 0.001 = 0.3
\]
On peut donc estimer qu'il y aura en moyenne $0.3$ personnes qui feront sonner le portique sur les 3 personnes.
\end{enumerate}
\end{solution}
\begin{exercise}[subtitle={Équation puissance}]
Résoudre les équations et inéquations suivantes
\begin{multicols}{2}
\begin{enumerate}
\item $10^x = 40$
\item $17^x = 2$
\item $0.14^x \leq 21$
\item $3 \times 0.62^x = 26$
\end{enumerate}
\end{multicols}
\end{exercise}
\begin{solution}
Les solutions ci-dessous ne sont pas justifiée car l'ordinateur ne sait pas faire. Par contre, vous vous devez savoir justifier vos réponses!
\begin{enumerate}
\item $x = \log(40)$
\item $x = \frac{\log(2)}{\log(17)}$
\item Il faut faire attention quand on divise par un log car ce dernier peut être négatif ce qui est le cas ici. Il faut donc pense à changer le sens de l'inégalité.
$x \geq \frac{\log(21)}{\log(0.14)}$
\item Il faut penser à faire la division à par $3$ avant d'utiliser le log car sinon, on ne peut pas utiliser la formule $\log(a^n) = n\times \log(a)$.
$x = \frac{\log(8.67)}{\log(0.62)}$
\end{enumerate}
\end{solution}
\begin{exercise}[subtitle={Étude de fonctions}]
Soit $f(x) = 6x^3 - 189x^2 - 828x - 3$ une fonction définie sur $\R$.
\begin{enumerate}
\item Calculer $f'(x)$ la dérivée de $f(x)$.
\item Calculer $f'(23)$ et $f'(-2)$.
\item En déduire une forme factorisée de $f'(x)$.
\item Étudier le signe de $f'(x)$ et en déduire les variations de $f(x)$.
\item Est-ce que la fonction $f(x)$ admet un maximum ou un minimum? Si oui, calculer sa valeur.
\end{enumerate}
\end{exercise}
\begin{solution}
\begin{enumerate}
\item Dérivée de $f(x)$: $f'(x) = 18x^2 - 378x - 828$
\item
\begin{align*}
f'(23) &= 18 \times 23^{2} - 378 \times 23 - 828\\&= 18 \times 529 - 8694 - 828\\&= 9522 - 9522\\&= 0
\end{align*}
\begin{align*}
f'(-2) &= 18 \times - 2^{2} - 378(- 2) - 828\\&= 18 \times 4 + 756 - 828\\&= 72 - 72\\&= 0
\end{align*}
Donc $x = 23$ et $x=-2$ sont des racines de $f'(x) = 18x^2 - 378x - 828$.
\item On en déduit la forme factorisée suivante
\[
f'(x) = 18 (x - 23)(x--2)
\]
\item Pas de correction disponible
\item À causes des branches extérieurs, la fonction $f(x)$ n'a pas de maximum ou de minimum.
\end{enumerate}
\end{solution}
%\printsolutionstype{exercise}
\end{document}
%%% Local Variables:
%%% mode: latex
%%% TeX-master: "master"
%%% End:

View File

@ -0,0 +1,203 @@
\documentclass[a5paper,10pt]{article}
\usepackage{myXsim}
\usepackage{tasks}
% Title Page
\title{DM2 \hfill COULLON Anis}
\tribe{TST}
\date{\hfillÀ render pour le Mercredi 24 février}
\xsimsetup{
solution/print = false
}
\begin{document}
\maketitle
\begin{exercise}[subtitle={Loi binomiale}]
Trois personnes s'apprêtent à passer le portique de sécurité. On suppose que pour chaque personne la probabilité que le portique sonne est égale à $0.37$.
Soit $X$ la variable aléatoire donnant le nombre de personnes faisant sonner le portique, parmi les 3 personnes de ce groupe.
\begin{enumerate}
\item Tracer l'arbre représentant le situation.
\item Justifier que $X$ suit une loi binomiale dont on précisera les paramètres.
\item Quelle est la probabilité qu'une seule personne fasse sonner le portique?
\item Calculer puis interpréter les probabilités suivantes
\[
P(X = 0) \qquad \qquad P(X \geq 2)
\]
\item Calculer l'espérance de $X$ et interpréter le résultat.
\end{enumerate}
\end{exercise}
\begin{solution}
\begin{enumerate}
\item
\begin{tikzpicture}[sloped]
\node {.}
child {node {$0$}
child {node {$0$}
child {node {$0$}
edge from parent
node[above] {0.63}
}
child {node {$1$}
edge from parent
node[above] {0.37}
}
edge from parent
node[above] {0.63}
}
child[missing] {}
child {node {$1$}
child {node {$0$}
edge from parent
node[above] {0.63}
}
child {node {$1$}
edge from parent
node[above] {0.37}
}
edge from parent
node[above] {0.63}
}
edge from parent
node[above] {0.63}
}
child[missing] {}
child[missing] {}
child[missing] {}
child { node {$1$}
child {node {$0$}
child {node {$0$}
edge from parent
node[above] {0.63}
}
child {node {$1$}
edge from parent
node[above] {0.37}
}
edge from parent
node[above] {0.63}
}
child[missing] {}
child {node {$1$}
child {node {$0$}
edge from parent
node[above] {0.63}
}
child {node {$1$}
edge from parent
node[above] {0.37}
}
edge from parent
node[above] {0.63}
}
edge from parent
node[above] {0.37}
} ;
\end{tikzpicture}
\item Chaque personne a 2 possibilités (1: fait sonner ou 2: ne fait pas sonner) et l'on fait passer 3 personnes ce qui correspond à une répétition identique et aléatoire. On peut donc modéliser la situation par une loi binomiale.
\[
X \sim \mathcal{B}(3; 0.76)
\]
\item Probabilité qu'une seule personne fasse sonner le portique. On voit qu'il y a 3 branches qui correspondent à cette situation dont
\[
P(X = 1) = 3 \times 0.37^1 \times 0.63^2 \approx 0.441
\]
\item
\[
P(X = 0) = 0.63^3 \approx 0.25
\]
\[
P(X \geq 2) = P(X = 2) + P(X = 3) = 3 \times 0.37^2 \times 0.63^1 + 0.37^3 \approx 0.31
\]
\item Il faut d'abord tracer le tableau résumant la loi de probabilité:
\begin{center}
\begin{tabular}{|c|*{4}{c|}}
\hline
Valeur & 0 & 1 & 2 & 3 \\
\hline
Probabilité & $0.25$ & $0.441$ & $0.259$ &$0.051$ \\
\hline
\end{tabular}
\end{center}
On peut alors calculer l'espérance
\[
E[X] = 0 \times 0.25 + 1 \times 0.441 + 2 \times 0.259 + 3 \times 0.051 = 1.11
\]
On peut donc estimer qu'il y aura en moyenne $1.11$ personnes qui feront sonner le portique sur les 3 personnes.
\end{enumerate}
\end{solution}
\begin{exercise}[subtitle={Équation puissance}]
Résoudre les équations et inéquations suivantes
\begin{multicols}{2}
\begin{enumerate}
\item $10^x = 10$
\item $19^x = 35$
\item $0.59^x \leq 32$
\item $4 \times 0.92^x = 16$
\end{enumerate}
\end{multicols}
\end{exercise}
\begin{solution}
Les solutions ci-dessous ne sont pas justifiée car l'ordinateur ne sait pas faire. Par contre, vous vous devez savoir justifier vos réponses!
\begin{enumerate}
\item $x = \log(10)$
\item $x = \frac{\log(35)}{\log(19)}$
\item Il faut faire attention quand on divise par un log car ce dernier peut être négatif ce qui est le cas ici. Il faut donc pense à changer le sens de l'inégalité.
$x \geq \frac{\log(32)}{\log(0.59)}$
\item Il faut penser à faire la division à par $4$ avant d'utiliser le log car sinon, on ne peut pas utiliser la formule $\log(a^n) = n\times \log(a)$.
$x = \frac{\log(4.0)}{\log(0.92)}$
\end{enumerate}
\end{solution}
\begin{exercise}[subtitle={Étude de fonctions}]
Soit $f(x) = x^3 - 45x^2 + 648x - 33$ une fonction définie sur $\R$.
\begin{enumerate}
\item Calculer $f'(x)$ la dérivée de $f(x)$.
\item Calculer $f'(18)$ et $f'(12)$.
\item En déduire une forme factorisée de $f'(x)$.
\item Étudier le signe de $f'(x)$ et en déduire les variations de $f(x)$.
\item Est-ce que la fonction $f(x)$ admet un maximum ou un minimum? Si oui, calculer sa valeur.
\end{enumerate}
\end{exercise}
\begin{solution}
\begin{enumerate}
\item Dérivée de $f(x)$: $f'(x) = 3x^2 - 90x + 648$
\item
\begin{align*}
f'(18) &= 3 \times 18^{2} - 90 \times 18 + 648\\&= 3 \times 324 - 1620 + 648\\&= 972 - 972\\&= 0
\end{align*}
\begin{align*}
f'(12) &= 3 \times 12^{2} - 90 \times 12 + 648\\&= 3 \times 144 - 1080 + 648\\&= 432 - 432\\&= 0
\end{align*}
Donc $x = 18$ et $x=12$ sont des racines de $f'(x) = 3x^2 - 90x + 648$.
\item On en déduit la forme factorisée suivante
\[
f'(x) = 3 (x - 18)(x-12)
\]
\item Pas de correction disponible
\item À causes des branches extérieurs, la fonction $f(x)$ n'a pas de maximum ou de minimum.
\end{enumerate}
\end{solution}
%\printsolutionstype{exercise}
\end{document}
%%% Local Variables:
%%% mode: latex
%%% TeX-master: "master"
%%% End:

View File

@ -0,0 +1,203 @@
\documentclass[a5paper,10pt]{article}
\usepackage{myXsim}
\usepackage{tasks}
% Title Page
\title{DM2 \hfill DINGER Sölen}
\tribe{TST}
\date{\hfillÀ render pour le Mercredi 24 février}
\xsimsetup{
solution/print = false
}
\begin{document}
\maketitle
\begin{exercise}[subtitle={Loi binomiale}]
Trois personnes s'apprêtent à passer le portique de sécurité. On suppose que pour chaque personne la probabilité que le portique sonne est égale à $0.53$.
Soit $X$ la variable aléatoire donnant le nombre de personnes faisant sonner le portique, parmi les 3 personnes de ce groupe.
\begin{enumerate}
\item Tracer l'arbre représentant le situation.
\item Justifier que $X$ suit une loi binomiale dont on précisera les paramètres.
\item Quelle est la probabilité qu'une seule personne fasse sonner le portique?
\item Calculer puis interpréter les probabilités suivantes
\[
P(X = 0) \qquad \qquad P(X \geq 2)
\]
\item Calculer l'espérance de $X$ et interpréter le résultat.
\end{enumerate}
\end{exercise}
\begin{solution}
\begin{enumerate}
\item
\begin{tikzpicture}[sloped]
\node {.}
child {node {$0$}
child {node {$0$}
child {node {$0$}
edge from parent
node[above] {0.47}
}
child {node {$1$}
edge from parent
node[above] {0.53}
}
edge from parent
node[above] {0.47}
}
child[missing] {}
child {node {$1$}
child {node {$0$}
edge from parent
node[above] {0.47}
}
child {node {$1$}
edge from parent
node[above] {0.53}
}
edge from parent
node[above] {0.47}
}
edge from parent
node[above] {0.47}
}
child[missing] {}
child[missing] {}
child[missing] {}
child { node {$1$}
child {node {$0$}
child {node {$0$}
edge from parent
node[above] {0.47}
}
child {node {$1$}
edge from parent
node[above] {0.53}
}
edge from parent
node[above] {0.47}
}
child[missing] {}
child {node {$1$}
child {node {$0$}
edge from parent
node[above] {0.47}
}
child {node {$1$}
edge from parent
node[above] {0.53}
}
edge from parent
node[above] {0.47}
}
edge from parent
node[above] {0.53}
} ;
\end{tikzpicture}
\item Chaque personne a 2 possibilités (1: fait sonner ou 2: ne fait pas sonner) et l'on fait passer 3 personnes ce qui correspond à une répétition identique et aléatoire. On peut donc modéliser la situation par une loi binomiale.
\[
X \sim \mathcal{B}(3; 0.76)
\]
\item Probabilité qu'une seule personne fasse sonner le portique. On voit qu'il y a 3 branches qui correspondent à cette situation dont
\[
P(X = 1) = 3 \times 0.53^1 \times 0.47^2 \approx 0.351
\]
\item
\[
P(X = 0) = 0.47^3 \approx 0.104
\]
\[
P(X \geq 2) = P(X = 2) + P(X = 3) = 3 \times 0.53^2 \times 0.47^1 + 0.53^3 \approx 0.545
\]
\item Il faut d'abord tracer le tableau résumant la loi de probabilité:
\begin{center}
\begin{tabular}{|c|*{4}{c|}}
\hline
Valeur & 0 & 1 & 2 & 3 \\
\hline
Probabilité & $0.104$ & $0.351$ & $0.396$ &$0.149$ \\
\hline
\end{tabular}
\end{center}
On peut alors calculer l'espérance
\[
E[X] = 0 \times 0.104 + 1 \times 0.351 + 2 \times 0.396 + 3 \times 0.149 = 1.59
\]
On peut donc estimer qu'il y aura en moyenne $1.59$ personnes qui feront sonner le portique sur les 3 personnes.
\end{enumerate}
\end{solution}
\begin{exercise}[subtitle={Équation puissance}]
Résoudre les équations et inéquations suivantes
\begin{multicols}{2}
\begin{enumerate}
\item $10^x = 22$
\item $20^x = 6$
\item $0.11^x \leq 14$
\item $8 \times 0.45^x = 46$
\end{enumerate}
\end{multicols}
\end{exercise}
\begin{solution}
Les solutions ci-dessous ne sont pas justifiée car l'ordinateur ne sait pas faire. Par contre, vous vous devez savoir justifier vos réponses!
\begin{enumerate}
\item $x = \log(22)$
\item $x = \frac{\log(6)}{\log(20)}$
\item Il faut faire attention quand on divise par un log car ce dernier peut être négatif ce qui est le cas ici. Il faut donc pense à changer le sens de l'inégalité.
$x \geq \frac{\log(14)}{\log(0.11)}$
\item Il faut penser à faire la division à par $8$ avant d'utiliser le log car sinon, on ne peut pas utiliser la formule $\log(a^n) = n\times \log(a)$.
$x = \frac{\log(5.75)}{\log(0.45)}$
\end{enumerate}
\end{solution}
\begin{exercise}[subtitle={Étude de fonctions}]
Soit $f(x) = - 4x^3 - 48x^2 + 2880x + 18$ une fonction définie sur $\R$.
\begin{enumerate}
\item Calculer $f'(x)$ la dérivée de $f(x)$.
\item Calculer $f'(12)$ et $f'(-20)$.
\item En déduire une forme factorisée de $f'(x)$.
\item Étudier le signe de $f'(x)$ et en déduire les variations de $f(x)$.
\item Est-ce que la fonction $f(x)$ admet un maximum ou un minimum? Si oui, calculer sa valeur.
\end{enumerate}
\end{exercise}
\begin{solution}
\begin{enumerate}
\item Dérivée de $f(x)$: $f'(x) = - 12x^2 - 96x + 2880$
\item
\begin{align*}
f'(12) &= - 12 \times 12^{2} - 96 \times 12 + 2880\\&= - 12 \times 144 - 1152 + 2880\\&= - 1728 + 1728\\&= 0
\end{align*}
\begin{align*}
f'(-20) &= - 12 \times - 20^{2} - 96(- 20) + 2880\\&= - 12 \times 400 + 1920 + 2880\\&= - 4800 + 4800\\&= 0
\end{align*}
Donc $x = 12$ et $x=-20$ sont des racines de $f'(x) = - 12x^2 - 96x + 2880$.
\item On en déduit la forme factorisée suivante
\[
f'(x) = -12 (x - 12)(x--20)
\]
\item Pas de correction disponible
\item À causes des branches extérieurs, la fonction $f(x)$ n'a pas de maximum ou de minimum.
\end{enumerate}
\end{solution}
%\printsolutionstype{exercise}
\end{document}
%%% Local Variables:
%%% mode: latex
%%% TeX-master: "master"
%%% End:

View File

@ -0,0 +1,203 @@
\documentclass[a5paper,10pt]{article}
\usepackage{myXsim}
\usepackage{tasks}
% Title Page
\title{DM2 \hfill EYRAUD Cynthia}
\tribe{TST}
\date{\hfillÀ render pour le Mercredi 24 février}
\xsimsetup{
solution/print = false
}
\begin{document}
\maketitle
\begin{exercise}[subtitle={Loi binomiale}]
Trois personnes s'apprêtent à passer le portique de sécurité. On suppose que pour chaque personne la probabilité que le portique sonne est égale à $0.14$.
Soit $X$ la variable aléatoire donnant le nombre de personnes faisant sonner le portique, parmi les 3 personnes de ce groupe.
\begin{enumerate}
\item Tracer l'arbre représentant le situation.
\item Justifier que $X$ suit une loi binomiale dont on précisera les paramètres.
\item Quelle est la probabilité qu'une seule personne fasse sonner le portique?
\item Calculer puis interpréter les probabilités suivantes
\[
P(X = 0) \qquad \qquad P(X \geq 2)
\]
\item Calculer l'espérance de $X$ et interpréter le résultat.
\end{enumerate}
\end{exercise}
\begin{solution}
\begin{enumerate}
\item
\begin{tikzpicture}[sloped]
\node {.}
child {node {$0$}
child {node {$0$}
child {node {$0$}
edge from parent
node[above] {0.86}
}
child {node {$1$}
edge from parent
node[above] {0.14}
}
edge from parent
node[above] {0.86}
}
child[missing] {}
child {node {$1$}
child {node {$0$}
edge from parent
node[above] {0.86}
}
child {node {$1$}
edge from parent
node[above] {0.14}
}
edge from parent
node[above] {0.86}
}
edge from parent
node[above] {0.86}
}
child[missing] {}
child[missing] {}
child[missing] {}
child { node {$1$}
child {node {$0$}
child {node {$0$}
edge from parent
node[above] {0.86}
}
child {node {$1$}
edge from parent
node[above] {0.14}
}
edge from parent
node[above] {0.86}
}
child[missing] {}
child {node {$1$}
child {node {$0$}
edge from parent
node[above] {0.86}
}
child {node {$1$}
edge from parent
node[above] {0.14}
}
edge from parent
node[above] {0.86}
}
edge from parent
node[above] {0.14}
} ;
\end{tikzpicture}
\item Chaque personne a 2 possibilités (1: fait sonner ou 2: ne fait pas sonner) et l'on fait passer 3 personnes ce qui correspond à une répétition identique et aléatoire. On peut donc modéliser la situation par une loi binomiale.
\[
X \sim \mathcal{B}(3; 0.76)
\]
\item Probabilité qu'une seule personne fasse sonner le portique. On voit qu'il y a 3 branches qui correspondent à cette situation dont
\[
P(X = 1) = 3 \times 0.14^1 \times 0.86^2 \approx 0.311
\]
\item
\[
P(X = 0) = 0.86^3 \approx 0.636
\]
\[
P(X \geq 2) = P(X = 2) + P(X = 3) = 3 \times 0.14^2 \times 0.86^1 + 0.14^3 \approx 0.054
\]
\item Il faut d'abord tracer le tableau résumant la loi de probabilité:
\begin{center}
\begin{tabular}{|c|*{4}{c|}}
\hline
Valeur & 0 & 1 & 2 & 3 \\
\hline
Probabilité & $0.636$ & $0.311$ & $0.051$ &$0.003$ \\
\hline
\end{tabular}
\end{center}
On peut alors calculer l'espérance
\[
E[X] = 0 \times 0.636 + 1 \times 0.311 + 2 \times 0.051 + 3 \times 0.003 = 0.42
\]
On peut donc estimer qu'il y aura en moyenne $0.42$ personnes qui feront sonner le portique sur les 3 personnes.
\end{enumerate}
\end{solution}
\begin{exercise}[subtitle={Équation puissance}]
Résoudre les équations et inéquations suivantes
\begin{multicols}{2}
\begin{enumerate}
\item $10^x = 5$
\item $20^x = 40$
\item $0.85^x \leq 38$
\item $2 \times 0.24^x = 24$
\end{enumerate}
\end{multicols}
\end{exercise}
\begin{solution}
Les solutions ci-dessous ne sont pas justifiée car l'ordinateur ne sait pas faire. Par contre, vous vous devez savoir justifier vos réponses!
\begin{enumerate}
\item $x = \log(5)$
\item $x = \frac{\log(40)}{\log(20)}$
\item Il faut faire attention quand on divise par un log car ce dernier peut être négatif ce qui est le cas ici. Il faut donc pense à changer le sens de l'inégalité.
$x \geq \frac{\log(38)}{\log(0.85)}$
\item Il faut penser à faire la division à par $2$ avant d'utiliser le log car sinon, on ne peut pas utiliser la formule $\log(a^n) = n\times \log(a)$.
$x = \frac{\log(12.0)}{\log(0.24)}$
\end{enumerate}
\end{solution}
\begin{exercise}[subtitle={Étude de fonctions}]
Soit $f(x) = - 3x^3 + 49.5x^2 + 5580x - 3$ une fonction définie sur $\R$.
\begin{enumerate}
\item Calculer $f'(x)$ la dérivée de $f(x)$.
\item Calculer $f'(31)$ et $f'(-20)$.
\item En déduire une forme factorisée de $f'(x)$.
\item Étudier le signe de $f'(x)$ et en déduire les variations de $f(x)$.
\item Est-ce que la fonction $f(x)$ admet un maximum ou un minimum? Si oui, calculer sa valeur.
\end{enumerate}
\end{exercise}
\begin{solution}
\begin{enumerate}
\item Dérivée de $f(x)$: $f'(x) = - 9x^2 + 99x + 5580$
\item
\begin{align*}
f'(31) &= - 9 \times 31^{2} + 99 \times 31 + 5580\\&= - 9 \times 961 + 3069 + 5580\\&= - 8649 + 8649\\&= 0
\end{align*}
\begin{align*}
f'(-20) &= - 9 \times - 20^{2} + 99(- 20) + 5580\\&= - 9 \times 400 - 1980 + 5580\\&= - 3600 + 3600\\&= 0
\end{align*}
Donc $x = 31$ et $x=-20$ sont des racines de $f'(x) = - 9x^2 + 99x + 5580$.
\item On en déduit la forme factorisée suivante
\[
f'(x) = -9 (x - 31)(x--20)
\]
\item Pas de correction disponible
\item À causes des branches extérieurs, la fonction $f(x)$ n'a pas de maximum ou de minimum.
\end{enumerate}
\end{solution}
%\printsolutionstype{exercise}
\end{document}
%%% Local Variables:
%%% mode: latex
%%% TeX-master: "master"
%%% End:

View File

@ -0,0 +1,203 @@
\documentclass[a5paper,10pt]{article}
\usepackage{myXsim}
\usepackage{tasks}
% Title Page
\title{DM2 \hfill FERREIRA Léo}
\tribe{TST}
\date{\hfillÀ render pour le Mercredi 24 février}
\xsimsetup{
solution/print = false
}
\begin{document}
\maketitle
\begin{exercise}[subtitle={Loi binomiale}]
Trois personnes s'apprêtent à passer le portique de sécurité. On suppose que pour chaque personne la probabilité que le portique sonne est égale à $0.05$.
Soit $X$ la variable aléatoire donnant le nombre de personnes faisant sonner le portique, parmi les 3 personnes de ce groupe.
\begin{enumerate}
\item Tracer l'arbre représentant le situation.
\item Justifier que $X$ suit une loi binomiale dont on précisera les paramètres.
\item Quelle est la probabilité qu'une seule personne fasse sonner le portique?
\item Calculer puis interpréter les probabilités suivantes
\[
P(X = 0) \qquad \qquad P(X \geq 2)
\]
\item Calculer l'espérance de $X$ et interpréter le résultat.
\end{enumerate}
\end{exercise}
\begin{solution}
\begin{enumerate}
\item
\begin{tikzpicture}[sloped]
\node {.}
child {node {$0$}
child {node {$0$}
child {node {$0$}
edge from parent
node[above] {0.95}
}
child {node {$1$}
edge from parent
node[above] {0.05}
}
edge from parent
node[above] {0.95}
}
child[missing] {}
child {node {$1$}
child {node {$0$}
edge from parent
node[above] {0.95}
}
child {node {$1$}
edge from parent
node[above] {0.05}
}
edge from parent
node[above] {0.95}
}
edge from parent
node[above] {0.95}
}
child[missing] {}
child[missing] {}
child[missing] {}
child { node {$1$}
child {node {$0$}
child {node {$0$}
edge from parent
node[above] {0.95}
}
child {node {$1$}
edge from parent
node[above] {0.05}
}
edge from parent
node[above] {0.95}
}
child[missing] {}
child {node {$1$}
child {node {$0$}
edge from parent
node[above] {0.95}
}
child {node {$1$}
edge from parent
node[above] {0.05}
}
edge from parent
node[above] {0.95}
}
edge from parent
node[above] {0.05}
} ;
\end{tikzpicture}
\item Chaque personne a 2 possibilités (1: fait sonner ou 2: ne fait pas sonner) et l'on fait passer 3 personnes ce qui correspond à une répétition identique et aléatoire. On peut donc modéliser la situation par une loi binomiale.
\[
X \sim \mathcal{B}(3; 0.76)
\]
\item Probabilité qu'une seule personne fasse sonner le portique. On voit qu'il y a 3 branches qui correspondent à cette situation dont
\[
P(X = 1) = 3 \times 0.05^1 \times 0.95^2 \approx 0.135
\]
\item
\[
P(X = 0) = 0.95^3 \approx 0.857
\]
\[
P(X \geq 2) = P(X = 2) + P(X = 3) = 3 \times 0.05^2 \times 0.95^1 + 0.05^3 \approx 0.007
\]
\item Il faut d'abord tracer le tableau résumant la loi de probabilité:
\begin{center}
\begin{tabular}{|c|*{4}{c|}}
\hline
Valeur & 0 & 1 & 2 & 3 \\
\hline
Probabilité & $0.857$ & $0.135$ & $0.007$ &$0.0$ \\
\hline
\end{tabular}
\end{center}
On peut alors calculer l'espérance
\[
E[X] = 0 \times 0.857 + 1 \times 0.135 + 2 \times 0.007 + 3 \times 0.0 = 0.15
\]
On peut donc estimer qu'il y aura en moyenne $0.15$ personnes qui feront sonner le portique sur les 3 personnes.
\end{enumerate}
\end{solution}
\begin{exercise}[subtitle={Équation puissance}]
Résoudre les équations et inéquations suivantes
\begin{multicols}{2}
\begin{enumerate}
\item $10^x = 39$
\item $19^x = 43$
\item $0.24^x \leq 2$
\item $5 \times 0.52^x = 16$
\end{enumerate}
\end{multicols}
\end{exercise}
\begin{solution}
Les solutions ci-dessous ne sont pas justifiée car l'ordinateur ne sait pas faire. Par contre, vous vous devez savoir justifier vos réponses!
\begin{enumerate}
\item $x = \log(39)$
\item $x = \frac{\log(43)}{\log(19)}$
\item Il faut faire attention quand on divise par un log car ce dernier peut être négatif ce qui est le cas ici. Il faut donc pense à changer le sens de l'inégalité.
$x \geq \frac{\log(2)}{\log(0.24)}$
\item Il faut penser à faire la division à par $5$ avant d'utiliser le log car sinon, on ne peut pas utiliser la formule $\log(a^n) = n\times \log(a)$.
$x = \frac{\log(3.2)}{\log(0.52)}$
\end{enumerate}
\end{solution}
\begin{exercise}[subtitle={Étude de fonctions}]
Soit $f(x) = 2x^3 - 171x^2 + 2592x + 12$ une fonction définie sur $\R$.
\begin{enumerate}
\item Calculer $f'(x)$ la dérivée de $f(x)$.
\item Calculer $f'(48)$ et $f'(9)$.
\item En déduire une forme factorisée de $f'(x)$.
\item Étudier le signe de $f'(x)$ et en déduire les variations de $f(x)$.
\item Est-ce que la fonction $f(x)$ admet un maximum ou un minimum? Si oui, calculer sa valeur.
\end{enumerate}
\end{exercise}
\begin{solution}
\begin{enumerate}
\item Dérivée de $f(x)$: $f'(x) = 6x^2 - 342x + 2592$
\item
\begin{align*}
f'(48) &= 6 \times 48^{2} - 342 \times 48 + 2592\\&= 6 \times 2304 - 16416 + 2592\\&= 13824 - 13824\\&= 0
\end{align*}
\begin{align*}
f'(9) &= 6 \times 9^{2} - 342 \times 9 + 2592\\&= 6 \times 81 - 3078 + 2592\\&= 486 - 486\\&= 0
\end{align*}
Donc $x = 48$ et $x=9$ sont des racines de $f'(x) = 6x^2 - 342x + 2592$.
\item On en déduit la forme factorisée suivante
\[
f'(x) = 6 (x - 48)(x-9)
\]
\item Pas de correction disponible
\item À causes des branches extérieurs, la fonction $f(x)$ n'a pas de maximum ou de minimum.
\end{enumerate}
\end{solution}
%\printsolutionstype{exercise}
\end{document}
%%% Local Variables:
%%% mode: latex
%%% TeX-master: "master"
%%% End:

View File

@ -0,0 +1,203 @@
\documentclass[a5paper,10pt]{article}
\usepackage{myXsim}
\usepackage{tasks}
% Title Page
\title{DM2 \hfill FILALI Zakaria}
\tribe{TST}
\date{\hfillÀ render pour le Mercredi 24 février}
\xsimsetup{
solution/print = false
}
\begin{document}
\maketitle
\begin{exercise}[subtitle={Loi binomiale}]
Trois personnes s'apprêtent à passer le portique de sécurité. On suppose que pour chaque personne la probabilité que le portique sonne est égale à $0.01$.
Soit $X$ la variable aléatoire donnant le nombre de personnes faisant sonner le portique, parmi les 3 personnes de ce groupe.
\begin{enumerate}
\item Tracer l'arbre représentant le situation.
\item Justifier que $X$ suit une loi binomiale dont on précisera les paramètres.
\item Quelle est la probabilité qu'une seule personne fasse sonner le portique?
\item Calculer puis interpréter les probabilités suivantes
\[
P(X = 0) \qquad \qquad P(X \geq 2)
\]
\item Calculer l'espérance de $X$ et interpréter le résultat.
\end{enumerate}
\end{exercise}
\begin{solution}
\begin{enumerate}
\item
\begin{tikzpicture}[sloped]
\node {.}
child {node {$0$}
child {node {$0$}
child {node {$0$}
edge from parent
node[above] {0.99}
}
child {node {$1$}
edge from parent
node[above] {0.01}
}
edge from parent
node[above] {0.99}
}
child[missing] {}
child {node {$1$}
child {node {$0$}
edge from parent
node[above] {0.99}
}
child {node {$1$}
edge from parent
node[above] {0.01}
}
edge from parent
node[above] {0.99}
}
edge from parent
node[above] {0.99}
}
child[missing] {}
child[missing] {}
child[missing] {}
child { node {$1$}
child {node {$0$}
child {node {$0$}
edge from parent
node[above] {0.99}
}
child {node {$1$}
edge from parent
node[above] {0.01}
}
edge from parent
node[above] {0.99}
}
child[missing] {}
child {node {$1$}
child {node {$0$}
edge from parent
node[above] {0.99}
}
child {node {$1$}
edge from parent
node[above] {0.01}
}
edge from parent
node[above] {0.99}
}
edge from parent
node[above] {0.01}
} ;
\end{tikzpicture}
\item Chaque personne a 2 possibilités (1: fait sonner ou 2: ne fait pas sonner) et l'on fait passer 3 personnes ce qui correspond à une répétition identique et aléatoire. On peut donc modéliser la situation par une loi binomiale.
\[
X \sim \mathcal{B}(3; 0.76)
\]
\item Probabilité qu'une seule personne fasse sonner le portique. On voit qu'il y a 3 branches qui correspondent à cette situation dont
\[
P(X = 1) = 3 \times 0.01^1 \times 0.99^2 \approx 0.029
\]
\item
\[
P(X = 0) = 0.99^3 \approx 0.97
\]
\[
P(X \geq 2) = P(X = 2) + P(X = 3) = 3 \times 0.01^2 \times 0.99^1 + 0.01^3 \approx 0.0
\]
\item Il faut d'abord tracer le tableau résumant la loi de probabilité:
\begin{center}
\begin{tabular}{|c|*{4}{c|}}
\hline
Valeur & 0 & 1 & 2 & 3 \\
\hline
Probabilité & $0.97$ & $0.029$ & $0.0$ &$0.0$ \\
\hline
\end{tabular}
\end{center}
On peut alors calculer l'espérance
\[
E[X] = 0 \times 0.97 + 1 \times 0.029 + 2 \times 0.0 + 3 \times 0.0 = 0.03
\]
On peut donc estimer qu'il y aura en moyenne $0.03$ personnes qui feront sonner le portique sur les 3 personnes.
\end{enumerate}
\end{solution}
\begin{exercise}[subtitle={Équation puissance}]
Résoudre les équations et inéquations suivantes
\begin{multicols}{2}
\begin{enumerate}
\item $10^x = 39$
\item $19^x = 15$
\item $0.73^x \leq 39$
\item $8 \times 0.15^x = 20$
\end{enumerate}
\end{multicols}
\end{exercise}
\begin{solution}
Les solutions ci-dessous ne sont pas justifiée car l'ordinateur ne sait pas faire. Par contre, vous vous devez savoir justifier vos réponses!
\begin{enumerate}
\item $x = \log(39)$
\item $x = \frac{\log(15)}{\log(19)}$
\item Il faut faire attention quand on divise par un log car ce dernier peut être négatif ce qui est le cas ici. Il faut donc pense à changer le sens de l'inégalité.
$x \geq \frac{\log(39)}{\log(0.73)}$
\item Il faut penser à faire la division à par $8$ avant d'utiliser le log car sinon, on ne peut pas utiliser la formule $\log(a^n) = n\times \log(a)$.
$x = \frac{\log(2.5)}{\log(0.15)}$
\end{enumerate}
\end{solution}
\begin{exercise}[subtitle={Étude de fonctions}]
Soit $f(x) = 8x^3 - 264x^2 - 20160x - 32$ une fonction définie sur $\R$.
\begin{enumerate}
\item Calculer $f'(x)$ la dérivée de $f(x)$.
\item Calculer $f'(42)$ et $f'(-20)$.
\item En déduire une forme factorisée de $f'(x)$.
\item Étudier le signe de $f'(x)$ et en déduire les variations de $f(x)$.
\item Est-ce que la fonction $f(x)$ admet un maximum ou un minimum? Si oui, calculer sa valeur.
\end{enumerate}
\end{exercise}
\begin{solution}
\begin{enumerate}
\item Dérivée de $f(x)$: $f'(x) = 24x^2 - 528x - 20160$
\item
\begin{align*}
f'(42) &= 24 \times 42^{2} - 528 \times 42 - 20160\\&= 24 \times 1764 - 22176 - 20160\\&= 42336 - 42336\\&= 0
\end{align*}
\begin{align*}
f'(-20) &= 24 \times - 20^{2} - 528(- 20) - 20160\\&= 24 \times 400 + 10560 - 20160\\&= 9600 - 9600\\&= 0
\end{align*}
Donc $x = 42$ et $x=-20$ sont des racines de $f'(x) = 24x^2 - 528x - 20160$.
\item On en déduit la forme factorisée suivante
\[
f'(x) = 24 (x - 42)(x--20)
\]
\item Pas de correction disponible
\item À causes des branches extérieurs, la fonction $f(x)$ n'a pas de maximum ou de minimum.
\end{enumerate}
\end{solution}
%\printsolutionstype{exercise}
\end{document}
%%% Local Variables:
%%% mode: latex
%%% TeX-master: "master"
%%% End:

View File

@ -0,0 +1,203 @@
\documentclass[a5paper,10pt]{article}
\usepackage{myXsim}
\usepackage{tasks}
% Title Page
\title{DM2 \hfill FOIGNY Romain}
\tribe{TST}
\date{\hfillÀ render pour le Mercredi 24 février}
\xsimsetup{
solution/print = false
}
\begin{document}
\maketitle
\begin{exercise}[subtitle={Loi binomiale}]
Trois personnes s'apprêtent à passer le portique de sécurité. On suppose que pour chaque personne la probabilité que le portique sonne est égale à $0.25$.
Soit $X$ la variable aléatoire donnant le nombre de personnes faisant sonner le portique, parmi les 3 personnes de ce groupe.
\begin{enumerate}
\item Tracer l'arbre représentant le situation.
\item Justifier que $X$ suit une loi binomiale dont on précisera les paramètres.
\item Quelle est la probabilité qu'une seule personne fasse sonner le portique?
\item Calculer puis interpréter les probabilités suivantes
\[
P(X = 0) \qquad \qquad P(X \geq 2)
\]
\item Calculer l'espérance de $X$ et interpréter le résultat.
\end{enumerate}
\end{exercise}
\begin{solution}
\begin{enumerate}
\item
\begin{tikzpicture}[sloped]
\node {.}
child {node {$0$}
child {node {$0$}
child {node {$0$}
edge from parent
node[above] {0.75}
}
child {node {$1$}
edge from parent
node[above] {0.25}
}
edge from parent
node[above] {0.75}
}
child[missing] {}
child {node {$1$}
child {node {$0$}
edge from parent
node[above] {0.75}
}
child {node {$1$}
edge from parent
node[above] {0.25}
}
edge from parent
node[above] {0.75}
}
edge from parent
node[above] {0.75}
}
child[missing] {}
child[missing] {}
child[missing] {}
child { node {$1$}
child {node {$0$}
child {node {$0$}
edge from parent
node[above] {0.75}
}
child {node {$1$}
edge from parent
node[above] {0.25}
}
edge from parent
node[above] {0.75}
}
child[missing] {}
child {node {$1$}
child {node {$0$}
edge from parent
node[above] {0.75}
}
child {node {$1$}
edge from parent
node[above] {0.25}
}
edge from parent
node[above] {0.75}
}
edge from parent
node[above] {0.25}
} ;
\end{tikzpicture}
\item Chaque personne a 2 possibilités (1: fait sonner ou 2: ne fait pas sonner) et l'on fait passer 3 personnes ce qui correspond à une répétition identique et aléatoire. On peut donc modéliser la situation par une loi binomiale.
\[
X \sim \mathcal{B}(3; 0.76)
\]
\item Probabilité qu'une seule personne fasse sonner le portique. On voit qu'il y a 3 branches qui correspondent à cette situation dont
\[
P(X = 1) = 3 \times 0.25^1 \times 0.75^2 \approx 0.422
\]
\item
\[
P(X = 0) = 0.75^3 \approx 0.422
\]
\[
P(X \geq 2) = P(X = 2) + P(X = 3) = 3 \times 0.25^2 \times 0.75^1 + 0.25^3 \approx 0.157
\]
\item Il faut d'abord tracer le tableau résumant la loi de probabilité:
\begin{center}
\begin{tabular}{|c|*{4}{c|}}
\hline
Valeur & 0 & 1 & 2 & 3 \\
\hline
Probabilité & $0.422$ & $0.422$ & $0.141$ &$0.016$ \\
\hline
\end{tabular}
\end{center}
On peut alors calculer l'espérance
\[
E[X] = 0 \times 0.422 + 1 \times 0.422 + 2 \times 0.141 + 3 \times 0.016 = 0.75
\]
On peut donc estimer qu'il y aura en moyenne $0.75$ personnes qui feront sonner le portique sur les 3 personnes.
\end{enumerate}
\end{solution}
\begin{exercise}[subtitle={Équation puissance}]
Résoudre les équations et inéquations suivantes
\begin{multicols}{2}
\begin{enumerate}
\item $10^x = 13$
\item $17^x = 26$
\item $0.75^x \leq 40$
\item $9 \times 0.86^x = 44$
\end{enumerate}
\end{multicols}
\end{exercise}
\begin{solution}
Les solutions ci-dessous ne sont pas justifiée car l'ordinateur ne sait pas faire. Par contre, vous vous devez savoir justifier vos réponses!
\begin{enumerate}
\item $x = \log(13)$
\item $x = \frac{\log(26)}{\log(17)}$
\item Il faut faire attention quand on divise par un log car ce dernier peut être négatif ce qui est le cas ici. Il faut donc pense à changer le sens de l'inégalité.
$x \geq \frac{\log(40)}{\log(0.75)}$
\item Il faut penser à faire la division à par $9$ avant d'utiliser le log car sinon, on ne peut pas utiliser la formule $\log(a^n) = n\times \log(a)$.
$x = \frac{\log(4.89)}{\log(0.86)}$
\end{enumerate}
\end{solution}
\begin{exercise}[subtitle={Étude de fonctions}]
Soit $f(x) = 2x^3 - 84x^2 + 960x + 28$ une fonction définie sur $\R$.
\begin{enumerate}
\item Calculer $f'(x)$ la dérivée de $f(x)$.
\item Calculer $f'(8)$ et $f'(20)$.
\item En déduire une forme factorisée de $f'(x)$.
\item Étudier le signe de $f'(x)$ et en déduire les variations de $f(x)$.
\item Est-ce que la fonction $f(x)$ admet un maximum ou un minimum? Si oui, calculer sa valeur.
\end{enumerate}
\end{exercise}
\begin{solution}
\begin{enumerate}
\item Dérivée de $f(x)$: $f'(x) = 6x^2 - 168x + 960$
\item
\begin{align*}
f'(8) &= 6 \times 8^{2} - 168 \times 8 + 960\\&= 6 \times 64 - 1344 + 960\\&= 384 - 384\\&= 0
\end{align*}
\begin{align*}
f'(20) &= 6 \times 20^{2} - 168 \times 20 + 960\\&= 6 \times 400 - 3360 + 960\\&= 2400 - 2400\\&= 0
\end{align*}
Donc $x = 8$ et $x=20$ sont des racines de $f'(x) = 6x^2 - 168x + 960$.
\item On en déduit la forme factorisée suivante
\[
f'(x) = 6 (x - 8)(x-20)
\]
\item Pas de correction disponible
\item À causes des branches extérieurs, la fonction $f(x)$ n'a pas de maximum ou de minimum.
\end{enumerate}
\end{solution}
%\printsolutionstype{exercise}
\end{document}
%%% Local Variables:
%%% mode: latex
%%% TeX-master: "master"
%%% End:

View File

@ -0,0 +1,203 @@
\documentclass[a5paper,10pt]{article}
\usepackage{myXsim}
\usepackage{tasks}
% Title Page
\title{DM2 \hfill HIPOLITO DA SILVA Andréa}
\tribe{TST}
\date{\hfillÀ render pour le Mercredi 24 février}
\xsimsetup{
solution/print = false
}
\begin{document}
\maketitle
\begin{exercise}[subtitle={Loi binomiale}]
Trois personnes s'apprêtent à passer le portique de sécurité. On suppose que pour chaque personne la probabilité que le portique sonne est égale à $0.1$.
Soit $X$ la variable aléatoire donnant le nombre de personnes faisant sonner le portique, parmi les 3 personnes de ce groupe.
\begin{enumerate}
\item Tracer l'arbre représentant le situation.
\item Justifier que $X$ suit une loi binomiale dont on précisera les paramètres.
\item Quelle est la probabilité qu'une seule personne fasse sonner le portique?
\item Calculer puis interpréter les probabilités suivantes
\[
P(X = 0) \qquad \qquad P(X \geq 2)
\]
\item Calculer l'espérance de $X$ et interpréter le résultat.
\end{enumerate}
\end{exercise}
\begin{solution}
\begin{enumerate}
\item
\begin{tikzpicture}[sloped]
\node {.}
child {node {$0$}
child {node {$0$}
child {node {$0$}
edge from parent
node[above] {0.9}
}
child {node {$1$}
edge from parent
node[above] {0.1}
}
edge from parent
node[above] {0.9}
}
child[missing] {}
child {node {$1$}
child {node {$0$}
edge from parent
node[above] {0.9}
}
child {node {$1$}
edge from parent
node[above] {0.1}
}
edge from parent
node[above] {0.9}
}
edge from parent
node[above] {0.9}
}
child[missing] {}
child[missing] {}
child[missing] {}
child { node {$1$}
child {node {$0$}
child {node {$0$}
edge from parent
node[above] {0.9}
}
child {node {$1$}
edge from parent
node[above] {0.1}
}
edge from parent
node[above] {0.9}
}
child[missing] {}
child {node {$1$}
child {node {$0$}
edge from parent
node[above] {0.9}
}
child {node {$1$}
edge from parent
node[above] {0.1}
}
edge from parent
node[above] {0.9}
}
edge from parent
node[above] {0.1}
} ;
\end{tikzpicture}
\item Chaque personne a 2 possibilités (1: fait sonner ou 2: ne fait pas sonner) et l'on fait passer 3 personnes ce qui correspond à une répétition identique et aléatoire. On peut donc modéliser la situation par une loi binomiale.
\[
X \sim \mathcal{B}(3; 0.76)
\]
\item Probabilité qu'une seule personne fasse sonner le portique. On voit qu'il y a 3 branches qui correspondent à cette situation dont
\[
P(X = 1) = 3 \times 0.1^1 \times 0.9^2 \approx 0.243
\]
\item
\[
P(X = 0) = 0.9^3 \approx 0.729
\]
\[
P(X \geq 2) = P(X = 2) + P(X = 3) = 3 \times 0.1^2 \times 0.9^1 + 0.1^3 \approx 0.028
\]
\item Il faut d'abord tracer le tableau résumant la loi de probabilité:
\begin{center}
\begin{tabular}{|c|*{4}{c|}}
\hline
Valeur & 0 & 1 & 2 & 3 \\
\hline
Probabilité & $0.729$ & $0.243$ & $0.027$ &$0.001$ \\
\hline
\end{tabular}
\end{center}
On peut alors calculer l'espérance
\[
E[X] = 0 \times 0.729 + 1 \times 0.243 + 2 \times 0.027 + 3 \times 0.001 = 0.3
\]
On peut donc estimer qu'il y aura en moyenne $0.3$ personnes qui feront sonner le portique sur les 3 personnes.
\end{enumerate}
\end{solution}
\begin{exercise}[subtitle={Équation puissance}]
Résoudre les équations et inéquations suivantes
\begin{multicols}{2}
\begin{enumerate}
\item $10^x = 36$
\item $12^x = 44$
\item $0.08^x \leq 43$
\item $4 \times 0.93^x = 46$
\end{enumerate}
\end{multicols}
\end{exercise}
\begin{solution}
Les solutions ci-dessous ne sont pas justifiée car l'ordinateur ne sait pas faire. Par contre, vous vous devez savoir justifier vos réponses!
\begin{enumerate}
\item $x = \log(36)$
\item $x = \frac{\log(44)}{\log(12)}$
\item Il faut faire attention quand on divise par un log car ce dernier peut être négatif ce qui est le cas ici. Il faut donc pense à changer le sens de l'inégalité.
$x \geq \frac{\log(43)}{\log(0.08)}$
\item Il faut penser à faire la division à par $4$ avant d'utiliser le log car sinon, on ne peut pas utiliser la formule $\log(a^n) = n\times \log(a)$.
$x = \frac{\log(11.5)}{\log(0.93)}$
\end{enumerate}
\end{solution}
\begin{exercise}[subtitle={Étude de fonctions}]
Soit $f(x) = - 7x^3 + 294x^2 - 4032x - 33$ une fonction définie sur $\R$.
\begin{enumerate}
\item Calculer $f'(x)$ la dérivée de $f(x)$.
\item Calculer $f'(12)$ et $f'(16)$.
\item En déduire une forme factorisée de $f'(x)$.
\item Étudier le signe de $f'(x)$ et en déduire les variations de $f(x)$.
\item Est-ce que la fonction $f(x)$ admet un maximum ou un minimum? Si oui, calculer sa valeur.
\end{enumerate}
\end{exercise}
\begin{solution}
\begin{enumerate}
\item Dérivée de $f(x)$: $f'(x) = - 21x^2 + 588x - 4032$
\item
\begin{align*}
f'(12) &= - 21 \times 12^{2} + 588 \times 12 - 4032\\&= - 21 \times 144 + 7056 - 4032\\&= - 3024 + 3024\\&= 0
\end{align*}
\begin{align*}
f'(16) &= - 21 \times 16^{2} + 588 \times 16 - 4032\\&= - 21 \times 256 + 9408 - 4032\\&= - 5376 + 5376\\&= 0
\end{align*}
Donc $x = 12$ et $x=16$ sont des racines de $f'(x) = - 21x^2 + 588x - 4032$.
\item On en déduit la forme factorisée suivante
\[
f'(x) = -21 (x - 12)(x-16)
\]
\item Pas de correction disponible
\item À causes des branches extérieurs, la fonction $f(x)$ n'a pas de maximum ou de minimum.
\end{enumerate}
\end{solution}
%\printsolutionstype{exercise}
\end{document}
%%% Local Variables:
%%% mode: latex
%%% TeX-master: "master"
%%% End:

View File

@ -0,0 +1,203 @@
\documentclass[a5paper,10pt]{article}
\usepackage{myXsim}
\usepackage{tasks}
% Title Page
\title{DM2 \hfill HUMBERT Rayan}
\tribe{TST}
\date{\hfillÀ render pour le Mercredi 24 février}
\xsimsetup{
solution/print = false
}
\begin{document}
\maketitle
\begin{exercise}[subtitle={Loi binomiale}]
Trois personnes s'apprêtent à passer le portique de sécurité. On suppose que pour chaque personne la probabilité que le portique sonne est égale à $0.68$.
Soit $X$ la variable aléatoire donnant le nombre de personnes faisant sonner le portique, parmi les 3 personnes de ce groupe.
\begin{enumerate}
\item Tracer l'arbre représentant le situation.
\item Justifier que $X$ suit une loi binomiale dont on précisera les paramètres.
\item Quelle est la probabilité qu'une seule personne fasse sonner le portique?
\item Calculer puis interpréter les probabilités suivantes
\[
P(X = 0) \qquad \qquad P(X \geq 2)
\]
\item Calculer l'espérance de $X$ et interpréter le résultat.
\end{enumerate}
\end{exercise}
\begin{solution}
\begin{enumerate}
\item
\begin{tikzpicture}[sloped]
\node {.}
child {node {$0$}
child {node {$0$}
child {node {$0$}
edge from parent
node[above] {0.32}
}
child {node {$1$}
edge from parent
node[above] {0.68}
}
edge from parent
node[above] {0.32}
}
child[missing] {}
child {node {$1$}
child {node {$0$}
edge from parent
node[above] {0.32}
}
child {node {$1$}
edge from parent
node[above] {0.68}
}
edge from parent
node[above] {0.32}
}
edge from parent
node[above] {0.32}
}
child[missing] {}
child[missing] {}
child[missing] {}
child { node {$1$}
child {node {$0$}
child {node {$0$}
edge from parent
node[above] {0.32}
}
child {node {$1$}
edge from parent
node[above] {0.68}
}
edge from parent
node[above] {0.32}
}
child[missing] {}
child {node {$1$}
child {node {$0$}
edge from parent
node[above] {0.32}
}
child {node {$1$}
edge from parent
node[above] {0.68}
}
edge from parent
node[above] {0.32}
}
edge from parent
node[above] {0.68}
} ;
\end{tikzpicture}
\item Chaque personne a 2 possibilités (1: fait sonner ou 2: ne fait pas sonner) et l'on fait passer 3 personnes ce qui correspond à une répétition identique et aléatoire. On peut donc modéliser la situation par une loi binomiale.
\[
X \sim \mathcal{B}(3; 0.76)
\]
\item Probabilité qu'une seule personne fasse sonner le portique. On voit qu'il y a 3 branches qui correspondent à cette situation dont
\[
P(X = 1) = 3 \times 0.68^1 \times 0.32^2 \approx 0.209
\]
\item
\[
P(X = 0) = 0.32^3 \approx 0.033
\]
\[
P(X \geq 2) = P(X = 2) + P(X = 3) = 3 \times 0.68^2 \times 0.32^1 + 0.68^3 \approx 0.758
\]
\item Il faut d'abord tracer le tableau résumant la loi de probabilité:
\begin{center}
\begin{tabular}{|c|*{4}{c|}}
\hline
Valeur & 0 & 1 & 2 & 3 \\
\hline
Probabilité & $0.033$ & $0.209$ & $0.444$ &$0.314$ \\
\hline
\end{tabular}
\end{center}
On peut alors calculer l'espérance
\[
E[X] = 0 \times 0.033 + 1 \times 0.209 + 2 \times 0.444 + 3 \times 0.314 = 2.04
\]
On peut donc estimer qu'il y aura en moyenne $2.04$ personnes qui feront sonner le portique sur les 3 personnes.
\end{enumerate}
\end{solution}
\begin{exercise}[subtitle={Équation puissance}]
Résoudre les équations et inéquations suivantes
\begin{multicols}{2}
\begin{enumerate}
\item $10^x = 8$
\item $7^x = 21$
\item $0.19^x \leq 12$
\item $9 \times 0.76^x = 6$
\end{enumerate}
\end{multicols}
\end{exercise}
\begin{solution}
Les solutions ci-dessous ne sont pas justifiée car l'ordinateur ne sait pas faire. Par contre, vous vous devez savoir justifier vos réponses!
\begin{enumerate}
\item $x = \log(8)$
\item $x = \frac{\log(21)}{\log(7)}$
\item Il faut faire attention quand on divise par un log car ce dernier peut être négatif ce qui est le cas ici. Il faut donc pense à changer le sens de l'inégalité.
$x \geq \frac{\log(12)}{\log(0.19)}$
\item Il faut penser à faire la division à par $9$ avant d'utiliser le log car sinon, on ne peut pas utiliser la formule $\log(a^n) = n\times \log(a)$.
$x = \frac{\log(0.67)}{\log(0.76)}$
\end{enumerate}
\end{solution}
\begin{exercise}[subtitle={Étude de fonctions}]
Soit $f(x) = 7x^3 - 483x^2 + 10584x - 2$ une fonction définie sur $\R$.
\begin{enumerate}
\item Calculer $f'(x)$ la dérivée de $f(x)$.
\item Calculer $f'(28)$ et $f'(18)$.
\item En déduire une forme factorisée de $f'(x)$.
\item Étudier le signe de $f'(x)$ et en déduire les variations de $f(x)$.
\item Est-ce que la fonction $f(x)$ admet un maximum ou un minimum? Si oui, calculer sa valeur.
\end{enumerate}
\end{exercise}
\begin{solution}
\begin{enumerate}
\item Dérivée de $f(x)$: $f'(x) = 21x^2 - 966x + 10584$
\item
\begin{align*}
f'(28) &= 21 \times 28^{2} - 966 \times 28 + 10584\\&= 21 \times 784 - 27048 + 10584\\&= 16464 - 16464\\&= 0
\end{align*}
\begin{align*}
f'(18) &= 21 \times 18^{2} - 966 \times 18 + 10584\\&= 21 \times 324 - 17388 + 10584\\&= 6804 - 6804\\&= 0
\end{align*}
Donc $x = 28$ et $x=18$ sont des racines de $f'(x) = 21x^2 - 966x + 10584$.
\item On en déduit la forme factorisée suivante
\[
f'(x) = 21 (x - 28)(x-18)
\]
\item Pas de correction disponible
\item À causes des branches extérieurs, la fonction $f(x)$ n'a pas de maximum ou de minimum.
\end{enumerate}
\end{solution}
%\printsolutionstype{exercise}
\end{document}
%%% Local Variables:
%%% mode: latex
%%% TeX-master: "master"
%%% End:

View File

@ -0,0 +1,203 @@
\documentclass[a5paper,10pt]{article}
\usepackage{myXsim}
\usepackage{tasks}
% Title Page
\title{DM2 \hfill MASSON Grace}
\tribe{TST}
\date{\hfillÀ render pour le Mercredi 24 février}
\xsimsetup{
solution/print = false
}
\begin{document}
\maketitle
\begin{exercise}[subtitle={Loi binomiale}]
Trois personnes s'apprêtent à passer le portique de sécurité. On suppose que pour chaque personne la probabilité que le portique sonne est égale à $0.17$.
Soit $X$ la variable aléatoire donnant le nombre de personnes faisant sonner le portique, parmi les 3 personnes de ce groupe.
\begin{enumerate}
\item Tracer l'arbre représentant le situation.
\item Justifier que $X$ suit une loi binomiale dont on précisera les paramètres.
\item Quelle est la probabilité qu'une seule personne fasse sonner le portique?
\item Calculer puis interpréter les probabilités suivantes
\[
P(X = 0) \qquad \qquad P(X \geq 2)
\]
\item Calculer l'espérance de $X$ et interpréter le résultat.
\end{enumerate}
\end{exercise}
\begin{solution}
\begin{enumerate}
\item
\begin{tikzpicture}[sloped]
\node {.}
child {node {$0$}
child {node {$0$}
child {node {$0$}
edge from parent
node[above] {0.83}
}
child {node {$1$}
edge from parent
node[above] {0.17}
}
edge from parent
node[above] {0.83}
}
child[missing] {}
child {node {$1$}
child {node {$0$}
edge from parent
node[above] {0.83}
}
child {node {$1$}
edge from parent
node[above] {0.17}
}
edge from parent
node[above] {0.83}
}
edge from parent
node[above] {0.83}
}
child[missing] {}
child[missing] {}
child[missing] {}
child { node {$1$}
child {node {$0$}
child {node {$0$}
edge from parent
node[above] {0.83}
}
child {node {$1$}
edge from parent
node[above] {0.17}
}
edge from parent
node[above] {0.83}
}
child[missing] {}
child {node {$1$}
child {node {$0$}
edge from parent
node[above] {0.83}
}
child {node {$1$}
edge from parent
node[above] {0.17}
}
edge from parent
node[above] {0.83}
}
edge from parent
node[above] {0.17}
} ;
\end{tikzpicture}
\item Chaque personne a 2 possibilités (1: fait sonner ou 2: ne fait pas sonner) et l'on fait passer 3 personnes ce qui correspond à une répétition identique et aléatoire. On peut donc modéliser la situation par une loi binomiale.
\[
X \sim \mathcal{B}(3; 0.76)
\]
\item Probabilité qu'une seule personne fasse sonner le portique. On voit qu'il y a 3 branches qui correspondent à cette situation dont
\[
P(X = 1) = 3 \times 0.17^1 \times 0.83^2 \approx 0.351
\]
\item
\[
P(X = 0) = 0.83^3 \approx 0.572
\]
\[
P(X \geq 2) = P(X = 2) + P(X = 3) = 3 \times 0.17^2 \times 0.83^1 + 0.17^3 \approx 0.077
\]
\item Il faut d'abord tracer le tableau résumant la loi de probabilité:
\begin{center}
\begin{tabular}{|c|*{4}{c|}}
\hline
Valeur & 0 & 1 & 2 & 3 \\
\hline
Probabilité & $0.572$ & $0.351$ & $0.072$ &$0.005$ \\
\hline
\end{tabular}
\end{center}
On peut alors calculer l'espérance
\[
E[X] = 0 \times 0.572 + 1 \times 0.351 + 2 \times 0.072 + 3 \times 0.005 = 0.51
\]
On peut donc estimer qu'il y aura en moyenne $0.51$ personnes qui feront sonner le portique sur les 3 personnes.
\end{enumerate}
\end{solution}
\begin{exercise}[subtitle={Équation puissance}]
Résoudre les équations et inéquations suivantes
\begin{multicols}{2}
\begin{enumerate}
\item $10^x = 37$
\item $6^x = 32$
\item $0.89^x \leq 36$
\item $7 \times 0.02^x = 44$
\end{enumerate}
\end{multicols}
\end{exercise}
\begin{solution}
Les solutions ci-dessous ne sont pas justifiée car l'ordinateur ne sait pas faire. Par contre, vous vous devez savoir justifier vos réponses!
\begin{enumerate}
\item $x = \log(37)$
\item $x = \frac{\log(32)}{\log(6)}$
\item Il faut faire attention quand on divise par un log car ce dernier peut être négatif ce qui est le cas ici. Il faut donc pense à changer le sens de l'inégalité.
$x \geq \frac{\log(36)}{\log(0.89)}$
\item Il faut penser à faire la division à par $7$ avant d'utiliser le log car sinon, on ne peut pas utiliser la formule $\log(a^n) = n\times \log(a)$.
$x = \frac{\log(6.29)}{\log(0.02)}$
\end{enumerate}
\end{solution}
\begin{exercise}[subtitle={Étude de fonctions}]
Soit $f(x) = - 8x^3 + 96x - 33$ une fonction définie sur $\R$.
\begin{enumerate}
\item Calculer $f'(x)$ la dérivée de $f(x)$.
\item Calculer $f'(2)$ et $f'(-2)$.
\item En déduire une forme factorisée de $f'(x)$.
\item Étudier le signe de $f'(x)$ et en déduire les variations de $f(x)$.
\item Est-ce que la fonction $f(x)$ admet un maximum ou un minimum? Si oui, calculer sa valeur.
\end{enumerate}
\end{exercise}
\begin{solution}
\begin{enumerate}
\item Dérivée de $f(x)$: $f'(x) = - 24x^2 + 96$
\item
\begin{align*}
f'(2) &= - 24 \times 2^{2} + 96\\&= - 24 \times 4 + 96\\&= - 96 + 96\\&= 0
\end{align*}
\begin{align*}
f'(-2) &= - 24 \times - 2^{2} + 96\\&= - 24 \times 4 + 96\\&= - 96 + 96\\&= 0
\end{align*}
Donc $x = 2$ et $x=-2$ sont des racines de $f'(x) = - 24x^2 + 96$.
\item On en déduit la forme factorisée suivante
\[
f'(x) = -24 (x - 2)(x--2)
\]
\item Pas de correction disponible
\item À causes des branches extérieurs, la fonction $f(x)$ n'a pas de maximum ou de minimum.
\end{enumerate}
\end{solution}
%\printsolutionstype{exercise}
\end{document}
%%% Local Variables:
%%% mode: latex
%%% TeX-master: "master"
%%% End:

View File

@ -0,0 +1,203 @@
\documentclass[a5paper,10pt]{article}
\usepackage{myXsim}
\usepackage{tasks}
% Title Page
\title{DM2 \hfill MOKHTARI Nissrine}
\tribe{TST}
\date{\hfillÀ render pour le Mercredi 24 février}
\xsimsetup{
solution/print = false
}
\begin{document}
\maketitle
\begin{exercise}[subtitle={Loi binomiale}]
Trois personnes s'apprêtent à passer le portique de sécurité. On suppose que pour chaque personne la probabilité que le portique sonne est égale à $0.1$.
Soit $X$ la variable aléatoire donnant le nombre de personnes faisant sonner le portique, parmi les 3 personnes de ce groupe.
\begin{enumerate}
\item Tracer l'arbre représentant le situation.
\item Justifier que $X$ suit une loi binomiale dont on précisera les paramètres.
\item Quelle est la probabilité qu'une seule personne fasse sonner le portique?
\item Calculer puis interpréter les probabilités suivantes
\[
P(X = 0) \qquad \qquad P(X \geq 2)
\]
\item Calculer l'espérance de $X$ et interpréter le résultat.
\end{enumerate}
\end{exercise}
\begin{solution}
\begin{enumerate}
\item
\begin{tikzpicture}[sloped]
\node {.}
child {node {$0$}
child {node {$0$}
child {node {$0$}
edge from parent
node[above] {0.9}
}
child {node {$1$}
edge from parent
node[above] {0.1}
}
edge from parent
node[above] {0.9}
}
child[missing] {}
child {node {$1$}
child {node {$0$}
edge from parent
node[above] {0.9}
}
child {node {$1$}
edge from parent
node[above] {0.1}
}
edge from parent
node[above] {0.9}
}
edge from parent
node[above] {0.9}
}
child[missing] {}
child[missing] {}
child[missing] {}
child { node {$1$}
child {node {$0$}
child {node {$0$}
edge from parent
node[above] {0.9}
}
child {node {$1$}
edge from parent
node[above] {0.1}
}
edge from parent
node[above] {0.9}
}
child[missing] {}
child {node {$1$}
child {node {$0$}
edge from parent
node[above] {0.9}
}
child {node {$1$}
edge from parent
node[above] {0.1}
}
edge from parent
node[above] {0.9}
}
edge from parent
node[above] {0.1}
} ;
\end{tikzpicture}
\item Chaque personne a 2 possibilités (1: fait sonner ou 2: ne fait pas sonner) et l'on fait passer 3 personnes ce qui correspond à une répétition identique et aléatoire. On peut donc modéliser la situation par une loi binomiale.
\[
X \sim \mathcal{B}(3; 0.76)
\]
\item Probabilité qu'une seule personne fasse sonner le portique. On voit qu'il y a 3 branches qui correspondent à cette situation dont
\[
P(X = 1) = 3 \times 0.1^1 \times 0.9^2 \approx 0.243
\]
\item
\[
P(X = 0) = 0.9^3 \approx 0.729
\]
\[
P(X \geq 2) = P(X = 2) + P(X = 3) = 3 \times 0.1^2 \times 0.9^1 + 0.1^3 \approx 0.028
\]
\item Il faut d'abord tracer le tableau résumant la loi de probabilité:
\begin{center}
\begin{tabular}{|c|*{4}{c|}}
\hline
Valeur & 0 & 1 & 2 & 3 \\
\hline
Probabilité & $0.729$ & $0.243$ & $0.027$ &$0.001$ \\
\hline
\end{tabular}
\end{center}
On peut alors calculer l'espérance
\[
E[X] = 0 \times 0.729 + 1 \times 0.243 + 2 \times 0.027 + 3 \times 0.001 = 0.3
\]
On peut donc estimer qu'il y aura en moyenne $0.3$ personnes qui feront sonner le portique sur les 3 personnes.
\end{enumerate}
\end{solution}
\begin{exercise}[subtitle={Équation puissance}]
Résoudre les équations et inéquations suivantes
\begin{multicols}{2}
\begin{enumerate}
\item $10^x = 45$
\item $2^x = 25$
\item $0.83^x \leq 40$
\item $8 \times 0.07^x = 42$
\end{enumerate}
\end{multicols}
\end{exercise}
\begin{solution}
Les solutions ci-dessous ne sont pas justifiée car l'ordinateur ne sait pas faire. Par contre, vous vous devez savoir justifier vos réponses!
\begin{enumerate}
\item $x = \log(45)$
\item $x = \frac{\log(25)}{\log(2)}$
\item Il faut faire attention quand on divise par un log car ce dernier peut être négatif ce qui est le cas ici. Il faut donc pense à changer le sens de l'inégalité.
$x \geq \frac{\log(40)}{\log(0.83)}$
\item Il faut penser à faire la division à par $8$ avant d'utiliser le log car sinon, on ne peut pas utiliser la formule $\log(a^n) = n\times \log(a)$.
$x = \frac{\log(5.25)}{\log(0.07)}$
\end{enumerate}
\end{solution}
\begin{exercise}[subtitle={Étude de fonctions}]
Soit $f(x) = - 4x^3 + 384x^2 - 9216x - 22$ une fonction définie sur $\R$.
\begin{enumerate}
\item Calculer $f'(x)$ la dérivée de $f(x)$.
\item Calculer $f'(48)$ et $f'(16)$.
\item En déduire une forme factorisée de $f'(x)$.
\item Étudier le signe de $f'(x)$ et en déduire les variations de $f(x)$.
\item Est-ce que la fonction $f(x)$ admet un maximum ou un minimum? Si oui, calculer sa valeur.
\end{enumerate}
\end{exercise}
\begin{solution}
\begin{enumerate}
\item Dérivée de $f(x)$: $f'(x) = - 12x^2 + 768x - 9216$
\item
\begin{align*}
f'(48) &= - 12 \times 48^{2} + 768 \times 48 - 9216\\&= - 12 \times 2304 + 36864 - 9216\\&= - 27648 + 27648\\&= 0
\end{align*}
\begin{align*}
f'(16) &= - 12 \times 16^{2} + 768 \times 16 - 9216\\&= - 12 \times 256 + 12288 - 9216\\&= - 3072 + 3072\\&= 0
\end{align*}
Donc $x = 48$ et $x=16$ sont des racines de $f'(x) = - 12x^2 + 768x - 9216$.
\item On en déduit la forme factorisée suivante
\[
f'(x) = -12 (x - 48)(x-16)
\]
\item Pas de correction disponible
\item À causes des branches extérieurs, la fonction $f(x)$ n'a pas de maximum ou de minimum.
\end{enumerate}
\end{solution}
%\printsolutionstype{exercise}
\end{document}
%%% Local Variables:
%%% mode: latex
%%% TeX-master: "master"
%%% End:

View File

@ -0,0 +1,203 @@
\documentclass[a5paper,10pt]{article}
\usepackage{myXsim}
\usepackage{tasks}
% Title Page
\title{DM2 \hfill MOUFAQ Amine}
\tribe{TST}
\date{\hfillÀ render pour le Mercredi 24 février}
\xsimsetup{
solution/print = false
}
\begin{document}
\maketitle
\begin{exercise}[subtitle={Loi binomiale}]
Trois personnes s'apprêtent à passer le portique de sécurité. On suppose que pour chaque personne la probabilité que le portique sonne est égale à $0.95$.
Soit $X$ la variable aléatoire donnant le nombre de personnes faisant sonner le portique, parmi les 3 personnes de ce groupe.
\begin{enumerate}
\item Tracer l'arbre représentant le situation.
\item Justifier que $X$ suit une loi binomiale dont on précisera les paramètres.
\item Quelle est la probabilité qu'une seule personne fasse sonner le portique?
\item Calculer puis interpréter les probabilités suivantes
\[
P(X = 0) \qquad \qquad P(X \geq 2)
\]
\item Calculer l'espérance de $X$ et interpréter le résultat.
\end{enumerate}
\end{exercise}
\begin{solution}
\begin{enumerate}
\item
\begin{tikzpicture}[sloped]
\node {.}
child {node {$0$}
child {node {$0$}
child {node {$0$}
edge from parent
node[above] {0.05}
}
child {node {$1$}
edge from parent
node[above] {0.95}
}
edge from parent
node[above] {0.05}
}
child[missing] {}
child {node {$1$}
child {node {$0$}
edge from parent
node[above] {0.05}
}
child {node {$1$}
edge from parent
node[above] {0.95}
}
edge from parent
node[above] {0.05}
}
edge from parent
node[above] {0.05}
}
child[missing] {}
child[missing] {}
child[missing] {}
child { node {$1$}
child {node {$0$}
child {node {$0$}
edge from parent
node[above] {0.05}
}
child {node {$1$}
edge from parent
node[above] {0.95}
}
edge from parent
node[above] {0.05}
}
child[missing] {}
child {node {$1$}
child {node {$0$}
edge from parent
node[above] {0.05}
}
child {node {$1$}
edge from parent
node[above] {0.95}
}
edge from parent
node[above] {0.05}
}
edge from parent
node[above] {0.95}
} ;
\end{tikzpicture}
\item Chaque personne a 2 possibilités (1: fait sonner ou 2: ne fait pas sonner) et l'on fait passer 3 personnes ce qui correspond à une répétition identique et aléatoire. On peut donc modéliser la situation par une loi binomiale.
\[
X \sim \mathcal{B}(3; 0.76)
\]
\item Probabilité qu'une seule personne fasse sonner le portique. On voit qu'il y a 3 branches qui correspondent à cette situation dont
\[
P(X = 1) = 3 \times 0.95^1 \times 0.05^2 \approx 0.007
\]
\item
\[
P(X = 0) = 0.05^3 \approx 0.0
\]
\[
P(X \geq 2) = P(X = 2) + P(X = 3) = 3 \times 0.95^2 \times 0.05^1 + 0.95^3 \approx 0.992
\]
\item Il faut d'abord tracer le tableau résumant la loi de probabilité:
\begin{center}
\begin{tabular}{|c|*{4}{c|}}
\hline
Valeur & 0 & 1 & 2 & 3 \\
\hline
Probabilité & $0.0$ & $0.007$ & $0.135$ &$0.857$ \\
\hline
\end{tabular}
\end{center}
On peut alors calculer l'espérance
\[
E[X] = 0 \times 0.0 + 1 \times 0.007 + 2 \times 0.135 + 3 \times 0.857 = 2.85
\]
On peut donc estimer qu'il y aura en moyenne $2.85$ personnes qui feront sonner le portique sur les 3 personnes.
\end{enumerate}
\end{solution}
\begin{exercise}[subtitle={Équation puissance}]
Résoudre les équations et inéquations suivantes
\begin{multicols}{2}
\begin{enumerate}
\item $10^x = 26$
\item $11^x = 31$
\item $0.01^x \leq 10$
\item $5 \times 0.24^x = 3$
\end{enumerate}
\end{multicols}
\end{exercise}
\begin{solution}
Les solutions ci-dessous ne sont pas justifiée car l'ordinateur ne sait pas faire. Par contre, vous vous devez savoir justifier vos réponses!
\begin{enumerate}
\item $x = \log(26)$
\item $x = \frac{\log(31)}{\log(11)}$
\item Il faut faire attention quand on divise par un log car ce dernier peut être négatif ce qui est le cas ici. Il faut donc pense à changer le sens de l'inégalité.
$x \geq \frac{\log(10)}{\log(0.01)}$
\item Il faut penser à faire la division à par $5$ avant d'utiliser le log car sinon, on ne peut pas utiliser la formule $\log(a^n) = n\times \log(a)$.
$x = \frac{\log(0.6)}{\log(0.24)}$
\end{enumerate}
\end{solution}
\begin{exercise}[subtitle={Étude de fonctions}]
Soit $f(x) = 4x^3 - 192x^2 - 9216x - 28$ une fonction définie sur $\R$.
\begin{enumerate}
\item Calculer $f'(x)$ la dérivée de $f(x)$.
\item Calculer $f'(48)$ et $f'(-16)$.
\item En déduire une forme factorisée de $f'(x)$.
\item Étudier le signe de $f'(x)$ et en déduire les variations de $f(x)$.
\item Est-ce que la fonction $f(x)$ admet un maximum ou un minimum? Si oui, calculer sa valeur.
\end{enumerate}
\end{exercise}
\begin{solution}
\begin{enumerate}
\item Dérivée de $f(x)$: $f'(x) = 12x^2 - 384x - 9216$
\item
\begin{align*}
f'(48) &= 12 \times 48^{2} - 384 \times 48 - 9216\\&= 12 \times 2304 - 18432 - 9216\\&= 27648 - 27648\\&= 0
\end{align*}
\begin{align*}
f'(-16) &= 12 \times - 16^{2} - 384(- 16) - 9216\\&= 12 \times 256 + 6144 - 9216\\&= 3072 - 3072\\&= 0
\end{align*}
Donc $x = 48$ et $x=-16$ sont des racines de $f'(x) = 12x^2 - 384x - 9216$.
\item On en déduit la forme factorisée suivante
\[
f'(x) = 12 (x - 48)(x--16)
\]
\item Pas de correction disponible
\item À causes des branches extérieurs, la fonction $f(x)$ n'a pas de maximum ou de minimum.
\end{enumerate}
\end{solution}
%\printsolutionstype{exercise}
\end{document}
%%% Local Variables:
%%% mode: latex
%%% TeX-master: "master"
%%% End:

View File

@ -0,0 +1,203 @@
\documentclass[a5paper,10pt]{article}
\usepackage{myXsim}
\usepackage{tasks}
% Title Page
\title{DM2 \hfill ONAL Yakub}
\tribe{TST}
\date{\hfillÀ render pour le Mercredi 24 février}
\xsimsetup{
solution/print = false
}
\begin{document}
\maketitle
\begin{exercise}[subtitle={Loi binomiale}]
Trois personnes s'apprêtent à passer le portique de sécurité. On suppose que pour chaque personne la probabilité que le portique sonne est égale à $0.97$.
Soit $X$ la variable aléatoire donnant le nombre de personnes faisant sonner le portique, parmi les 3 personnes de ce groupe.
\begin{enumerate}
\item Tracer l'arbre représentant le situation.
\item Justifier que $X$ suit une loi binomiale dont on précisera les paramètres.
\item Quelle est la probabilité qu'une seule personne fasse sonner le portique?
\item Calculer puis interpréter les probabilités suivantes
\[
P(X = 0) \qquad \qquad P(X \geq 2)
\]
\item Calculer l'espérance de $X$ et interpréter le résultat.
\end{enumerate}
\end{exercise}
\begin{solution}
\begin{enumerate}
\item
\begin{tikzpicture}[sloped]
\node {.}
child {node {$0$}
child {node {$0$}
child {node {$0$}
edge from parent
node[above] {0.03}
}
child {node {$1$}
edge from parent
node[above] {0.97}
}
edge from parent
node[above] {0.03}
}
child[missing] {}
child {node {$1$}
child {node {$0$}
edge from parent
node[above] {0.03}
}
child {node {$1$}
edge from parent
node[above] {0.97}
}
edge from parent
node[above] {0.03}
}
edge from parent
node[above] {0.03}
}
child[missing] {}
child[missing] {}
child[missing] {}
child { node {$1$}
child {node {$0$}
child {node {$0$}
edge from parent
node[above] {0.03}
}
child {node {$1$}
edge from parent
node[above] {0.97}
}
edge from parent
node[above] {0.03}
}
child[missing] {}
child {node {$1$}
child {node {$0$}
edge from parent
node[above] {0.03}
}
child {node {$1$}
edge from parent
node[above] {0.97}
}
edge from parent
node[above] {0.03}
}
edge from parent
node[above] {0.97}
} ;
\end{tikzpicture}
\item Chaque personne a 2 possibilités (1: fait sonner ou 2: ne fait pas sonner) et l'on fait passer 3 personnes ce qui correspond à une répétition identique et aléatoire. On peut donc modéliser la situation par une loi binomiale.
\[
X \sim \mathcal{B}(3; 0.76)
\]
\item Probabilité qu'une seule personne fasse sonner le portique. On voit qu'il y a 3 branches qui correspondent à cette situation dont
\[
P(X = 1) = 3 \times 0.97^1 \times 0.03^2 \approx 0.003
\]
\item
\[
P(X = 0) = 0.03^3 \approx 0.0
\]
\[
P(X \geq 2) = P(X = 2) + P(X = 3) = 3 \times 0.97^2 \times 0.03^1 + 0.97^3 \approx 0.998
\]
\item Il faut d'abord tracer le tableau résumant la loi de probabilité:
\begin{center}
\begin{tabular}{|c|*{4}{c|}}
\hline
Valeur & 0 & 1 & 2 & 3 \\
\hline
Probabilité & $0.0$ & $0.003$ & $0.085$ &$0.913$ \\
\hline
\end{tabular}
\end{center}
On peut alors calculer l'espérance
\[
E[X] = 0 \times 0.0 + 1 \times 0.003 + 2 \times 0.085 + 3 \times 0.913 = 2.91
\]
On peut donc estimer qu'il y aura en moyenne $2.91$ personnes qui feront sonner le portique sur les 3 personnes.
\end{enumerate}
\end{solution}
\begin{exercise}[subtitle={Équation puissance}]
Résoudre les équations et inéquations suivantes
\begin{multicols}{2}
\begin{enumerate}
\item $10^x = 10$
\item $19^x = 8$
\item $0.75^x \leq 2$
\item $5 \times 0.57^x = 24$
\end{enumerate}
\end{multicols}
\end{exercise}
\begin{solution}
Les solutions ci-dessous ne sont pas justifiée car l'ordinateur ne sait pas faire. Par contre, vous vous devez savoir justifier vos réponses!
\begin{enumerate}
\item $x = \log(10)$
\item $x = \frac{\log(8)}{\log(19)}$
\item Il faut faire attention quand on divise par un log car ce dernier peut être négatif ce qui est le cas ici. Il faut donc pense à changer le sens de l'inégalité.
$x \geq \frac{\log(2)}{\log(0.75)}$
\item Il faut penser à faire la division à par $5$ avant d'utiliser le log car sinon, on ne peut pas utiliser la formule $\log(a^n) = n\times \log(a)$.
$x = \frac{\log(4.8)}{\log(0.57)}$
\end{enumerate}
\end{solution}
\begin{exercise}[subtitle={Étude de fonctions}]
Soit $f(x) = - 5x^3 - 7.5x^2 + 1080x - 33$ une fonction définie sur $\R$.
\begin{enumerate}
\item Calculer $f'(x)$ la dérivée de $f(x)$.
\item Calculer $f'(8)$ et $f'(-9)$.
\item En déduire une forme factorisée de $f'(x)$.
\item Étudier le signe de $f'(x)$ et en déduire les variations de $f(x)$.
\item Est-ce que la fonction $f(x)$ admet un maximum ou un minimum? Si oui, calculer sa valeur.
\end{enumerate}
\end{exercise}
\begin{solution}
\begin{enumerate}
\item Dérivée de $f(x)$: $f'(x) = - 15x^2 - 15x + 1080$
\item
\begin{align*}
f'(8) &= - 15 \times 8^{2} - 15 \times 8 + 1080\\&= - 15 \times 64 - 120 + 1080\\&= - 960 + 960\\&= 0
\end{align*}
\begin{align*}
f'(-9) &= - 15 \times - 9^{2} - 15(- 9) + 1080\\&= - 15 \times 81 + 135 + 1080\\&= - 1215 + 1215\\&= 0
\end{align*}
Donc $x = 8$ et $x=-9$ sont des racines de $f'(x) = - 15x^2 - 15x + 1080$.
\item On en déduit la forme factorisée suivante
\[
f'(x) = -15 (x - 8)(x--9)
\]
\item Pas de correction disponible
\item À causes des branches extérieurs, la fonction $f(x)$ n'a pas de maximum ou de minimum.
\end{enumerate}
\end{solution}
%\printsolutionstype{exercise}
\end{document}
%%% Local Variables:
%%% mode: latex
%%% TeX-master: "master"
%%% End:

View File

@ -0,0 +1,203 @@
\documentclass[a5paper,10pt]{article}
\usepackage{myXsim}
\usepackage{tasks}
% Title Page
\title{DM2 \hfill SORIANO Laura}
\tribe{TST}
\date{\hfillÀ render pour le Mercredi 24 février}
\xsimsetup{
solution/print = false
}
\begin{document}
\maketitle
\begin{exercise}[subtitle={Loi binomiale}]
Trois personnes s'apprêtent à passer le portique de sécurité. On suppose que pour chaque personne la probabilité que le portique sonne est égale à $0.76$.
Soit $X$ la variable aléatoire donnant le nombre de personnes faisant sonner le portique, parmi les 3 personnes de ce groupe.
\begin{enumerate}
\item Tracer l'arbre représentant le situation.
\item Justifier que $X$ suit une loi binomiale dont on précisera les paramètres.
\item Quelle est la probabilité qu'une seule personne fasse sonner le portique?
\item Calculer puis interpréter les probabilités suivantes
\[
P(X = 0) \qquad \qquad P(X \geq 2)
\]
\item Calculer l'espérance de $X$ et interpréter le résultat.
\end{enumerate}
\end{exercise}
\begin{solution}
\begin{enumerate}
\item
\begin{tikzpicture}[sloped]
\node {.}
child {node {$0$}
child {node {$0$}
child {node {$0$}
edge from parent
node[above] {0.24}
}
child {node {$1$}
edge from parent
node[above] {0.76}
}
edge from parent
node[above] {0.24}
}
child[missing] {}
child {node {$1$}
child {node {$0$}
edge from parent
node[above] {0.24}
}
child {node {$1$}
edge from parent
node[above] {0.76}
}
edge from parent
node[above] {0.24}
}
edge from parent
node[above] {0.24}
}
child[missing] {}
child[missing] {}
child[missing] {}
child { node {$1$}
child {node {$0$}
child {node {$0$}
edge from parent
node[above] {0.24}
}
child {node {$1$}
edge from parent
node[above] {0.76}
}
edge from parent
node[above] {0.24}
}
child[missing] {}
child {node {$1$}
child {node {$0$}
edge from parent
node[above] {0.24}
}
child {node {$1$}
edge from parent
node[above] {0.76}
}
edge from parent
node[above] {0.24}
}
edge from parent
node[above] {0.76}
} ;
\end{tikzpicture}
\item Chaque personne a 2 possibilités (1: fait sonner ou 2: ne fait pas sonner) et l'on fait passer 3 personnes ce qui correspond à une répétition identique et aléatoire. On peut donc modéliser la situation par une loi binomiale.
\[
X \sim \mathcal{B}(3; 0.76)
\]
\item Probabilité qu'une seule personne fasse sonner le portique. On voit qu'il y a 3 branches qui correspondent à cette situation dont
\[
P(X = 1) = 3 \times 0.76^1 \times 0.24^2 \approx 0.131
\]
\item
\[
P(X = 0) = 0.24^3 \approx 0.014
\]
\[
P(X \geq 2) = P(X = 2) + P(X = 3) = 3 \times 0.76^2 \times 0.24^1 + 0.76^3 \approx 0.855
\]
\item Il faut d'abord tracer le tableau résumant la loi de probabilité:
\begin{center}
\begin{tabular}{|c|*{4}{c|}}
\hline
Valeur & 0 & 1 & 2 & 3 \\
\hline
Probabilité & $0.014$ & $0.131$ & $0.416$ &$0.439$ \\
\hline
\end{tabular}
\end{center}
On peut alors calculer l'espérance
\[
E[X] = 0 \times 0.014 + 1 \times 0.131 + 2 \times 0.416 + 3 \times 0.439 = 2.28
\]
On peut donc estimer qu'il y aura en moyenne $2.28$ personnes qui feront sonner le portique sur les 3 personnes.
\end{enumerate}
\end{solution}
\begin{exercise}[subtitle={Équation puissance}]
Résoudre les équations et inéquations suivantes
\begin{multicols}{2}
\begin{enumerate}
\item $10^x = 14$
\item $14^x = 36$
\item $0.74^x \leq 6$
\item $10 \times 0.11^x = 18$
\end{enumerate}
\end{multicols}
\end{exercise}
\begin{solution}
Les solutions ci-dessous ne sont pas justifiée car l'ordinateur ne sait pas faire. Par contre, vous vous devez savoir justifier vos réponses!
\begin{enumerate}
\item $x = \log(14)$
\item $x = \frac{\log(36)}{\log(14)}$
\item Il faut faire attention quand on divise par un log car ce dernier peut être négatif ce qui est le cas ici. Il faut donc pense à changer le sens de l'inégalité.
$x \geq \frac{\log(6)}{\log(0.74)}$
\item Il faut penser à faire la division à par $10$ avant d'utiliser le log car sinon, on ne peut pas utiliser la formule $\log(a^n) = n\times \log(a)$.
$x = \frac{\log(1.8)}{\log(0.11)}$
\end{enumerate}
\end{solution}
\begin{exercise}[subtitle={Étude de fonctions}]
Soit $f(x) = 4x^3 - 228x^2 + 1632x + 16$ une fonction définie sur $\R$.
\begin{enumerate}
\item Calculer $f'(x)$ la dérivée de $f(x)$.
\item Calculer $f'(34)$ et $f'(4)$.
\item En déduire une forme factorisée de $f'(x)$.
\item Étudier le signe de $f'(x)$ et en déduire les variations de $f(x)$.
\item Est-ce que la fonction $f(x)$ admet un maximum ou un minimum? Si oui, calculer sa valeur.
\end{enumerate}
\end{exercise}
\begin{solution}
\begin{enumerate}
\item Dérivée de $f(x)$: $f'(x) = 12x^2 - 456x + 1632$
\item
\begin{align*}
f'(34) &= 12 \times 34^{2} - 456 \times 34 + 1632\\&= 12 \times 1156 - 15504 + 1632\\&= 13872 - 13872\\&= 0
\end{align*}
\begin{align*}
f'(4) &= 12 \times 4^{2} - 456 \times 4 + 1632\\&= 12 \times 16 - 1824 + 1632\\&= 192 - 192\\&= 0
\end{align*}
Donc $x = 34$ et $x=4$ sont des racines de $f'(x) = 12x^2 - 456x + 1632$.
\item On en déduit la forme factorisée suivante
\[
f'(x) = 12 (x - 34)(x-4)
\]
\item Pas de correction disponible
\item À causes des branches extérieurs, la fonction $f(x)$ n'a pas de maximum ou de minimum.
\end{enumerate}
\end{solution}
%\printsolutionstype{exercise}
\end{document}
%%% Local Variables:
%%% mode: latex
%%% TeX-master: "master"
%%% End:

View File

@ -0,0 +1,203 @@
\documentclass[a5paper,10pt]{article}
\usepackage{myXsim}
\usepackage{tasks}
% Title Page
\title{DM2 \hfill VECCHIO Léa}
\tribe{TST}
\date{\hfillÀ render pour le Mercredi 24 février}
\xsimsetup{
solution/print = false
}
\begin{document}
\maketitle
\begin{exercise}[subtitle={Loi binomiale}]
Trois personnes s'apprêtent à passer le portique de sécurité. On suppose que pour chaque personne la probabilité que le portique sonne est égale à $0.55$.
Soit $X$ la variable aléatoire donnant le nombre de personnes faisant sonner le portique, parmi les 3 personnes de ce groupe.
\begin{enumerate}
\item Tracer l'arbre représentant le situation.
\item Justifier que $X$ suit une loi binomiale dont on précisera les paramètres.
\item Quelle est la probabilité qu'une seule personne fasse sonner le portique?
\item Calculer puis interpréter les probabilités suivantes
\[
P(X = 0) \qquad \qquad P(X \geq 2)
\]
\item Calculer l'espérance de $X$ et interpréter le résultat.
\end{enumerate}
\end{exercise}
\begin{solution}
\begin{enumerate}
\item
\begin{tikzpicture}[sloped]
\node {.}
child {node {$0$}
child {node {$0$}
child {node {$0$}
edge from parent
node[above] {0.45}
}
child {node {$1$}
edge from parent
node[above] {0.55}
}
edge from parent
node[above] {0.45}
}
child[missing] {}
child {node {$1$}
child {node {$0$}
edge from parent
node[above] {0.45}
}
child {node {$1$}
edge from parent
node[above] {0.55}
}
edge from parent
node[above] {0.45}
}
edge from parent
node[above] {0.45}
}
child[missing] {}
child[missing] {}
child[missing] {}
child { node {$1$}
child {node {$0$}
child {node {$0$}
edge from parent
node[above] {0.45}
}
child {node {$1$}
edge from parent
node[above] {0.55}
}
edge from parent
node[above] {0.45}
}
child[missing] {}
child {node {$1$}
child {node {$0$}
edge from parent
node[above] {0.45}
}
child {node {$1$}
edge from parent
node[above] {0.55}
}
edge from parent
node[above] {0.45}
}
edge from parent
node[above] {0.55}
} ;
\end{tikzpicture}
\item Chaque personne a 2 possibilités (1: fait sonner ou 2: ne fait pas sonner) et l'on fait passer 3 personnes ce qui correspond à une répétition identique et aléatoire. On peut donc modéliser la situation par une loi binomiale.
\[
X \sim \mathcal{B}(3; 0.76)
\]
\item Probabilité qu'une seule personne fasse sonner le portique. On voit qu'il y a 3 branches qui correspondent à cette situation dont
\[
P(X = 1) = 3 \times 0.55^1 \times 0.45^2 \approx 0.334
\]
\item
\[
P(X = 0) = 0.45^3 \approx 0.091
\]
\[
P(X \geq 2) = P(X = 2) + P(X = 3) = 3 \times 0.55^2 \times 0.45^1 + 0.55^3 \approx 0.574
\]
\item Il faut d'abord tracer le tableau résumant la loi de probabilité:
\begin{center}
\begin{tabular}{|c|*{4}{c|}}
\hline
Valeur & 0 & 1 & 2 & 3 \\
\hline
Probabilité & $0.091$ & $0.334$ & $0.408$ &$0.166$ \\
\hline
\end{tabular}
\end{center}
On peut alors calculer l'espérance
\[
E[X] = 0 \times 0.091 + 1 \times 0.334 + 2 \times 0.408 + 3 \times 0.166 = 1.65
\]
On peut donc estimer qu'il y aura en moyenne $1.65$ personnes qui feront sonner le portique sur les 3 personnes.
\end{enumerate}
\end{solution}
\begin{exercise}[subtitle={Équation puissance}]
Résoudre les équations et inéquations suivantes
\begin{multicols}{2}
\begin{enumerate}
\item $10^x = 14$
\item $7^x = 47$
\item $0.18^x \leq 40$
\item $3 \times 0.21^x = 25$
\end{enumerate}
\end{multicols}
\end{exercise}
\begin{solution}
Les solutions ci-dessous ne sont pas justifiée car l'ordinateur ne sait pas faire. Par contre, vous vous devez savoir justifier vos réponses!
\begin{enumerate}
\item $x = \log(14)$
\item $x = \frac{\log(47)}{\log(7)}$
\item Il faut faire attention quand on divise par un log car ce dernier peut être négatif ce qui est le cas ici. Il faut donc pense à changer le sens de l'inégalité.
$x \geq \frac{\log(40)}{\log(0.18)}$
\item Il faut penser à faire la division à par $3$ avant d'utiliser le log car sinon, on ne peut pas utiliser la formule $\log(a^n) = n\times \log(a)$.
$x = \frac{\log(8.33)}{\log(0.21)}$
\end{enumerate}
\end{solution}
\begin{exercise}[subtitle={Étude de fonctions}]
Soit $f(x) = x^3 - 64.5x^2 + 360x - 44$ une fonction définie sur $\R$.
\begin{enumerate}
\item Calculer $f'(x)$ la dérivée de $f(x)$.
\item Calculer $f'(40)$ et $f'(3)$.
\item En déduire une forme factorisée de $f'(x)$.
\item Étudier le signe de $f'(x)$ et en déduire les variations de $f(x)$.
\item Est-ce que la fonction $f(x)$ admet un maximum ou un minimum? Si oui, calculer sa valeur.
\end{enumerate}
\end{exercise}
\begin{solution}
\begin{enumerate}
\item Dérivée de $f(x)$: $f'(x) = 3x^2 - 129x + 360$
\item
\begin{align*}
f'(40) &= 3 \times 40^{2} - 129 \times 40 + 360\\&= 3 \times 1600 - 5160 + 360\\&= 4800 - 4800\\&= 0
\end{align*}
\begin{align*}
f'(3) &= 3 \times 3^{2} - 129 \times 3 + 360\\&= 3 \times 9 - 387 + 360\\&= 27 - 27\\&= 0
\end{align*}
Donc $x = 40$ et $x=3$ sont des racines de $f'(x) = 3x^2 - 129x + 360$.
\item On en déduit la forme factorisée suivante
\[
f'(x) = 3 (x - 40)(x-3)
\]
\item Pas de correction disponible
\item À causes des branches extérieurs, la fonction $f(x)$ n'a pas de maximum ou de minimum.
\end{enumerate}
\end{solution}
%\printsolutionstype{exercise}
\end{document}
%%% Local Variables:
%%% mode: latex
%%% TeX-master: "master"
%%% End:

Binary file not shown.

View File

@ -0,0 +1,203 @@
\documentclass[a5paper,10pt]{article}
\usepackage{myXsim}
\usepackage{tasks}
% Title Page
\title{DM2 \hfill AIOUAZ Ahmed}
\tribe{TST}
\date{\hfillÀ render pour le Mercredi 24 février}
\xsimsetup{
solution/print = true
}
\begin{document}
\maketitle
\begin{exercise}[subtitle={Loi binomiale}]
Trois personnes s'apprêtent à passer le portique de sécurité. On suppose que pour chaque personne la probabilité que le portique sonne est égale à $0.8$.
Soit $X$ la variable aléatoire donnant le nombre de personnes faisant sonner le portique, parmi les 3 personnes de ce groupe.
\begin{enumerate}
\item Tracer l'arbre représentant le situation.
\item Justifier que $X$ suit une loi binomiale dont on précisera les paramètres.
\item Quelle est la probabilité qu'une seule personne fasse sonner le portique?
\item Calculer puis interpréter les probabilités suivantes
\[
P(X = 0) \qquad \qquad P(X \geq 2)
\]
\item Calculer l'espérance de $X$ et interpréter le résultat.
\end{enumerate}
\end{exercise}
\begin{solution}
\begin{enumerate}
\item
\begin{tikzpicture}[sloped]
\node {.}
child {node {$0$}
child {node {$0$}
child {node {$0$}
edge from parent
node[above] {0.2}
}
child {node {$1$}
edge from parent
node[above] {0.8}
}
edge from parent
node[above] {0.2}
}
child[missing] {}
child {node {$1$}
child {node {$0$}
edge from parent
node[above] {0.2}
}
child {node {$1$}
edge from parent
node[above] {0.8}
}
edge from parent
node[above] {0.2}
}
edge from parent
node[above] {0.2}
}
child[missing] {}
child[missing] {}
child[missing] {}
child { node {$1$}
child {node {$0$}
child {node {$0$}
edge from parent
node[above] {0.2}
}
child {node {$1$}
edge from parent
node[above] {0.8}
}
edge from parent
node[above] {0.2}
}
child[missing] {}
child {node {$1$}
child {node {$0$}
edge from parent
node[above] {0.2}
}
child {node {$1$}
edge from parent
node[above] {0.8}
}
edge from parent
node[above] {0.2}
}
edge from parent
node[above] {0.8}
} ;
\end{tikzpicture}
\item Chaque personne a 2 possibilités (1: fait sonner ou 2: ne fait pas sonner) et l'on fait passer 3 personnes ce qui correspond à une répétition identique et aléatoire. On peut donc modéliser la situation par une loi binomiale.
\[
X \sim \mathcal{B}(3; 0.76)
\]
\item Probabilité qu'une seule personne fasse sonner le portique. On voit qu'il y a 3 branches qui correspondent à cette situation dont
\[
P(X = 1) = 3 \times 0.8^1 \times 0.2^2 \approx 0.096
\]
\item
\[
P(X = 0) = 0.2^3 \approx 0.008
\]
\[
P(X \geq 2) = P(X = 2) + P(X = 3) = 3 \times 0.8^2 \times 0.2^1 + 0.8^3 \approx 0.896
\]
\item Il faut d'abord tracer le tableau résumant la loi de probabilité:
\begin{center}
\begin{tabular}{|c|*{4}{c|}}
\hline
Valeur & 0 & 1 & 2 & 3 \\
\hline
Probabilité & $0.008$ & $0.096$ & $0.384$ &$0.512$ \\
\hline
\end{tabular}
\end{center}
On peut alors calculer l'espérance
\[
E[X] = 0 \times 0.008 + 1 \times 0.096 + 2 \times 0.384 + 3 \times 0.512 = 2.4
\]
On peut donc estimer qu'il y aura en moyenne $2.4$ personnes qui feront sonner le portique sur les 3 personnes.
\end{enumerate}
\end{solution}
\begin{exercise}[subtitle={Équation puissance}]
Résoudre les équations et inéquations suivantes
\begin{multicols}{2}
\begin{enumerate}
\item $10^x = 27$
\item $20^x = 45$
\item $0.55^x \leq 27$
\item $3 \times 0.31^x = 37$
\end{enumerate}
\end{multicols}
\end{exercise}
\begin{solution}
Les solutions ci-dessous ne sont pas justifiée car l'ordinateur ne sait pas faire. Par contre, vous vous devez savoir justifier vos réponses!
\begin{enumerate}
\item $x = \log(27)$
\item $x = \frac{\log(45)}{\log(20)}$
\item Il faut faire attention quand on divise par un log car ce dernier peut être négatif ce qui est le cas ici. Il faut donc pense à changer le sens de l'inégalité.
$x \geq \frac{\log(27)}{\log(0.55)}$
\item Il faut penser à faire la division à par $3$ avant d'utiliser le log car sinon, on ne peut pas utiliser la formule $\log(a^n) = n\times \log(a)$.
$x = \frac{\log(12.33)}{\log(0.31)}$
\end{enumerate}
\end{solution}
\begin{exercise}[subtitle={Étude de fonctions}]
Soit $f(x) = - 10x^3 + 720x^2 - 11400x - 9$ une fonction définie sur $\R$.
\begin{enumerate}
\item Calculer $f'(x)$ la dérivée de $f(x)$.
\item Calculer $f'(38)$ et $f'(10)$.
\item En déduire une forme factorisée de $f'(x)$.
\item Étudier le signe de $f'(x)$ et en déduire les variations de $f(x)$.
\item Est-ce que la fonction $f(x)$ admet un maximum ou un minimum? Si oui, calculer sa valeur.
\end{enumerate}
\end{exercise}
\begin{solution}
\begin{enumerate}
\item Dérivée de $f(x)$: $f'(x) = - 30x^2 + 1440x - 11400$
\item
\begin{align*}
f'(38) &= - 30 \times 38^{2} + 1440 \times 38 - 11400\\&= - 30 \times 1444 + 54720 - 11400\\&= - 43320 + 43320\\&= 0
\end{align*}
\begin{align*}
f'(10) &= - 30 \times 10^{2} + 1440 \times 10 - 11400\\&= - 30 \times 100 + 14400 - 11400\\&= - 3000 + 3000\\&= 0
\end{align*}
Donc $x = 38$ et $x=10$ sont des racines de $f'(x) = - 30x^2 + 1440x - 11400$.
\item On en déduit la forme factorisée suivante
\[
f'(x) = -30 (x - 38)(x-10)
\]
\item Pas de correction disponible
\item À causes des branches extérieurs, la fonction $f(x)$ n'a pas de maximum ou de minimum.
\end{enumerate}
\end{solution}
%\printsolutionstype{exercise}
\end{document}
%%% Local Variables:
%%% mode: latex
%%% TeX-master: "master"
%%% End:

View File

@ -0,0 +1,203 @@
\documentclass[a5paper,10pt]{article}
\usepackage{myXsim}
\usepackage{tasks}
% Title Page
\title{DM2 \hfill BAHBAH Zakaria}
\tribe{TST}
\date{\hfillÀ render pour le Mercredi 24 février}
\xsimsetup{
solution/print = true
}
\begin{document}
\maketitle
\begin{exercise}[subtitle={Loi binomiale}]
Trois personnes s'apprêtent à passer le portique de sécurité. On suppose que pour chaque personne la probabilité que le portique sonne est égale à $0.25$.
Soit $X$ la variable aléatoire donnant le nombre de personnes faisant sonner le portique, parmi les 3 personnes de ce groupe.
\begin{enumerate}
\item Tracer l'arbre représentant le situation.
\item Justifier que $X$ suit une loi binomiale dont on précisera les paramètres.
\item Quelle est la probabilité qu'une seule personne fasse sonner le portique?
\item Calculer puis interpréter les probabilités suivantes
\[
P(X = 0) \qquad \qquad P(X \geq 2)
\]
\item Calculer l'espérance de $X$ et interpréter le résultat.
\end{enumerate}
\end{exercise}
\begin{solution}
\begin{enumerate}
\item
\begin{tikzpicture}[sloped]
\node {.}
child {node {$0$}
child {node {$0$}
child {node {$0$}
edge from parent
node[above] {0.75}
}
child {node {$1$}
edge from parent
node[above] {0.25}
}
edge from parent
node[above] {0.75}
}
child[missing] {}
child {node {$1$}
child {node {$0$}
edge from parent
node[above] {0.75}
}
child {node {$1$}
edge from parent
node[above] {0.25}
}
edge from parent
node[above] {0.75}
}
edge from parent
node[above] {0.75}
}
child[missing] {}
child[missing] {}
child[missing] {}
child { node {$1$}
child {node {$0$}
child {node {$0$}
edge from parent
node[above] {0.75}
}
child {node {$1$}
edge from parent
node[above] {0.25}
}
edge from parent
node[above] {0.75}
}
child[missing] {}
child {node {$1$}
child {node {$0$}
edge from parent
node[above] {0.75}
}
child {node {$1$}
edge from parent
node[above] {0.25}
}
edge from parent
node[above] {0.75}
}
edge from parent
node[above] {0.25}
} ;
\end{tikzpicture}
\item Chaque personne a 2 possibilités (1: fait sonner ou 2: ne fait pas sonner) et l'on fait passer 3 personnes ce qui correspond à une répétition identique et aléatoire. On peut donc modéliser la situation par une loi binomiale.
\[
X \sim \mathcal{B}(3; 0.76)
\]
\item Probabilité qu'une seule personne fasse sonner le portique. On voit qu'il y a 3 branches qui correspondent à cette situation dont
\[
P(X = 1) = 3 \times 0.25^1 \times 0.75^2 \approx 0.422
\]
\item
\[
P(X = 0) = 0.75^3 \approx 0.422
\]
\[
P(X \geq 2) = P(X = 2) + P(X = 3) = 3 \times 0.25^2 \times 0.75^1 + 0.25^3 \approx 0.157
\]
\item Il faut d'abord tracer le tableau résumant la loi de probabilité:
\begin{center}
\begin{tabular}{|c|*{4}{c|}}
\hline
Valeur & 0 & 1 & 2 & 3 \\
\hline
Probabilité & $0.422$ & $0.422$ & $0.141$ &$0.016$ \\
\hline
\end{tabular}
\end{center}
On peut alors calculer l'espérance
\[
E[X] = 0 \times 0.422 + 1 \times 0.422 + 2 \times 0.141 + 3 \times 0.016 = 0.75
\]
On peut donc estimer qu'il y aura en moyenne $0.75$ personnes qui feront sonner le portique sur les 3 personnes.
\end{enumerate}
\end{solution}
\begin{exercise}[subtitle={Équation puissance}]
Résoudre les équations et inéquations suivantes
\begin{multicols}{2}
\begin{enumerate}
\item $10^x = 6$
\item $4^x = 46$
\item $0.74^x \leq 39$
\item $4 \times 0.52^x = 19$
\end{enumerate}
\end{multicols}
\end{exercise}
\begin{solution}
Les solutions ci-dessous ne sont pas justifiée car l'ordinateur ne sait pas faire. Par contre, vous vous devez savoir justifier vos réponses!
\begin{enumerate}
\item $x = \log(6)$
\item $x = \frac{\log(46)}{\log(4)}$
\item Il faut faire attention quand on divise par un log car ce dernier peut être négatif ce qui est le cas ici. Il faut donc pense à changer le sens de l'inégalité.
$x \geq \frac{\log(39)}{\log(0.74)}$
\item Il faut penser à faire la division à par $4$ avant d'utiliser le log car sinon, on ne peut pas utiliser la formule $\log(a^n) = n\times \log(a)$.
$x = \frac{\log(4.75)}{\log(0.52)}$
\end{enumerate}
\end{solution}
\begin{exercise}[subtitle={Étude de fonctions}]
Soit $f(x) = 10x^3 - 600x^2 - 15000x + 35$ une fonction définie sur $\R$.
\begin{enumerate}
\item Calculer $f'(x)$ la dérivée de $f(x)$.
\item Calculer $f'(50)$ et $f'(-10)$.
\item En déduire une forme factorisée de $f'(x)$.
\item Étudier le signe de $f'(x)$ et en déduire les variations de $f(x)$.
\item Est-ce que la fonction $f(x)$ admet un maximum ou un minimum? Si oui, calculer sa valeur.
\end{enumerate}
\end{exercise}
\begin{solution}
\begin{enumerate}
\item Dérivée de $f(x)$: $f'(x) = 30x^2 - 1200x - 15000$
\item
\begin{align*}
f'(50) &= 30 \times 50^{2} - 1200 \times 50 - 15000\\&= 30 \times 2500 - 60000 - 15000\\&= 75000 - 75000\\&= 0
\end{align*}
\begin{align*}
f'(-10) &= 30 \times - 10^{2} - 1200(- 10) - 15000\\&= 30 \times 100 + 12000 - 15000\\&= 3000 - 3000\\&= 0
\end{align*}
Donc $x = 50$ et $x=-10$ sont des racines de $f'(x) = 30x^2 - 1200x - 15000$.
\item On en déduit la forme factorisée suivante
\[
f'(x) = 30 (x - 50)(x--10)
\]
\item Pas de correction disponible
\item À causes des branches extérieurs, la fonction $f(x)$ n'a pas de maximum ou de minimum.
\end{enumerate}
\end{solution}
%\printsolutionstype{exercise}
\end{document}
%%% Local Variables:
%%% mode: latex
%%% TeX-master: "master"
%%% End:

View File

@ -0,0 +1,203 @@
\documentclass[a5paper,10pt]{article}
\usepackage{myXsim}
\usepackage{tasks}
% Title Page
\title{DM2 \hfill BALLOFFET Kenza}
\tribe{TST}
\date{\hfillÀ render pour le Mercredi 24 février}
\xsimsetup{
solution/print = true
}
\begin{document}
\maketitle
\begin{exercise}[subtitle={Loi binomiale}]
Trois personnes s'apprêtent à passer le portique de sécurité. On suppose que pour chaque personne la probabilité que le portique sonne est égale à $0.83$.
Soit $X$ la variable aléatoire donnant le nombre de personnes faisant sonner le portique, parmi les 3 personnes de ce groupe.
\begin{enumerate}
\item Tracer l'arbre représentant le situation.
\item Justifier que $X$ suit une loi binomiale dont on précisera les paramètres.
\item Quelle est la probabilité qu'une seule personne fasse sonner le portique?
\item Calculer puis interpréter les probabilités suivantes
\[
P(X = 0) \qquad \qquad P(X \geq 2)
\]
\item Calculer l'espérance de $X$ et interpréter le résultat.
\end{enumerate}
\end{exercise}
\begin{solution}
\begin{enumerate}
\item
\begin{tikzpicture}[sloped]
\node {.}
child {node {$0$}
child {node {$0$}
child {node {$0$}
edge from parent
node[above] {0.17}
}
child {node {$1$}
edge from parent
node[above] {0.83}
}
edge from parent
node[above] {0.17}
}
child[missing] {}
child {node {$1$}
child {node {$0$}
edge from parent
node[above] {0.17}
}
child {node {$1$}
edge from parent
node[above] {0.83}
}
edge from parent
node[above] {0.17}
}
edge from parent
node[above] {0.17}
}
child[missing] {}
child[missing] {}
child[missing] {}
child { node {$1$}
child {node {$0$}
child {node {$0$}
edge from parent
node[above] {0.17}
}
child {node {$1$}
edge from parent
node[above] {0.83}
}
edge from parent
node[above] {0.17}
}
child[missing] {}
child {node {$1$}
child {node {$0$}
edge from parent
node[above] {0.17}
}
child {node {$1$}
edge from parent
node[above] {0.83}
}
edge from parent
node[above] {0.17}
}
edge from parent
node[above] {0.83}
} ;
\end{tikzpicture}
\item Chaque personne a 2 possibilités (1: fait sonner ou 2: ne fait pas sonner) et l'on fait passer 3 personnes ce qui correspond à une répétition identique et aléatoire. On peut donc modéliser la situation par une loi binomiale.
\[
X \sim \mathcal{B}(3; 0.76)
\]
\item Probabilité qu'une seule personne fasse sonner le portique. On voit qu'il y a 3 branches qui correspondent à cette situation dont
\[
P(X = 1) = 3 \times 0.83^1 \times 0.17^2 \approx 0.072
\]
\item
\[
P(X = 0) = 0.17^3 \approx 0.005
\]
\[
P(X \geq 2) = P(X = 2) + P(X = 3) = 3 \times 0.83^2 \times 0.17^1 + 0.83^3 \approx 0.923
\]
\item Il faut d'abord tracer le tableau résumant la loi de probabilité:
\begin{center}
\begin{tabular}{|c|*{4}{c|}}
\hline
Valeur & 0 & 1 & 2 & 3 \\
\hline
Probabilité & $0.005$ & $0.072$ & $0.351$ &$0.572$ \\
\hline
\end{tabular}
\end{center}
On peut alors calculer l'espérance
\[
E[X] = 0 \times 0.005 + 1 \times 0.072 + 2 \times 0.351 + 3 \times 0.572 = 2.49
\]
On peut donc estimer qu'il y aura en moyenne $2.49$ personnes qui feront sonner le portique sur les 3 personnes.
\end{enumerate}
\end{solution}
\begin{exercise}[subtitle={Équation puissance}]
Résoudre les équations et inéquations suivantes
\begin{multicols}{2}
\begin{enumerate}
\item $10^x = 15$
\item $6^x = 15$
\item $0.63^x \leq 2$
\item $8 \times 0.02^x = 23$
\end{enumerate}
\end{multicols}
\end{exercise}
\begin{solution}
Les solutions ci-dessous ne sont pas justifiée car l'ordinateur ne sait pas faire. Par contre, vous vous devez savoir justifier vos réponses!
\begin{enumerate}
\item $x = \log(15)$
\item $x = \frac{\log(15)}{\log(6)}$
\item Il faut faire attention quand on divise par un log car ce dernier peut être négatif ce qui est le cas ici. Il faut donc pense à changer le sens de l'inégalité.
$x \geq \frac{\log(2)}{\log(0.63)}$
\item Il faut penser à faire la division à par $8$ avant d'utiliser le log car sinon, on ne peut pas utiliser la formule $\log(a^n) = n\times \log(a)$.
$x = \frac{\log(2.88)}{\log(0.02)}$
\end{enumerate}
\end{solution}
\begin{exercise}[subtitle={Étude de fonctions}]
Soit $f(x) = 3x^3 + 18x^2 - 1485x - 39$ une fonction définie sur $\R$.
\begin{enumerate}
\item Calculer $f'(x)$ la dérivée de $f(x)$.
\item Calculer $f'(11)$ et $f'(-15)$.
\item En déduire une forme factorisée de $f'(x)$.
\item Étudier le signe de $f'(x)$ et en déduire les variations de $f(x)$.
\item Est-ce que la fonction $f(x)$ admet un maximum ou un minimum? Si oui, calculer sa valeur.
\end{enumerate}
\end{exercise}
\begin{solution}
\begin{enumerate}
\item Dérivée de $f(x)$: $f'(x) = 9x^2 + 36x - 1485$
\item
\begin{align*}
f'(11) &= 9 \times 11^{2} + 36 \times 11 - 1485\\&= 9 \times 121 + 396 - 1485\\&= 1089 - 1089\\&= 0
\end{align*}
\begin{align*}
f'(-15) &= 9 \times - 15^{2} + 36(- 15) - 1485\\&= 9 \times 225 - 540 - 1485\\&= 2025 - 2025\\&= 0
\end{align*}
Donc $x = 11$ et $x=-15$ sont des racines de $f'(x) = 9x^2 + 36x - 1485$.
\item On en déduit la forme factorisée suivante
\[
f'(x) = 9 (x - 11)(x--15)
\]
\item Pas de correction disponible
\item À causes des branches extérieurs, la fonction $f(x)$ n'a pas de maximum ou de minimum.
\end{enumerate}
\end{solution}
%\printsolutionstype{exercise}
\end{document}
%%% Local Variables:
%%% mode: latex
%%% TeX-master: "master"
%%% End:

View File

@ -0,0 +1,203 @@
\documentclass[a5paper,10pt]{article}
\usepackage{myXsim}
\usepackage{tasks}
% Title Page
\title{DM2 \hfill BENHATTAL Chakir}
\tribe{TST}
\date{\hfillÀ render pour le Mercredi 24 février}
\xsimsetup{
solution/print = true
}
\begin{document}
\maketitle
\begin{exercise}[subtitle={Loi binomiale}]
Trois personnes s'apprêtent à passer le portique de sécurité. On suppose que pour chaque personne la probabilité que le portique sonne est égale à $0.16$.
Soit $X$ la variable aléatoire donnant le nombre de personnes faisant sonner le portique, parmi les 3 personnes de ce groupe.
\begin{enumerate}
\item Tracer l'arbre représentant le situation.
\item Justifier que $X$ suit une loi binomiale dont on précisera les paramètres.
\item Quelle est la probabilité qu'une seule personne fasse sonner le portique?
\item Calculer puis interpréter les probabilités suivantes
\[
P(X = 0) \qquad \qquad P(X \geq 2)
\]
\item Calculer l'espérance de $X$ et interpréter le résultat.
\end{enumerate}
\end{exercise}
\begin{solution}
\begin{enumerate}
\item
\begin{tikzpicture}[sloped]
\node {.}
child {node {$0$}
child {node {$0$}
child {node {$0$}
edge from parent
node[above] {0.84}
}
child {node {$1$}
edge from parent
node[above] {0.16}
}
edge from parent
node[above] {0.84}
}
child[missing] {}
child {node {$1$}
child {node {$0$}
edge from parent
node[above] {0.84}
}
child {node {$1$}
edge from parent
node[above] {0.16}
}
edge from parent
node[above] {0.84}
}
edge from parent
node[above] {0.84}
}
child[missing] {}
child[missing] {}
child[missing] {}
child { node {$1$}
child {node {$0$}
child {node {$0$}
edge from parent
node[above] {0.84}
}
child {node {$1$}
edge from parent
node[above] {0.16}
}
edge from parent
node[above] {0.84}
}
child[missing] {}
child {node {$1$}
child {node {$0$}
edge from parent
node[above] {0.84}
}
child {node {$1$}
edge from parent
node[above] {0.16}
}
edge from parent
node[above] {0.84}
}
edge from parent
node[above] {0.16}
} ;
\end{tikzpicture}
\item Chaque personne a 2 possibilités (1: fait sonner ou 2: ne fait pas sonner) et l'on fait passer 3 personnes ce qui correspond à une répétition identique et aléatoire. On peut donc modéliser la situation par une loi binomiale.
\[
X \sim \mathcal{B}(3; 0.76)
\]
\item Probabilité qu'une seule personne fasse sonner le portique. On voit qu'il y a 3 branches qui correspondent à cette situation dont
\[
P(X = 1) = 3 \times 0.16^1 \times 0.84^2 \approx 0.339
\]
\item
\[
P(X = 0) = 0.84^3 \approx 0.593
\]
\[
P(X \geq 2) = P(X = 2) + P(X = 3) = 3 \times 0.16^2 \times 0.84^1 + 0.16^3 \approx 0.069
\]
\item Il faut d'abord tracer le tableau résumant la loi de probabilité:
\begin{center}
\begin{tabular}{|c|*{4}{c|}}
\hline
Valeur & 0 & 1 & 2 & 3 \\
\hline
Probabilité & $0.593$ & $0.339$ & $0.065$ &$0.004$ \\
\hline
\end{tabular}
\end{center}
On peut alors calculer l'espérance
\[
E[X] = 0 \times 0.593 + 1 \times 0.339 + 2 \times 0.065 + 3 \times 0.004 = 0.48
\]
On peut donc estimer qu'il y aura en moyenne $0.48$ personnes qui feront sonner le portique sur les 3 personnes.
\end{enumerate}
\end{solution}
\begin{exercise}[subtitle={Équation puissance}]
Résoudre les équations et inéquations suivantes
\begin{multicols}{2}
\begin{enumerate}
\item $10^x = 31$
\item $11^x = 39$
\item $0.65^x \leq 46$
\item $2 \times 0.75^x = 11$
\end{enumerate}
\end{multicols}
\end{exercise}
\begin{solution}
Les solutions ci-dessous ne sont pas justifiée car l'ordinateur ne sait pas faire. Par contre, vous vous devez savoir justifier vos réponses!
\begin{enumerate}
\item $x = \log(31)$
\item $x = \frac{\log(39)}{\log(11)}$
\item Il faut faire attention quand on divise par un log car ce dernier peut être négatif ce qui est le cas ici. Il faut donc pense à changer le sens de l'inégalité.
$x \geq \frac{\log(46)}{\log(0.65)}$
\item Il faut penser à faire la division à par $2$ avant d'utiliser le log car sinon, on ne peut pas utiliser la formule $\log(a^n) = n\times \log(a)$.
$x = \frac{\log(5.5)}{\log(0.75)}$
\end{enumerate}
\end{solution}
\begin{exercise}[subtitle={Étude de fonctions}]
Soit $f(x) = - 4x^3 + 354x^2 - 9120x + 5$ une fonction définie sur $\R$.
\begin{enumerate}
\item Calculer $f'(x)$ la dérivée de $f(x)$.
\item Calculer $f'(40)$ et $f'(19)$.
\item En déduire une forme factorisée de $f'(x)$.
\item Étudier le signe de $f'(x)$ et en déduire les variations de $f(x)$.
\item Est-ce que la fonction $f(x)$ admet un maximum ou un minimum? Si oui, calculer sa valeur.
\end{enumerate}
\end{exercise}
\begin{solution}
\begin{enumerate}
\item Dérivée de $f(x)$: $f'(x) = - 12x^2 + 708x - 9120$
\item
\begin{align*}
f'(40) &= - 12 \times 40^{2} + 708 \times 40 - 9120\\&= - 12 \times 1600 + 28320 - 9120\\&= - 19200 + 19200\\&= 0
\end{align*}
\begin{align*}
f'(19) &= - 12 \times 19^{2} + 708 \times 19 - 9120\\&= - 12 \times 361 + 13452 - 9120\\&= - 4332 + 4332\\&= 0
\end{align*}
Donc $x = 40$ et $x=19$ sont des racines de $f'(x) = - 12x^2 + 708x - 9120$.
\item On en déduit la forme factorisée suivante
\[
f'(x) = -12 (x - 40)(x-19)
\]
\item Pas de correction disponible
\item À causes des branches extérieurs, la fonction $f(x)$ n'a pas de maximum ou de minimum.
\end{enumerate}
\end{solution}
%\printsolutionstype{exercise}
\end{document}
%%% Local Variables:
%%% mode: latex
%%% TeX-master: "master"
%%% End:

View File

@ -0,0 +1,203 @@
\documentclass[a5paper,10pt]{article}
\usepackage{myXsim}
\usepackage{tasks}
% Title Page
\title{DM2 \hfill CLAIN Avinash}
\tribe{TST}
\date{\hfillÀ render pour le Mercredi 24 février}
\xsimsetup{
solution/print = true
}
\begin{document}
\maketitle
\begin{exercise}[subtitle={Loi binomiale}]
Trois personnes s'apprêtent à passer le portique de sécurité. On suppose que pour chaque personne la probabilité que le portique sonne est égale à $0.29$.
Soit $X$ la variable aléatoire donnant le nombre de personnes faisant sonner le portique, parmi les 3 personnes de ce groupe.
\begin{enumerate}
\item Tracer l'arbre représentant le situation.
\item Justifier que $X$ suit une loi binomiale dont on précisera les paramètres.
\item Quelle est la probabilité qu'une seule personne fasse sonner le portique?
\item Calculer puis interpréter les probabilités suivantes
\[
P(X = 0) \qquad \qquad P(X \geq 2)
\]
\item Calculer l'espérance de $X$ et interpréter le résultat.
\end{enumerate}
\end{exercise}
\begin{solution}
\begin{enumerate}
\item
\begin{tikzpicture}[sloped]
\node {.}
child {node {$0$}
child {node {$0$}
child {node {$0$}
edge from parent
node[above] {0.71}
}
child {node {$1$}
edge from parent
node[above] {0.29}
}
edge from parent
node[above] {0.71}
}
child[missing] {}
child {node {$1$}
child {node {$0$}
edge from parent
node[above] {0.71}
}
child {node {$1$}
edge from parent
node[above] {0.29}
}
edge from parent
node[above] {0.71}
}
edge from parent
node[above] {0.71}
}
child[missing] {}
child[missing] {}
child[missing] {}
child { node {$1$}
child {node {$0$}
child {node {$0$}
edge from parent
node[above] {0.71}
}
child {node {$1$}
edge from parent
node[above] {0.29}
}
edge from parent
node[above] {0.71}
}
child[missing] {}
child {node {$1$}
child {node {$0$}
edge from parent
node[above] {0.71}
}
child {node {$1$}
edge from parent
node[above] {0.29}
}
edge from parent
node[above] {0.71}
}
edge from parent
node[above] {0.29}
} ;
\end{tikzpicture}
\item Chaque personne a 2 possibilités (1: fait sonner ou 2: ne fait pas sonner) et l'on fait passer 3 personnes ce qui correspond à une répétition identique et aléatoire. On peut donc modéliser la situation par une loi binomiale.
\[
X \sim \mathcal{B}(3; 0.76)
\]
\item Probabilité qu'une seule personne fasse sonner le portique. On voit qu'il y a 3 branches qui correspondent à cette situation dont
\[
P(X = 1) = 3 \times 0.29^1 \times 0.71^2 \approx 0.439
\]
\item
\[
P(X = 0) = 0.71^3 \approx 0.358
\]
\[
P(X \geq 2) = P(X = 2) + P(X = 3) = 3 \times 0.29^2 \times 0.71^1 + 0.29^3 \approx 0.203
\]
\item Il faut d'abord tracer le tableau résumant la loi de probabilité:
\begin{center}
\begin{tabular}{|c|*{4}{c|}}
\hline
Valeur & 0 & 1 & 2 & 3 \\
\hline
Probabilité & $0.358$ & $0.439$ & $0.179$ &$0.024$ \\
\hline
\end{tabular}
\end{center}
On peut alors calculer l'espérance
\[
E[X] = 0 \times 0.358 + 1 \times 0.439 + 2 \times 0.179 + 3 \times 0.024 = 0.87
\]
On peut donc estimer qu'il y aura en moyenne $0.87$ personnes qui feront sonner le portique sur les 3 personnes.
\end{enumerate}
\end{solution}
\begin{exercise}[subtitle={Équation puissance}]
Résoudre les équations et inéquations suivantes
\begin{multicols}{2}
\begin{enumerate}
\item $10^x = 8$
\item $10^x = 44$
\item $0.23^x \leq 48$
\item $3 \times 0.81^x = 10$
\end{enumerate}
\end{multicols}
\end{exercise}
\begin{solution}
Les solutions ci-dessous ne sont pas justifiée car l'ordinateur ne sait pas faire. Par contre, vous vous devez savoir justifier vos réponses!
\begin{enumerate}
\item $x = \log(8)$
\item $x = \frac{\log(44)}{\log(10)}$
\item Il faut faire attention quand on divise par un log car ce dernier peut être négatif ce qui est le cas ici. Il faut donc pense à changer le sens de l'inégalité.
$x \geq \frac{\log(48)}{\log(0.23)}$
\item Il faut penser à faire la division à par $3$ avant d'utiliser le log car sinon, on ne peut pas utiliser la formule $\log(a^n) = n\times \log(a)$.
$x = \frac{\log(3.33)}{\log(0.81)}$
\end{enumerate}
\end{solution}
\begin{exercise}[subtitle={Étude de fonctions}]
Soit $f(x) = 9x^3 - 837x^2 + 19872x - 40$ une fonction définie sur $\R$.
\begin{enumerate}
\item Calculer $f'(x)$ la dérivée de $f(x)$.
\item Calculer $f'(46)$ et $f'(16)$.
\item En déduire une forme factorisée de $f'(x)$.
\item Étudier le signe de $f'(x)$ et en déduire les variations de $f(x)$.
\item Est-ce que la fonction $f(x)$ admet un maximum ou un minimum? Si oui, calculer sa valeur.
\end{enumerate}
\end{exercise}
\begin{solution}
\begin{enumerate}
\item Dérivée de $f(x)$: $f'(x) = 27x^2 - 1674x + 19872$
\item
\begin{align*}
f'(46) &= 27 \times 46^{2} - 1674 \times 46 + 19872\\&= 27 \times 2116 - 77004 + 19872\\&= 57132 - 57132\\&= 0
\end{align*}
\begin{align*}
f'(16) &= 27 \times 16^{2} - 1674 \times 16 + 19872\\&= 27 \times 256 - 26784 + 19872\\&= 6912 - 6912\\&= 0
\end{align*}
Donc $x = 46$ et $x=16$ sont des racines de $f'(x) = 27x^2 - 1674x + 19872$.
\item On en déduit la forme factorisée suivante
\[
f'(x) = 27 (x - 46)(x-16)
\]
\item Pas de correction disponible
\item À causes des branches extérieurs, la fonction $f(x)$ n'a pas de maximum ou de minimum.
\end{enumerate}
\end{solution}
%\printsolutionstype{exercise}
\end{document}
%%% Local Variables:
%%% mode: latex
%%% TeX-master: "master"
%%% End:

View File

@ -0,0 +1,203 @@
\documentclass[a5paper,10pt]{article}
\usepackage{myXsim}
\usepackage{tasks}
% Title Page
\title{DM2 \hfill COLASSI Alexis}
\tribe{TST}
\date{\hfillÀ render pour le Mercredi 24 février}
\xsimsetup{
solution/print = true
}
\begin{document}
\maketitle
\begin{exercise}[subtitle={Loi binomiale}]
Trois personnes s'apprêtent à passer le portique de sécurité. On suppose que pour chaque personne la probabilité que le portique sonne est égale à $0.0$.
Soit $X$ la variable aléatoire donnant le nombre de personnes faisant sonner le portique, parmi les 3 personnes de ce groupe.
\begin{enumerate}
\item Tracer l'arbre représentant le situation.
\item Justifier que $X$ suit une loi binomiale dont on précisera les paramètres.
\item Quelle est la probabilité qu'une seule personne fasse sonner le portique?
\item Calculer puis interpréter les probabilités suivantes
\[
P(X = 0) \qquad \qquad P(X \geq 2)
\]
\item Calculer l'espérance de $X$ et interpréter le résultat.
\end{enumerate}
\end{exercise}
\begin{solution}
\begin{enumerate}
\item
\begin{tikzpicture}[sloped]
\node {.}
child {node {$0$}
child {node {$0$}
child {node {$0$}
edge from parent
node[above] {1.0}
}
child {node {$1$}
edge from parent
node[above] {0.0}
}
edge from parent
node[above] {1.0}
}
child[missing] {}
child {node {$1$}
child {node {$0$}
edge from parent
node[above] {1.0}
}
child {node {$1$}
edge from parent
node[above] {0.0}
}
edge from parent
node[above] {1.0}
}
edge from parent
node[above] {1.0}
}
child[missing] {}
child[missing] {}
child[missing] {}
child { node {$1$}
child {node {$0$}
child {node {$0$}
edge from parent
node[above] {1.0}
}
child {node {$1$}
edge from parent
node[above] {0.0}
}
edge from parent
node[above] {1.0}
}
child[missing] {}
child {node {$1$}
child {node {$0$}
edge from parent
node[above] {1.0}
}
child {node {$1$}
edge from parent
node[above] {0.0}
}
edge from parent
node[above] {1.0}
}
edge from parent
node[above] {0.0}
} ;
\end{tikzpicture}
\item Chaque personne a 2 possibilités (1: fait sonner ou 2: ne fait pas sonner) et l'on fait passer 3 personnes ce qui correspond à une répétition identique et aléatoire. On peut donc modéliser la situation par une loi binomiale.
\[
X \sim \mathcal{B}(3; 0.76)
\]
\item Probabilité qu'une seule personne fasse sonner le portique. On voit qu'il y a 3 branches qui correspondent à cette situation dont
\[
P(X = 1) = 3 \times 0.0^1 \times 1.0^2 \approx 0.0
\]
\item
\[
P(X = 0) = 1.0^3 \approx 1.0
\]
\[
P(X \geq 2) = P(X = 2) + P(X = 3) = 3 \times 0.0^2 \times 1.0^1 + 0.0^3 \approx 0.0
\]
\item Il faut d'abord tracer le tableau résumant la loi de probabilité:
\begin{center}
\begin{tabular}{|c|*{4}{c|}}
\hline
Valeur & 0 & 1 & 2 & 3 \\
\hline
Probabilité & $1.0$ & $0.0$ & $0.0$ &$0.0$ \\
\hline
\end{tabular}
\end{center}
On peut alors calculer l'espérance
\[
E[X] = 0 \times 1.0 + 1 \times 0.0 + 2 \times 0.0 + 3 \times 0.0 = 0.0
\]
On peut donc estimer qu'il y aura en moyenne $0.0$ personnes qui feront sonner le portique sur les 3 personnes.
\end{enumerate}
\end{solution}
\begin{exercise}[subtitle={Équation puissance}]
Résoudre les équations et inéquations suivantes
\begin{multicols}{2}
\begin{enumerate}
\item $10^x = 18$
\item $16^x = 40$
\item $0.19^x \leq 17$
\item $6 \times 0.07^x = 45$
\end{enumerate}
\end{multicols}
\end{exercise}
\begin{solution}
Les solutions ci-dessous ne sont pas justifiée car l'ordinateur ne sait pas faire. Par contre, vous vous devez savoir justifier vos réponses!
\begin{enumerate}
\item $x = \log(18)$
\item $x = \frac{\log(40)}{\log(16)}$
\item Il faut faire attention quand on divise par un log car ce dernier peut être négatif ce qui est le cas ici. Il faut donc pense à changer le sens de l'inégalité.
$x \geq \frac{\log(17)}{\log(0.19)}$
\item Il faut penser à faire la division à par $6$ avant d'utiliser le log car sinon, on ne peut pas utiliser la formule $\log(a^n) = n\times \log(a)$.
$x = \frac{\log(7.5)}{\log(0.07)}$
\end{enumerate}
\end{solution}
\begin{exercise}[subtitle={Étude de fonctions}]
Soit $f(x) = 6x^3 - 27x^2 - 972x + 2$ une fonction définie sur $\R$.
\begin{enumerate}
\item Calculer $f'(x)$ la dérivée de $f(x)$.
\item Calculer $f'(9)$ et $f'(-6)$.
\item En déduire une forme factorisée de $f'(x)$.
\item Étudier le signe de $f'(x)$ et en déduire les variations de $f(x)$.
\item Est-ce que la fonction $f(x)$ admet un maximum ou un minimum? Si oui, calculer sa valeur.
\end{enumerate}
\end{exercise}
\begin{solution}
\begin{enumerate}
\item Dérivée de $f(x)$: $f'(x) = 18x^2 - 54x - 972$
\item
\begin{align*}
f'(9) &= 18 \times 9^{2} - 54 \times 9 - 972\\&= 18 \times 81 - 486 - 972\\&= 1458 - 1458\\&= 0
\end{align*}
\begin{align*}
f'(-6) &= 18 \times - 6^{2} - 54(- 6) - 972\\&= 18 \times 36 + 324 - 972\\&= 648 - 648\\&= 0
\end{align*}
Donc $x = 9$ et $x=-6$ sont des racines de $f'(x) = 18x^2 - 54x - 972$.
\item On en déduit la forme factorisée suivante
\[
f'(x) = 18 (x - 9)(x--6)
\]
\item Pas de correction disponible
\item À causes des branches extérieurs, la fonction $f(x)$ n'a pas de maximum ou de minimum.
\end{enumerate}
\end{solution}
%\printsolutionstype{exercise}
\end{document}
%%% Local Variables:
%%% mode: latex
%%% TeX-master: "master"
%%% End:

View File

@ -0,0 +1,203 @@
\documentclass[a5paper,10pt]{article}
\usepackage{myXsim}
\usepackage{tasks}
% Title Page
\title{DM2 \hfill COUBAT Alexis}
\tribe{TST}
\date{\hfillÀ render pour le Mercredi 24 février}
\xsimsetup{
solution/print = true
}
\begin{document}
\maketitle
\begin{exercise}[subtitle={Loi binomiale}]
Trois personnes s'apprêtent à passer le portique de sécurité. On suppose que pour chaque personne la probabilité que le portique sonne est égale à $0.1$.
Soit $X$ la variable aléatoire donnant le nombre de personnes faisant sonner le portique, parmi les 3 personnes de ce groupe.
\begin{enumerate}
\item Tracer l'arbre représentant le situation.
\item Justifier que $X$ suit une loi binomiale dont on précisera les paramètres.
\item Quelle est la probabilité qu'une seule personne fasse sonner le portique?
\item Calculer puis interpréter les probabilités suivantes
\[
P(X = 0) \qquad \qquad P(X \geq 2)
\]
\item Calculer l'espérance de $X$ et interpréter le résultat.
\end{enumerate}
\end{exercise}
\begin{solution}
\begin{enumerate}
\item
\begin{tikzpicture}[sloped]
\node {.}
child {node {$0$}
child {node {$0$}
child {node {$0$}
edge from parent
node[above] {0.9}
}
child {node {$1$}
edge from parent
node[above] {0.1}
}
edge from parent
node[above] {0.9}
}
child[missing] {}
child {node {$1$}
child {node {$0$}
edge from parent
node[above] {0.9}
}
child {node {$1$}
edge from parent
node[above] {0.1}
}
edge from parent
node[above] {0.9}
}
edge from parent
node[above] {0.9}
}
child[missing] {}
child[missing] {}
child[missing] {}
child { node {$1$}
child {node {$0$}
child {node {$0$}
edge from parent
node[above] {0.9}
}
child {node {$1$}
edge from parent
node[above] {0.1}
}
edge from parent
node[above] {0.9}
}
child[missing] {}
child {node {$1$}
child {node {$0$}
edge from parent
node[above] {0.9}
}
child {node {$1$}
edge from parent
node[above] {0.1}
}
edge from parent
node[above] {0.9}
}
edge from parent
node[above] {0.1}
} ;
\end{tikzpicture}
\item Chaque personne a 2 possibilités (1: fait sonner ou 2: ne fait pas sonner) et l'on fait passer 3 personnes ce qui correspond à une répétition identique et aléatoire. On peut donc modéliser la situation par une loi binomiale.
\[
X \sim \mathcal{B}(3; 0.76)
\]
\item Probabilité qu'une seule personne fasse sonner le portique. On voit qu'il y a 3 branches qui correspondent à cette situation dont
\[
P(X = 1) = 3 \times 0.1^1 \times 0.9^2 \approx 0.243
\]
\item
\[
P(X = 0) = 0.9^3 \approx 0.729
\]
\[
P(X \geq 2) = P(X = 2) + P(X = 3) = 3 \times 0.1^2 \times 0.9^1 + 0.1^3 \approx 0.028
\]
\item Il faut d'abord tracer le tableau résumant la loi de probabilité:
\begin{center}
\begin{tabular}{|c|*{4}{c|}}
\hline
Valeur & 0 & 1 & 2 & 3 \\
\hline
Probabilité & $0.729$ & $0.243$ & $0.027$ &$0.001$ \\
\hline
\end{tabular}
\end{center}
On peut alors calculer l'espérance
\[
E[X] = 0 \times 0.729 + 1 \times 0.243 + 2 \times 0.027 + 3 \times 0.001 = 0.3
\]
On peut donc estimer qu'il y aura en moyenne $0.3$ personnes qui feront sonner le portique sur les 3 personnes.
\end{enumerate}
\end{solution}
\begin{exercise}[subtitle={Équation puissance}]
Résoudre les équations et inéquations suivantes
\begin{multicols}{2}
\begin{enumerate}
\item $10^x = 40$
\item $17^x = 2$
\item $0.14^x \leq 21$
\item $3 \times 0.62^x = 26$
\end{enumerate}
\end{multicols}
\end{exercise}
\begin{solution}
Les solutions ci-dessous ne sont pas justifiée car l'ordinateur ne sait pas faire. Par contre, vous vous devez savoir justifier vos réponses!
\begin{enumerate}
\item $x = \log(40)$
\item $x = \frac{\log(2)}{\log(17)}$
\item Il faut faire attention quand on divise par un log car ce dernier peut être négatif ce qui est le cas ici. Il faut donc pense à changer le sens de l'inégalité.
$x \geq \frac{\log(21)}{\log(0.14)}$
\item Il faut penser à faire la division à par $3$ avant d'utiliser le log car sinon, on ne peut pas utiliser la formule $\log(a^n) = n\times \log(a)$.
$x = \frac{\log(8.67)}{\log(0.62)}$
\end{enumerate}
\end{solution}
\begin{exercise}[subtitle={Étude de fonctions}]
Soit $f(x) = 6x^3 - 189x^2 - 828x - 3$ une fonction définie sur $\R$.
\begin{enumerate}
\item Calculer $f'(x)$ la dérivée de $f(x)$.
\item Calculer $f'(23)$ et $f'(-2)$.
\item En déduire une forme factorisée de $f'(x)$.
\item Étudier le signe de $f'(x)$ et en déduire les variations de $f(x)$.
\item Est-ce que la fonction $f(x)$ admet un maximum ou un minimum? Si oui, calculer sa valeur.
\end{enumerate}
\end{exercise}
\begin{solution}
\begin{enumerate}
\item Dérivée de $f(x)$: $f'(x) = 18x^2 - 378x - 828$
\item
\begin{align*}
f'(23) &= 18 \times 23^{2} - 378 \times 23 - 828\\&= 18 \times 529 - 8694 - 828\\&= 9522 - 9522\\&= 0
\end{align*}
\begin{align*}
f'(-2) &= 18 \times - 2^{2} - 378(- 2) - 828\\&= 18 \times 4 + 756 - 828\\&= 72 - 72\\&= 0
\end{align*}
Donc $x = 23$ et $x=-2$ sont des racines de $f'(x) = 18x^2 - 378x - 828$.
\item On en déduit la forme factorisée suivante
\[
f'(x) = 18 (x - 23)(x--2)
\]
\item Pas de correction disponible
\item À causes des branches extérieurs, la fonction $f(x)$ n'a pas de maximum ou de minimum.
\end{enumerate}
\end{solution}
%\printsolutionstype{exercise}
\end{document}
%%% Local Variables:
%%% mode: latex
%%% TeX-master: "master"
%%% End:

View File

@ -0,0 +1,203 @@
\documentclass[a5paper,10pt]{article}
\usepackage{myXsim}
\usepackage{tasks}
% Title Page
\title{DM2 \hfill COULLON Anis}
\tribe{TST}
\date{\hfillÀ render pour le Mercredi 24 février}
\xsimsetup{
solution/print = true
}
\begin{document}
\maketitle
\begin{exercise}[subtitle={Loi binomiale}]
Trois personnes s'apprêtent à passer le portique de sécurité. On suppose que pour chaque personne la probabilité que le portique sonne est égale à $0.37$.
Soit $X$ la variable aléatoire donnant le nombre de personnes faisant sonner le portique, parmi les 3 personnes de ce groupe.
\begin{enumerate}
\item Tracer l'arbre représentant le situation.
\item Justifier que $X$ suit une loi binomiale dont on précisera les paramètres.
\item Quelle est la probabilité qu'une seule personne fasse sonner le portique?
\item Calculer puis interpréter les probabilités suivantes
\[
P(X = 0) \qquad \qquad P(X \geq 2)
\]
\item Calculer l'espérance de $X$ et interpréter le résultat.
\end{enumerate}
\end{exercise}
\begin{solution}
\begin{enumerate}
\item
\begin{tikzpicture}[sloped]
\node {.}
child {node {$0$}
child {node {$0$}
child {node {$0$}
edge from parent
node[above] {0.63}
}
child {node {$1$}
edge from parent
node[above] {0.37}
}
edge from parent
node[above] {0.63}
}
child[missing] {}
child {node {$1$}
child {node {$0$}
edge from parent
node[above] {0.63}
}
child {node {$1$}
edge from parent
node[above] {0.37}
}
edge from parent
node[above] {0.63}
}
edge from parent
node[above] {0.63}
}
child[missing] {}
child[missing] {}
child[missing] {}
child { node {$1$}
child {node {$0$}
child {node {$0$}
edge from parent
node[above] {0.63}
}
child {node {$1$}
edge from parent
node[above] {0.37}
}
edge from parent
node[above] {0.63}
}
child[missing] {}
child {node {$1$}
child {node {$0$}
edge from parent
node[above] {0.63}
}
child {node {$1$}
edge from parent
node[above] {0.37}
}
edge from parent
node[above] {0.63}
}
edge from parent
node[above] {0.37}
} ;
\end{tikzpicture}
\item Chaque personne a 2 possibilités (1: fait sonner ou 2: ne fait pas sonner) et l'on fait passer 3 personnes ce qui correspond à une répétition identique et aléatoire. On peut donc modéliser la situation par une loi binomiale.
\[
X \sim \mathcal{B}(3; 0.76)
\]
\item Probabilité qu'une seule personne fasse sonner le portique. On voit qu'il y a 3 branches qui correspondent à cette situation dont
\[
P(X = 1) = 3 \times 0.37^1 \times 0.63^2 \approx 0.441
\]
\item
\[
P(X = 0) = 0.63^3 \approx 0.25
\]
\[
P(X \geq 2) = P(X = 2) + P(X = 3) = 3 \times 0.37^2 \times 0.63^1 + 0.37^3 \approx 0.31
\]
\item Il faut d'abord tracer le tableau résumant la loi de probabilité:
\begin{center}
\begin{tabular}{|c|*{4}{c|}}
\hline
Valeur & 0 & 1 & 2 & 3 \\
\hline
Probabilité & $0.25$ & $0.441$ & $0.259$ &$0.051$ \\
\hline
\end{tabular}
\end{center}
On peut alors calculer l'espérance
\[
E[X] = 0 \times 0.25 + 1 \times 0.441 + 2 \times 0.259 + 3 \times 0.051 = 1.11
\]
On peut donc estimer qu'il y aura en moyenne $1.11$ personnes qui feront sonner le portique sur les 3 personnes.
\end{enumerate}
\end{solution}
\begin{exercise}[subtitle={Équation puissance}]
Résoudre les équations et inéquations suivantes
\begin{multicols}{2}
\begin{enumerate}
\item $10^x = 10$
\item $19^x = 35$
\item $0.59^x \leq 32$
\item $4 \times 0.92^x = 16$
\end{enumerate}
\end{multicols}
\end{exercise}
\begin{solution}
Les solutions ci-dessous ne sont pas justifiée car l'ordinateur ne sait pas faire. Par contre, vous vous devez savoir justifier vos réponses!
\begin{enumerate}
\item $x = \log(10)$
\item $x = \frac{\log(35)}{\log(19)}$
\item Il faut faire attention quand on divise par un log car ce dernier peut être négatif ce qui est le cas ici. Il faut donc pense à changer le sens de l'inégalité.
$x \geq \frac{\log(32)}{\log(0.59)}$
\item Il faut penser à faire la division à par $4$ avant d'utiliser le log car sinon, on ne peut pas utiliser la formule $\log(a^n) = n\times \log(a)$.
$x = \frac{\log(4.0)}{\log(0.92)}$
\end{enumerate}
\end{solution}
\begin{exercise}[subtitle={Étude de fonctions}]
Soit $f(x) = x^3 - 45x^2 + 648x - 33$ une fonction définie sur $\R$.
\begin{enumerate}
\item Calculer $f'(x)$ la dérivée de $f(x)$.
\item Calculer $f'(18)$ et $f'(12)$.
\item En déduire une forme factorisée de $f'(x)$.
\item Étudier le signe de $f'(x)$ et en déduire les variations de $f(x)$.
\item Est-ce que la fonction $f(x)$ admet un maximum ou un minimum? Si oui, calculer sa valeur.
\end{enumerate}
\end{exercise}
\begin{solution}
\begin{enumerate}
\item Dérivée de $f(x)$: $f'(x) = 3x^2 - 90x + 648$
\item
\begin{align*}
f'(18) &= 3 \times 18^{2} - 90 \times 18 + 648\\&= 3 \times 324 - 1620 + 648\\&= 972 - 972\\&= 0
\end{align*}
\begin{align*}
f'(12) &= 3 \times 12^{2} - 90 \times 12 + 648\\&= 3 \times 144 - 1080 + 648\\&= 432 - 432\\&= 0
\end{align*}
Donc $x = 18$ et $x=12$ sont des racines de $f'(x) = 3x^2 - 90x + 648$.
\item On en déduit la forme factorisée suivante
\[
f'(x) = 3 (x - 18)(x-12)
\]
\item Pas de correction disponible
\item À causes des branches extérieurs, la fonction $f(x)$ n'a pas de maximum ou de minimum.
\end{enumerate}
\end{solution}
%\printsolutionstype{exercise}
\end{document}
%%% Local Variables:
%%% mode: latex
%%% TeX-master: "master"
%%% End:

View File

@ -0,0 +1,203 @@
\documentclass[a5paper,10pt]{article}
\usepackage{myXsim}
\usepackage{tasks}
% Title Page
\title{DM2 \hfill DINGER Sölen}
\tribe{TST}
\date{\hfillÀ render pour le Mercredi 24 février}
\xsimsetup{
solution/print = true
}
\begin{document}
\maketitle
\begin{exercise}[subtitle={Loi binomiale}]
Trois personnes s'apprêtent à passer le portique de sécurité. On suppose que pour chaque personne la probabilité que le portique sonne est égale à $0.53$.
Soit $X$ la variable aléatoire donnant le nombre de personnes faisant sonner le portique, parmi les 3 personnes de ce groupe.
\begin{enumerate}
\item Tracer l'arbre représentant le situation.
\item Justifier que $X$ suit une loi binomiale dont on précisera les paramètres.
\item Quelle est la probabilité qu'une seule personne fasse sonner le portique?
\item Calculer puis interpréter les probabilités suivantes
\[
P(X = 0) \qquad \qquad P(X \geq 2)
\]
\item Calculer l'espérance de $X$ et interpréter le résultat.
\end{enumerate}
\end{exercise}
\begin{solution}
\begin{enumerate}
\item
\begin{tikzpicture}[sloped]
\node {.}
child {node {$0$}
child {node {$0$}
child {node {$0$}
edge from parent
node[above] {0.47}
}
child {node {$1$}
edge from parent
node[above] {0.53}
}
edge from parent
node[above] {0.47}
}
child[missing] {}
child {node {$1$}
child {node {$0$}
edge from parent
node[above] {0.47}
}
child {node {$1$}
edge from parent
node[above] {0.53}
}
edge from parent
node[above] {0.47}
}
edge from parent
node[above] {0.47}
}
child[missing] {}
child[missing] {}
child[missing] {}
child { node {$1$}
child {node {$0$}
child {node {$0$}
edge from parent
node[above] {0.47}
}
child {node {$1$}
edge from parent
node[above] {0.53}
}
edge from parent
node[above] {0.47}
}
child[missing] {}
child {node {$1$}
child {node {$0$}
edge from parent
node[above] {0.47}
}
child {node {$1$}
edge from parent
node[above] {0.53}
}
edge from parent
node[above] {0.47}
}
edge from parent
node[above] {0.53}
} ;
\end{tikzpicture}
\item Chaque personne a 2 possibilités (1: fait sonner ou 2: ne fait pas sonner) et l'on fait passer 3 personnes ce qui correspond à une répétition identique et aléatoire. On peut donc modéliser la situation par une loi binomiale.
\[
X \sim \mathcal{B}(3; 0.76)
\]
\item Probabilité qu'une seule personne fasse sonner le portique. On voit qu'il y a 3 branches qui correspondent à cette situation dont
\[
P(X = 1) = 3 \times 0.53^1 \times 0.47^2 \approx 0.351
\]
\item
\[
P(X = 0) = 0.47^3 \approx 0.104
\]
\[
P(X \geq 2) = P(X = 2) + P(X = 3) = 3 \times 0.53^2 \times 0.47^1 + 0.53^3 \approx 0.545
\]
\item Il faut d'abord tracer le tableau résumant la loi de probabilité:
\begin{center}
\begin{tabular}{|c|*{4}{c|}}
\hline
Valeur & 0 & 1 & 2 & 3 \\
\hline
Probabilité & $0.104$ & $0.351$ & $0.396$ &$0.149$ \\
\hline
\end{tabular}
\end{center}
On peut alors calculer l'espérance
\[
E[X] = 0 \times 0.104 + 1 \times 0.351 + 2 \times 0.396 + 3 \times 0.149 = 1.59
\]
On peut donc estimer qu'il y aura en moyenne $1.59$ personnes qui feront sonner le portique sur les 3 personnes.
\end{enumerate}
\end{solution}
\begin{exercise}[subtitle={Équation puissance}]
Résoudre les équations et inéquations suivantes
\begin{multicols}{2}
\begin{enumerate}
\item $10^x = 22$
\item $20^x = 6$
\item $0.11^x \leq 14$
\item $8 \times 0.45^x = 46$
\end{enumerate}
\end{multicols}
\end{exercise}
\begin{solution}
Les solutions ci-dessous ne sont pas justifiée car l'ordinateur ne sait pas faire. Par contre, vous vous devez savoir justifier vos réponses!
\begin{enumerate}
\item $x = \log(22)$
\item $x = \frac{\log(6)}{\log(20)}$
\item Il faut faire attention quand on divise par un log car ce dernier peut être négatif ce qui est le cas ici. Il faut donc pense à changer le sens de l'inégalité.
$x \geq \frac{\log(14)}{\log(0.11)}$
\item Il faut penser à faire la division à par $8$ avant d'utiliser le log car sinon, on ne peut pas utiliser la formule $\log(a^n) = n\times \log(a)$.
$x = \frac{\log(5.75)}{\log(0.45)}$
\end{enumerate}
\end{solution}
\begin{exercise}[subtitle={Étude de fonctions}]
Soit $f(x) = - 4x^3 - 48x^2 + 2880x + 18$ une fonction définie sur $\R$.
\begin{enumerate}
\item Calculer $f'(x)$ la dérivée de $f(x)$.
\item Calculer $f'(12)$ et $f'(-20)$.
\item En déduire une forme factorisée de $f'(x)$.
\item Étudier le signe de $f'(x)$ et en déduire les variations de $f(x)$.
\item Est-ce que la fonction $f(x)$ admet un maximum ou un minimum? Si oui, calculer sa valeur.
\end{enumerate}
\end{exercise}
\begin{solution}
\begin{enumerate}
\item Dérivée de $f(x)$: $f'(x) = - 12x^2 - 96x + 2880$
\item
\begin{align*}
f'(12) &= - 12 \times 12^{2} - 96 \times 12 + 2880\\&= - 12 \times 144 - 1152 + 2880\\&= - 1728 + 1728\\&= 0
\end{align*}
\begin{align*}
f'(-20) &= - 12 \times - 20^{2} - 96(- 20) + 2880\\&= - 12 \times 400 + 1920 + 2880\\&= - 4800 + 4800\\&= 0
\end{align*}
Donc $x = 12$ et $x=-20$ sont des racines de $f'(x) = - 12x^2 - 96x + 2880$.
\item On en déduit la forme factorisée suivante
\[
f'(x) = -12 (x - 12)(x--20)
\]
\item Pas de correction disponible
\item À causes des branches extérieurs, la fonction $f(x)$ n'a pas de maximum ou de minimum.
\end{enumerate}
\end{solution}
%\printsolutionstype{exercise}
\end{document}
%%% Local Variables:
%%% mode: latex
%%% TeX-master: "master"
%%% End:

View File

@ -0,0 +1,203 @@
\documentclass[a5paper,10pt]{article}
\usepackage{myXsim}
\usepackage{tasks}
% Title Page
\title{DM2 \hfill EYRAUD Cynthia}
\tribe{TST}
\date{\hfillÀ render pour le Mercredi 24 février}
\xsimsetup{
solution/print = true
}
\begin{document}
\maketitle
\begin{exercise}[subtitle={Loi binomiale}]
Trois personnes s'apprêtent à passer le portique de sécurité. On suppose que pour chaque personne la probabilité que le portique sonne est égale à $0.14$.
Soit $X$ la variable aléatoire donnant le nombre de personnes faisant sonner le portique, parmi les 3 personnes de ce groupe.
\begin{enumerate}
\item Tracer l'arbre représentant le situation.
\item Justifier que $X$ suit une loi binomiale dont on précisera les paramètres.
\item Quelle est la probabilité qu'une seule personne fasse sonner le portique?
\item Calculer puis interpréter les probabilités suivantes
\[
P(X = 0) \qquad \qquad P(X \geq 2)
\]
\item Calculer l'espérance de $X$ et interpréter le résultat.
\end{enumerate}
\end{exercise}
\begin{solution}
\begin{enumerate}
\item
\begin{tikzpicture}[sloped]
\node {.}
child {node {$0$}
child {node {$0$}
child {node {$0$}
edge from parent
node[above] {0.86}
}
child {node {$1$}
edge from parent
node[above] {0.14}
}
edge from parent
node[above] {0.86}
}
child[missing] {}
child {node {$1$}
child {node {$0$}
edge from parent
node[above] {0.86}
}
child {node {$1$}
edge from parent
node[above] {0.14}
}
edge from parent
node[above] {0.86}
}
edge from parent
node[above] {0.86}
}
child[missing] {}
child[missing] {}
child[missing] {}
child { node {$1$}
child {node {$0$}
child {node {$0$}
edge from parent
node[above] {0.86}
}
child {node {$1$}
edge from parent
node[above] {0.14}
}
edge from parent
node[above] {0.86}
}
child[missing] {}
child {node {$1$}
child {node {$0$}
edge from parent
node[above] {0.86}
}
child {node {$1$}
edge from parent
node[above] {0.14}
}
edge from parent
node[above] {0.86}
}
edge from parent
node[above] {0.14}
} ;
\end{tikzpicture}
\item Chaque personne a 2 possibilités (1: fait sonner ou 2: ne fait pas sonner) et l'on fait passer 3 personnes ce qui correspond à une répétition identique et aléatoire. On peut donc modéliser la situation par une loi binomiale.
\[
X \sim \mathcal{B}(3; 0.76)
\]
\item Probabilité qu'une seule personne fasse sonner le portique. On voit qu'il y a 3 branches qui correspondent à cette situation dont
\[
P(X = 1) = 3 \times 0.14^1 \times 0.86^2 \approx 0.311
\]
\item
\[
P(X = 0) = 0.86^3 \approx 0.636
\]
\[
P(X \geq 2) = P(X = 2) + P(X = 3) = 3 \times 0.14^2 \times 0.86^1 + 0.14^3 \approx 0.054
\]
\item Il faut d'abord tracer le tableau résumant la loi de probabilité:
\begin{center}
\begin{tabular}{|c|*{4}{c|}}
\hline
Valeur & 0 & 1 & 2 & 3 \\
\hline
Probabilité & $0.636$ & $0.311$ & $0.051$ &$0.003$ \\
\hline
\end{tabular}
\end{center}
On peut alors calculer l'espérance
\[
E[X] = 0 \times 0.636 + 1 \times 0.311 + 2 \times 0.051 + 3 \times 0.003 = 0.42
\]
On peut donc estimer qu'il y aura en moyenne $0.42$ personnes qui feront sonner le portique sur les 3 personnes.
\end{enumerate}
\end{solution}
\begin{exercise}[subtitle={Équation puissance}]
Résoudre les équations et inéquations suivantes
\begin{multicols}{2}
\begin{enumerate}
\item $10^x = 5$
\item $20^x = 40$
\item $0.85^x \leq 38$
\item $2 \times 0.24^x = 24$
\end{enumerate}
\end{multicols}
\end{exercise}
\begin{solution}
Les solutions ci-dessous ne sont pas justifiée car l'ordinateur ne sait pas faire. Par contre, vous vous devez savoir justifier vos réponses!
\begin{enumerate}
\item $x = \log(5)$
\item $x = \frac{\log(40)}{\log(20)}$
\item Il faut faire attention quand on divise par un log car ce dernier peut être négatif ce qui est le cas ici. Il faut donc pense à changer le sens de l'inégalité.
$x \geq \frac{\log(38)}{\log(0.85)}$
\item Il faut penser à faire la division à par $2$ avant d'utiliser le log car sinon, on ne peut pas utiliser la formule $\log(a^n) = n\times \log(a)$.
$x = \frac{\log(12.0)}{\log(0.24)}$
\end{enumerate}
\end{solution}
\begin{exercise}[subtitle={Étude de fonctions}]
Soit $f(x) = - 3x^3 + 49.5x^2 + 5580x - 3$ une fonction définie sur $\R$.
\begin{enumerate}
\item Calculer $f'(x)$ la dérivée de $f(x)$.
\item Calculer $f'(31)$ et $f'(-20)$.
\item En déduire une forme factorisée de $f'(x)$.
\item Étudier le signe de $f'(x)$ et en déduire les variations de $f(x)$.
\item Est-ce que la fonction $f(x)$ admet un maximum ou un minimum? Si oui, calculer sa valeur.
\end{enumerate}
\end{exercise}
\begin{solution}
\begin{enumerate}
\item Dérivée de $f(x)$: $f'(x) = - 9x^2 + 99x + 5580$
\item
\begin{align*}
f'(31) &= - 9 \times 31^{2} + 99 \times 31 + 5580\\&= - 9 \times 961 + 3069 + 5580\\&= - 8649 + 8649\\&= 0
\end{align*}
\begin{align*}
f'(-20) &= - 9 \times - 20^{2} + 99(- 20) + 5580\\&= - 9 \times 400 - 1980 + 5580\\&= - 3600 + 3600\\&= 0
\end{align*}
Donc $x = 31$ et $x=-20$ sont des racines de $f'(x) = - 9x^2 + 99x + 5580$.
\item On en déduit la forme factorisée suivante
\[
f'(x) = -9 (x - 31)(x--20)
\]
\item Pas de correction disponible
\item À causes des branches extérieurs, la fonction $f(x)$ n'a pas de maximum ou de minimum.
\end{enumerate}
\end{solution}
%\printsolutionstype{exercise}
\end{document}
%%% Local Variables:
%%% mode: latex
%%% TeX-master: "master"
%%% End:

View File

@ -0,0 +1,203 @@
\documentclass[a5paper,10pt]{article}
\usepackage{myXsim}
\usepackage{tasks}
% Title Page
\title{DM2 \hfill FERREIRA Léo}
\tribe{TST}
\date{\hfillÀ render pour le Mercredi 24 février}
\xsimsetup{
solution/print = true
}
\begin{document}
\maketitle
\begin{exercise}[subtitle={Loi binomiale}]
Trois personnes s'apprêtent à passer le portique de sécurité. On suppose que pour chaque personne la probabilité que le portique sonne est égale à $0.05$.
Soit $X$ la variable aléatoire donnant le nombre de personnes faisant sonner le portique, parmi les 3 personnes de ce groupe.
\begin{enumerate}
\item Tracer l'arbre représentant le situation.
\item Justifier que $X$ suit une loi binomiale dont on précisera les paramètres.
\item Quelle est la probabilité qu'une seule personne fasse sonner le portique?
\item Calculer puis interpréter les probabilités suivantes
\[
P(X = 0) \qquad \qquad P(X \geq 2)
\]
\item Calculer l'espérance de $X$ et interpréter le résultat.
\end{enumerate}
\end{exercise}
\begin{solution}
\begin{enumerate}
\item
\begin{tikzpicture}[sloped]
\node {.}
child {node {$0$}
child {node {$0$}
child {node {$0$}
edge from parent
node[above] {0.95}
}
child {node {$1$}
edge from parent
node[above] {0.05}
}
edge from parent
node[above] {0.95}
}
child[missing] {}
child {node {$1$}
child {node {$0$}
edge from parent
node[above] {0.95}
}
child {node {$1$}
edge from parent
node[above] {0.05}
}
edge from parent
node[above] {0.95}
}
edge from parent
node[above] {0.95}
}
child[missing] {}
child[missing] {}
child[missing] {}
child { node {$1$}
child {node {$0$}
child {node {$0$}
edge from parent
node[above] {0.95}
}
child {node {$1$}
edge from parent
node[above] {0.05}
}
edge from parent
node[above] {0.95}
}
child[missing] {}
child {node {$1$}
child {node {$0$}
edge from parent
node[above] {0.95}
}
child {node {$1$}
edge from parent
node[above] {0.05}
}
edge from parent
node[above] {0.95}
}
edge from parent
node[above] {0.05}
} ;
\end{tikzpicture}
\item Chaque personne a 2 possibilités (1: fait sonner ou 2: ne fait pas sonner) et l'on fait passer 3 personnes ce qui correspond à une répétition identique et aléatoire. On peut donc modéliser la situation par une loi binomiale.
\[
X \sim \mathcal{B}(3; 0.76)
\]
\item Probabilité qu'une seule personne fasse sonner le portique. On voit qu'il y a 3 branches qui correspondent à cette situation dont
\[
P(X = 1) = 3 \times 0.05^1 \times 0.95^2 \approx 0.135
\]
\item
\[
P(X = 0) = 0.95^3 \approx 0.857
\]
\[
P(X \geq 2) = P(X = 2) + P(X = 3) = 3 \times 0.05^2 \times 0.95^1 + 0.05^3 \approx 0.007
\]
\item Il faut d'abord tracer le tableau résumant la loi de probabilité:
\begin{center}
\begin{tabular}{|c|*{4}{c|}}
\hline
Valeur & 0 & 1 & 2 & 3 \\
\hline
Probabilité & $0.857$ & $0.135$ & $0.007$ &$0.0$ \\
\hline
\end{tabular}
\end{center}
On peut alors calculer l'espérance
\[
E[X] = 0 \times 0.857 + 1 \times 0.135 + 2 \times 0.007 + 3 \times 0.0 = 0.15
\]
On peut donc estimer qu'il y aura en moyenne $0.15$ personnes qui feront sonner le portique sur les 3 personnes.
\end{enumerate}
\end{solution}
\begin{exercise}[subtitle={Équation puissance}]
Résoudre les équations et inéquations suivantes
\begin{multicols}{2}
\begin{enumerate}
\item $10^x = 39$
\item $19^x = 43$
\item $0.24^x \leq 2$
\item $5 \times 0.52^x = 16$
\end{enumerate}
\end{multicols}
\end{exercise}
\begin{solution}
Les solutions ci-dessous ne sont pas justifiée car l'ordinateur ne sait pas faire. Par contre, vous vous devez savoir justifier vos réponses!
\begin{enumerate}
\item $x = \log(39)$
\item $x = \frac{\log(43)}{\log(19)}$
\item Il faut faire attention quand on divise par un log car ce dernier peut être négatif ce qui est le cas ici. Il faut donc pense à changer le sens de l'inégalité.
$x \geq \frac{\log(2)}{\log(0.24)}$
\item Il faut penser à faire la division à par $5$ avant d'utiliser le log car sinon, on ne peut pas utiliser la formule $\log(a^n) = n\times \log(a)$.
$x = \frac{\log(3.2)}{\log(0.52)}$
\end{enumerate}
\end{solution}
\begin{exercise}[subtitle={Étude de fonctions}]
Soit $f(x) = 2x^3 - 171x^2 + 2592x + 12$ une fonction définie sur $\R$.
\begin{enumerate}
\item Calculer $f'(x)$ la dérivée de $f(x)$.
\item Calculer $f'(48)$ et $f'(9)$.
\item En déduire une forme factorisée de $f'(x)$.
\item Étudier le signe de $f'(x)$ et en déduire les variations de $f(x)$.
\item Est-ce que la fonction $f(x)$ admet un maximum ou un minimum? Si oui, calculer sa valeur.
\end{enumerate}
\end{exercise}
\begin{solution}
\begin{enumerate}
\item Dérivée de $f(x)$: $f'(x) = 6x^2 - 342x + 2592$
\item
\begin{align*}
f'(48) &= 6 \times 48^{2} - 342 \times 48 + 2592\\&= 6 \times 2304 - 16416 + 2592\\&= 13824 - 13824\\&= 0
\end{align*}
\begin{align*}
f'(9) &= 6 \times 9^{2} - 342 \times 9 + 2592\\&= 6 \times 81 - 3078 + 2592\\&= 486 - 486\\&= 0
\end{align*}
Donc $x = 48$ et $x=9$ sont des racines de $f'(x) = 6x^2 - 342x + 2592$.
\item On en déduit la forme factorisée suivante
\[
f'(x) = 6 (x - 48)(x-9)
\]
\item Pas de correction disponible
\item À causes des branches extérieurs, la fonction $f(x)$ n'a pas de maximum ou de minimum.
\end{enumerate}
\end{solution}
%\printsolutionstype{exercise}
\end{document}
%%% Local Variables:
%%% mode: latex
%%% TeX-master: "master"
%%% End:

View File

@ -0,0 +1,203 @@
\documentclass[a5paper,10pt]{article}
\usepackage{myXsim}
\usepackage{tasks}
% Title Page
\title{DM2 \hfill FILALI Zakaria}
\tribe{TST}
\date{\hfillÀ render pour le Mercredi 24 février}
\xsimsetup{
solution/print = true
}
\begin{document}
\maketitle
\begin{exercise}[subtitle={Loi binomiale}]
Trois personnes s'apprêtent à passer le portique de sécurité. On suppose que pour chaque personne la probabilité que le portique sonne est égale à $0.01$.
Soit $X$ la variable aléatoire donnant le nombre de personnes faisant sonner le portique, parmi les 3 personnes de ce groupe.
\begin{enumerate}
\item Tracer l'arbre représentant le situation.
\item Justifier que $X$ suit une loi binomiale dont on précisera les paramètres.
\item Quelle est la probabilité qu'une seule personne fasse sonner le portique?
\item Calculer puis interpréter les probabilités suivantes
\[
P(X = 0) \qquad \qquad P(X \geq 2)
\]
\item Calculer l'espérance de $X$ et interpréter le résultat.
\end{enumerate}
\end{exercise}
\begin{solution}
\begin{enumerate}
\item
\begin{tikzpicture}[sloped]
\node {.}
child {node {$0$}
child {node {$0$}
child {node {$0$}
edge from parent
node[above] {0.99}
}
child {node {$1$}
edge from parent
node[above] {0.01}
}
edge from parent
node[above] {0.99}
}
child[missing] {}
child {node {$1$}
child {node {$0$}
edge from parent
node[above] {0.99}
}
child {node {$1$}
edge from parent
node[above] {0.01}
}
edge from parent
node[above] {0.99}
}
edge from parent
node[above] {0.99}
}
child[missing] {}
child[missing] {}
child[missing] {}
child { node {$1$}
child {node {$0$}
child {node {$0$}
edge from parent
node[above] {0.99}
}
child {node {$1$}
edge from parent
node[above] {0.01}
}
edge from parent
node[above] {0.99}
}
child[missing] {}
child {node {$1$}
child {node {$0$}
edge from parent
node[above] {0.99}
}
child {node {$1$}
edge from parent
node[above] {0.01}
}
edge from parent
node[above] {0.99}
}
edge from parent
node[above] {0.01}
} ;
\end{tikzpicture}
\item Chaque personne a 2 possibilités (1: fait sonner ou 2: ne fait pas sonner) et l'on fait passer 3 personnes ce qui correspond à une répétition identique et aléatoire. On peut donc modéliser la situation par une loi binomiale.
\[
X \sim \mathcal{B}(3; 0.76)
\]
\item Probabilité qu'une seule personne fasse sonner le portique. On voit qu'il y a 3 branches qui correspondent à cette situation dont
\[
P(X = 1) = 3 \times 0.01^1 \times 0.99^2 \approx 0.029
\]
\item
\[
P(X = 0) = 0.99^3 \approx 0.97
\]
\[
P(X \geq 2) = P(X = 2) + P(X = 3) = 3 \times 0.01^2 \times 0.99^1 + 0.01^3 \approx 0.0
\]
\item Il faut d'abord tracer le tableau résumant la loi de probabilité:
\begin{center}
\begin{tabular}{|c|*{4}{c|}}
\hline
Valeur & 0 & 1 & 2 & 3 \\
\hline
Probabilité & $0.97$ & $0.029$ & $0.0$ &$0.0$ \\
\hline
\end{tabular}
\end{center}
On peut alors calculer l'espérance
\[
E[X] = 0 \times 0.97 + 1 \times 0.029 + 2 \times 0.0 + 3 \times 0.0 = 0.03
\]
On peut donc estimer qu'il y aura en moyenne $0.03$ personnes qui feront sonner le portique sur les 3 personnes.
\end{enumerate}
\end{solution}
\begin{exercise}[subtitle={Équation puissance}]
Résoudre les équations et inéquations suivantes
\begin{multicols}{2}
\begin{enumerate}
\item $10^x = 39$
\item $19^x = 15$
\item $0.73^x \leq 39$
\item $8 \times 0.15^x = 20$
\end{enumerate}
\end{multicols}
\end{exercise}
\begin{solution}
Les solutions ci-dessous ne sont pas justifiée car l'ordinateur ne sait pas faire. Par contre, vous vous devez savoir justifier vos réponses!
\begin{enumerate}
\item $x = \log(39)$
\item $x = \frac{\log(15)}{\log(19)}$
\item Il faut faire attention quand on divise par un log car ce dernier peut être négatif ce qui est le cas ici. Il faut donc pense à changer le sens de l'inégalité.
$x \geq \frac{\log(39)}{\log(0.73)}$
\item Il faut penser à faire la division à par $8$ avant d'utiliser le log car sinon, on ne peut pas utiliser la formule $\log(a^n) = n\times \log(a)$.
$x = \frac{\log(2.5)}{\log(0.15)}$
\end{enumerate}
\end{solution}
\begin{exercise}[subtitle={Étude de fonctions}]
Soit $f(x) = 8x^3 - 264x^2 - 20160x - 32$ une fonction définie sur $\R$.
\begin{enumerate}
\item Calculer $f'(x)$ la dérivée de $f(x)$.
\item Calculer $f'(42)$ et $f'(-20)$.
\item En déduire une forme factorisée de $f'(x)$.
\item Étudier le signe de $f'(x)$ et en déduire les variations de $f(x)$.
\item Est-ce que la fonction $f(x)$ admet un maximum ou un minimum? Si oui, calculer sa valeur.
\end{enumerate}
\end{exercise}
\begin{solution}
\begin{enumerate}
\item Dérivée de $f(x)$: $f'(x) = 24x^2 - 528x - 20160$
\item
\begin{align*}
f'(42) &= 24 \times 42^{2} - 528 \times 42 - 20160\\&= 24 \times 1764 - 22176 - 20160\\&= 42336 - 42336\\&= 0
\end{align*}
\begin{align*}
f'(-20) &= 24 \times - 20^{2} - 528(- 20) - 20160\\&= 24 \times 400 + 10560 - 20160\\&= 9600 - 9600\\&= 0
\end{align*}
Donc $x = 42$ et $x=-20$ sont des racines de $f'(x) = 24x^2 - 528x - 20160$.
\item On en déduit la forme factorisée suivante
\[
f'(x) = 24 (x - 42)(x--20)
\]
\item Pas de correction disponible
\item À causes des branches extérieurs, la fonction $f(x)$ n'a pas de maximum ou de minimum.
\end{enumerate}
\end{solution}
%\printsolutionstype{exercise}
\end{document}
%%% Local Variables:
%%% mode: latex
%%% TeX-master: "master"
%%% End:

View File

@ -0,0 +1,203 @@
\documentclass[a5paper,10pt]{article}
\usepackage{myXsim}
\usepackage{tasks}
% Title Page
\title{DM2 \hfill FOIGNY Romain}
\tribe{TST}
\date{\hfillÀ render pour le Mercredi 24 février}
\xsimsetup{
solution/print = true
}
\begin{document}
\maketitle
\begin{exercise}[subtitle={Loi binomiale}]
Trois personnes s'apprêtent à passer le portique de sécurité. On suppose que pour chaque personne la probabilité que le portique sonne est égale à $0.25$.
Soit $X$ la variable aléatoire donnant le nombre de personnes faisant sonner le portique, parmi les 3 personnes de ce groupe.
\begin{enumerate}
\item Tracer l'arbre représentant le situation.
\item Justifier que $X$ suit une loi binomiale dont on précisera les paramètres.
\item Quelle est la probabilité qu'une seule personne fasse sonner le portique?
\item Calculer puis interpréter les probabilités suivantes
\[
P(X = 0) \qquad \qquad P(X \geq 2)
\]
\item Calculer l'espérance de $X$ et interpréter le résultat.
\end{enumerate}
\end{exercise}
\begin{solution}
\begin{enumerate}
\item
\begin{tikzpicture}[sloped]
\node {.}
child {node {$0$}
child {node {$0$}
child {node {$0$}
edge from parent
node[above] {0.75}
}
child {node {$1$}
edge from parent
node[above] {0.25}
}
edge from parent
node[above] {0.75}
}
child[missing] {}
child {node {$1$}
child {node {$0$}
edge from parent
node[above] {0.75}
}
child {node {$1$}
edge from parent
node[above] {0.25}
}
edge from parent
node[above] {0.75}
}
edge from parent
node[above] {0.75}
}
child[missing] {}
child[missing] {}
child[missing] {}
child { node {$1$}
child {node {$0$}
child {node {$0$}
edge from parent
node[above] {0.75}
}
child {node {$1$}
edge from parent
node[above] {0.25}
}
edge from parent
node[above] {0.75}
}
child[missing] {}
child {node {$1$}
child {node {$0$}
edge from parent
node[above] {0.75}
}
child {node {$1$}
edge from parent
node[above] {0.25}
}
edge from parent
node[above] {0.75}
}
edge from parent
node[above] {0.25}
} ;
\end{tikzpicture}
\item Chaque personne a 2 possibilités (1: fait sonner ou 2: ne fait pas sonner) et l'on fait passer 3 personnes ce qui correspond à une répétition identique et aléatoire. On peut donc modéliser la situation par une loi binomiale.
\[
X \sim \mathcal{B}(3; 0.76)
\]
\item Probabilité qu'une seule personne fasse sonner le portique. On voit qu'il y a 3 branches qui correspondent à cette situation dont
\[
P(X = 1) = 3 \times 0.25^1 \times 0.75^2 \approx 0.422
\]
\item
\[
P(X = 0) = 0.75^3 \approx 0.422
\]
\[
P(X \geq 2) = P(X = 2) + P(X = 3) = 3 \times 0.25^2 \times 0.75^1 + 0.25^3 \approx 0.157
\]
\item Il faut d'abord tracer le tableau résumant la loi de probabilité:
\begin{center}
\begin{tabular}{|c|*{4}{c|}}
\hline
Valeur & 0 & 1 & 2 & 3 \\
\hline
Probabilité & $0.422$ & $0.422$ & $0.141$ &$0.016$ \\
\hline
\end{tabular}
\end{center}
On peut alors calculer l'espérance
\[
E[X] = 0 \times 0.422 + 1 \times 0.422 + 2 \times 0.141 + 3 \times 0.016 = 0.75
\]
On peut donc estimer qu'il y aura en moyenne $0.75$ personnes qui feront sonner le portique sur les 3 personnes.
\end{enumerate}
\end{solution}
\begin{exercise}[subtitle={Équation puissance}]
Résoudre les équations et inéquations suivantes
\begin{multicols}{2}
\begin{enumerate}
\item $10^x = 13$
\item $17^x = 26$
\item $0.75^x \leq 40$
\item $9 \times 0.86^x = 44$
\end{enumerate}
\end{multicols}
\end{exercise}
\begin{solution}
Les solutions ci-dessous ne sont pas justifiée car l'ordinateur ne sait pas faire. Par contre, vous vous devez savoir justifier vos réponses!
\begin{enumerate}
\item $x = \log(13)$
\item $x = \frac{\log(26)}{\log(17)}$
\item Il faut faire attention quand on divise par un log car ce dernier peut être négatif ce qui est le cas ici. Il faut donc pense à changer le sens de l'inégalité.
$x \geq \frac{\log(40)}{\log(0.75)}$
\item Il faut penser à faire la division à par $9$ avant d'utiliser le log car sinon, on ne peut pas utiliser la formule $\log(a^n) = n\times \log(a)$.
$x = \frac{\log(4.89)}{\log(0.86)}$
\end{enumerate}
\end{solution}
\begin{exercise}[subtitle={Étude de fonctions}]
Soit $f(x) = 2x^3 - 84x^2 + 960x + 28$ une fonction définie sur $\R$.
\begin{enumerate}
\item Calculer $f'(x)$ la dérivée de $f(x)$.
\item Calculer $f'(8)$ et $f'(20)$.
\item En déduire une forme factorisée de $f'(x)$.
\item Étudier le signe de $f'(x)$ et en déduire les variations de $f(x)$.
\item Est-ce que la fonction $f(x)$ admet un maximum ou un minimum? Si oui, calculer sa valeur.
\end{enumerate}
\end{exercise}
\begin{solution}
\begin{enumerate}
\item Dérivée de $f(x)$: $f'(x) = 6x^2 - 168x + 960$
\item
\begin{align*}
f'(8) &= 6 \times 8^{2} - 168 \times 8 + 960\\&= 6 \times 64 - 1344 + 960\\&= 384 - 384\\&= 0
\end{align*}
\begin{align*}
f'(20) &= 6 \times 20^{2} - 168 \times 20 + 960\\&= 6 \times 400 - 3360 + 960\\&= 2400 - 2400\\&= 0
\end{align*}
Donc $x = 8$ et $x=20$ sont des racines de $f'(x) = 6x^2 - 168x + 960$.
\item On en déduit la forme factorisée suivante
\[
f'(x) = 6 (x - 8)(x-20)
\]
\item Pas de correction disponible
\item À causes des branches extérieurs, la fonction $f(x)$ n'a pas de maximum ou de minimum.
\end{enumerate}
\end{solution}
%\printsolutionstype{exercise}
\end{document}
%%% Local Variables:
%%% mode: latex
%%% TeX-master: "master"
%%% End:

View File

@ -0,0 +1,203 @@
\documentclass[a5paper,10pt]{article}
\usepackage{myXsim}
\usepackage{tasks}
% Title Page
\title{DM2 \hfill HIPOLITO DA SILVA Andréa}
\tribe{TST}
\date{\hfillÀ render pour le Mercredi 24 février}
\xsimsetup{
solution/print = true
}
\begin{document}
\maketitle
\begin{exercise}[subtitle={Loi binomiale}]
Trois personnes s'apprêtent à passer le portique de sécurité. On suppose que pour chaque personne la probabilité que le portique sonne est égale à $0.1$.
Soit $X$ la variable aléatoire donnant le nombre de personnes faisant sonner le portique, parmi les 3 personnes de ce groupe.
\begin{enumerate}
\item Tracer l'arbre représentant le situation.
\item Justifier que $X$ suit une loi binomiale dont on précisera les paramètres.
\item Quelle est la probabilité qu'une seule personne fasse sonner le portique?
\item Calculer puis interpréter les probabilités suivantes
\[
P(X = 0) \qquad \qquad P(X \geq 2)
\]
\item Calculer l'espérance de $X$ et interpréter le résultat.
\end{enumerate}
\end{exercise}
\begin{solution}
\begin{enumerate}
\item
\begin{tikzpicture}[sloped]
\node {.}
child {node {$0$}
child {node {$0$}
child {node {$0$}
edge from parent
node[above] {0.9}
}
child {node {$1$}
edge from parent
node[above] {0.1}
}
edge from parent
node[above] {0.9}
}
child[missing] {}
child {node {$1$}
child {node {$0$}
edge from parent
node[above] {0.9}
}
child {node {$1$}
edge from parent
node[above] {0.1}
}
edge from parent
node[above] {0.9}
}
edge from parent
node[above] {0.9}
}
child[missing] {}
child[missing] {}
child[missing] {}
child { node {$1$}
child {node {$0$}
child {node {$0$}
edge from parent
node[above] {0.9}
}
child {node {$1$}
edge from parent
node[above] {0.1}
}
edge from parent
node[above] {0.9}
}
child[missing] {}
child {node {$1$}
child {node {$0$}
edge from parent
node[above] {0.9}
}
child {node {$1$}
edge from parent
node[above] {0.1}
}
edge from parent
node[above] {0.9}
}
edge from parent
node[above] {0.1}
} ;
\end{tikzpicture}
\item Chaque personne a 2 possibilités (1: fait sonner ou 2: ne fait pas sonner) et l'on fait passer 3 personnes ce qui correspond à une répétition identique et aléatoire. On peut donc modéliser la situation par une loi binomiale.
\[
X \sim \mathcal{B}(3; 0.76)
\]
\item Probabilité qu'une seule personne fasse sonner le portique. On voit qu'il y a 3 branches qui correspondent à cette situation dont
\[
P(X = 1) = 3 \times 0.1^1 \times 0.9^2 \approx 0.243
\]
\item
\[
P(X = 0) = 0.9^3 \approx 0.729
\]
\[
P(X \geq 2) = P(X = 2) + P(X = 3) = 3 \times 0.1^2 \times 0.9^1 + 0.1^3 \approx 0.028
\]
\item Il faut d'abord tracer le tableau résumant la loi de probabilité:
\begin{center}
\begin{tabular}{|c|*{4}{c|}}
\hline
Valeur & 0 & 1 & 2 & 3 \\
\hline
Probabilité & $0.729$ & $0.243$ & $0.027$ &$0.001$ \\
\hline
\end{tabular}
\end{center}
On peut alors calculer l'espérance
\[
E[X] = 0 \times 0.729 + 1 \times 0.243 + 2 \times 0.027 + 3 \times 0.001 = 0.3
\]
On peut donc estimer qu'il y aura en moyenne $0.3$ personnes qui feront sonner le portique sur les 3 personnes.
\end{enumerate}
\end{solution}
\begin{exercise}[subtitle={Équation puissance}]
Résoudre les équations et inéquations suivantes
\begin{multicols}{2}
\begin{enumerate}
\item $10^x = 36$
\item $12^x = 44$
\item $0.08^x \leq 43$
\item $4 \times 0.93^x = 46$
\end{enumerate}
\end{multicols}
\end{exercise}
\begin{solution}
Les solutions ci-dessous ne sont pas justifiée car l'ordinateur ne sait pas faire. Par contre, vous vous devez savoir justifier vos réponses!
\begin{enumerate}
\item $x = \log(36)$
\item $x = \frac{\log(44)}{\log(12)}$
\item Il faut faire attention quand on divise par un log car ce dernier peut être négatif ce qui est le cas ici. Il faut donc pense à changer le sens de l'inégalité.
$x \geq \frac{\log(43)}{\log(0.08)}$
\item Il faut penser à faire la division à par $4$ avant d'utiliser le log car sinon, on ne peut pas utiliser la formule $\log(a^n) = n\times \log(a)$.
$x = \frac{\log(11.5)}{\log(0.93)}$
\end{enumerate}
\end{solution}
\begin{exercise}[subtitle={Étude de fonctions}]
Soit $f(x) = - 7x^3 + 294x^2 - 4032x - 33$ une fonction définie sur $\R$.
\begin{enumerate}
\item Calculer $f'(x)$ la dérivée de $f(x)$.
\item Calculer $f'(12)$ et $f'(16)$.
\item En déduire une forme factorisée de $f'(x)$.
\item Étudier le signe de $f'(x)$ et en déduire les variations de $f(x)$.
\item Est-ce que la fonction $f(x)$ admet un maximum ou un minimum? Si oui, calculer sa valeur.
\end{enumerate}
\end{exercise}
\begin{solution}
\begin{enumerate}
\item Dérivée de $f(x)$: $f'(x) = - 21x^2 + 588x - 4032$
\item
\begin{align*}
f'(12) &= - 21 \times 12^{2} + 588 \times 12 - 4032\\&= - 21 \times 144 + 7056 - 4032\\&= - 3024 + 3024\\&= 0
\end{align*}
\begin{align*}
f'(16) &= - 21 \times 16^{2} + 588 \times 16 - 4032\\&= - 21 \times 256 + 9408 - 4032\\&= - 5376 + 5376\\&= 0
\end{align*}
Donc $x = 12$ et $x=16$ sont des racines de $f'(x) = - 21x^2 + 588x - 4032$.
\item On en déduit la forme factorisée suivante
\[
f'(x) = -21 (x - 12)(x-16)
\]
\item Pas de correction disponible
\item À causes des branches extérieurs, la fonction $f(x)$ n'a pas de maximum ou de minimum.
\end{enumerate}
\end{solution}
%\printsolutionstype{exercise}
\end{document}
%%% Local Variables:
%%% mode: latex
%%% TeX-master: "master"
%%% End:

View File

@ -0,0 +1,203 @@
\documentclass[a5paper,10pt]{article}
\usepackage{myXsim}
\usepackage{tasks}
% Title Page
\title{DM2 \hfill HUMBERT Rayan}
\tribe{TST}
\date{\hfillÀ render pour le Mercredi 24 février}
\xsimsetup{
solution/print = true
}
\begin{document}
\maketitle
\begin{exercise}[subtitle={Loi binomiale}]
Trois personnes s'apprêtent à passer le portique de sécurité. On suppose que pour chaque personne la probabilité que le portique sonne est égale à $0.68$.
Soit $X$ la variable aléatoire donnant le nombre de personnes faisant sonner le portique, parmi les 3 personnes de ce groupe.
\begin{enumerate}
\item Tracer l'arbre représentant le situation.
\item Justifier que $X$ suit une loi binomiale dont on précisera les paramètres.
\item Quelle est la probabilité qu'une seule personne fasse sonner le portique?
\item Calculer puis interpréter les probabilités suivantes
\[
P(X = 0) \qquad \qquad P(X \geq 2)
\]
\item Calculer l'espérance de $X$ et interpréter le résultat.
\end{enumerate}
\end{exercise}
\begin{solution}
\begin{enumerate}
\item
\begin{tikzpicture}[sloped]
\node {.}
child {node {$0$}
child {node {$0$}
child {node {$0$}
edge from parent
node[above] {0.32}
}
child {node {$1$}
edge from parent
node[above] {0.68}
}
edge from parent
node[above] {0.32}
}
child[missing] {}
child {node {$1$}
child {node {$0$}
edge from parent
node[above] {0.32}
}
child {node {$1$}
edge from parent
node[above] {0.68}
}
edge from parent
node[above] {0.32}
}
edge from parent
node[above] {0.32}
}
child[missing] {}
child[missing] {}
child[missing] {}
child { node {$1$}
child {node {$0$}
child {node {$0$}
edge from parent
node[above] {0.32}
}
child {node {$1$}
edge from parent
node[above] {0.68}
}
edge from parent
node[above] {0.32}
}
child[missing] {}
child {node {$1$}
child {node {$0$}
edge from parent
node[above] {0.32}
}
child {node {$1$}
edge from parent
node[above] {0.68}
}
edge from parent
node[above] {0.32}
}
edge from parent
node[above] {0.68}
} ;
\end{tikzpicture}
\item Chaque personne a 2 possibilités (1: fait sonner ou 2: ne fait pas sonner) et l'on fait passer 3 personnes ce qui correspond à une répétition identique et aléatoire. On peut donc modéliser la situation par une loi binomiale.
\[
X \sim \mathcal{B}(3; 0.76)
\]
\item Probabilité qu'une seule personne fasse sonner le portique. On voit qu'il y a 3 branches qui correspondent à cette situation dont
\[
P(X = 1) = 3 \times 0.68^1 \times 0.32^2 \approx 0.209
\]
\item
\[
P(X = 0) = 0.32^3 \approx 0.033
\]
\[
P(X \geq 2) = P(X = 2) + P(X = 3) = 3 \times 0.68^2 \times 0.32^1 + 0.68^3 \approx 0.758
\]
\item Il faut d'abord tracer le tableau résumant la loi de probabilité:
\begin{center}
\begin{tabular}{|c|*{4}{c|}}
\hline
Valeur & 0 & 1 & 2 & 3 \\
\hline
Probabilité & $0.033$ & $0.209$ & $0.444$ &$0.314$ \\
\hline
\end{tabular}
\end{center}
On peut alors calculer l'espérance
\[
E[X] = 0 \times 0.033 + 1 \times 0.209 + 2 \times 0.444 + 3 \times 0.314 = 2.04
\]
On peut donc estimer qu'il y aura en moyenne $2.04$ personnes qui feront sonner le portique sur les 3 personnes.
\end{enumerate}
\end{solution}
\begin{exercise}[subtitle={Équation puissance}]
Résoudre les équations et inéquations suivantes
\begin{multicols}{2}
\begin{enumerate}
\item $10^x = 8$
\item $7^x = 21$
\item $0.19^x \leq 12$
\item $9 \times 0.76^x = 6$
\end{enumerate}
\end{multicols}
\end{exercise}
\begin{solution}
Les solutions ci-dessous ne sont pas justifiée car l'ordinateur ne sait pas faire. Par contre, vous vous devez savoir justifier vos réponses!
\begin{enumerate}
\item $x = \log(8)$
\item $x = \frac{\log(21)}{\log(7)}$
\item Il faut faire attention quand on divise par un log car ce dernier peut être négatif ce qui est le cas ici. Il faut donc pense à changer le sens de l'inégalité.
$x \geq \frac{\log(12)}{\log(0.19)}$
\item Il faut penser à faire la division à par $9$ avant d'utiliser le log car sinon, on ne peut pas utiliser la formule $\log(a^n) = n\times \log(a)$.
$x = \frac{\log(0.67)}{\log(0.76)}$
\end{enumerate}
\end{solution}
\begin{exercise}[subtitle={Étude de fonctions}]
Soit $f(x) = 7x^3 - 483x^2 + 10584x - 2$ une fonction définie sur $\R$.
\begin{enumerate}
\item Calculer $f'(x)$ la dérivée de $f(x)$.
\item Calculer $f'(28)$ et $f'(18)$.
\item En déduire une forme factorisée de $f'(x)$.
\item Étudier le signe de $f'(x)$ et en déduire les variations de $f(x)$.
\item Est-ce que la fonction $f(x)$ admet un maximum ou un minimum? Si oui, calculer sa valeur.
\end{enumerate}
\end{exercise}
\begin{solution}
\begin{enumerate}
\item Dérivée de $f(x)$: $f'(x) = 21x^2 - 966x + 10584$
\item
\begin{align*}
f'(28) &= 21 \times 28^{2} - 966 \times 28 + 10584\\&= 21 \times 784 - 27048 + 10584\\&= 16464 - 16464\\&= 0
\end{align*}
\begin{align*}
f'(18) &= 21 \times 18^{2} - 966 \times 18 + 10584\\&= 21 \times 324 - 17388 + 10584\\&= 6804 - 6804\\&= 0
\end{align*}
Donc $x = 28$ et $x=18$ sont des racines de $f'(x) = 21x^2 - 966x + 10584$.
\item On en déduit la forme factorisée suivante
\[
f'(x) = 21 (x - 28)(x-18)
\]
\item Pas de correction disponible
\item À causes des branches extérieurs, la fonction $f(x)$ n'a pas de maximum ou de minimum.
\end{enumerate}
\end{solution}
%\printsolutionstype{exercise}
\end{document}
%%% Local Variables:
%%% mode: latex
%%% TeX-master: "master"
%%% End:

View File

@ -0,0 +1,203 @@
\documentclass[a5paper,10pt]{article}
\usepackage{myXsim}
\usepackage{tasks}
% Title Page
\title{DM2 \hfill MASSON Grace}
\tribe{TST}
\date{\hfillÀ render pour le Mercredi 24 février}
\xsimsetup{
solution/print = true
}
\begin{document}
\maketitle
\begin{exercise}[subtitle={Loi binomiale}]
Trois personnes s'apprêtent à passer le portique de sécurité. On suppose que pour chaque personne la probabilité que le portique sonne est égale à $0.17$.
Soit $X$ la variable aléatoire donnant le nombre de personnes faisant sonner le portique, parmi les 3 personnes de ce groupe.
\begin{enumerate}
\item Tracer l'arbre représentant le situation.
\item Justifier que $X$ suit une loi binomiale dont on précisera les paramètres.
\item Quelle est la probabilité qu'une seule personne fasse sonner le portique?
\item Calculer puis interpréter les probabilités suivantes
\[
P(X = 0) \qquad \qquad P(X \geq 2)
\]
\item Calculer l'espérance de $X$ et interpréter le résultat.
\end{enumerate}
\end{exercise}
\begin{solution}
\begin{enumerate}
\item
\begin{tikzpicture}[sloped]
\node {.}
child {node {$0$}
child {node {$0$}
child {node {$0$}
edge from parent
node[above] {0.83}
}
child {node {$1$}
edge from parent
node[above] {0.17}
}
edge from parent
node[above] {0.83}
}
child[missing] {}
child {node {$1$}
child {node {$0$}
edge from parent
node[above] {0.83}
}
child {node {$1$}
edge from parent
node[above] {0.17}
}
edge from parent
node[above] {0.83}
}
edge from parent
node[above] {0.83}
}
child[missing] {}
child[missing] {}
child[missing] {}
child { node {$1$}
child {node {$0$}
child {node {$0$}
edge from parent
node[above] {0.83}
}
child {node {$1$}
edge from parent
node[above] {0.17}
}
edge from parent
node[above] {0.83}
}
child[missing] {}
child {node {$1$}
child {node {$0$}
edge from parent
node[above] {0.83}
}
child {node {$1$}
edge from parent
node[above] {0.17}
}
edge from parent
node[above] {0.83}
}
edge from parent
node[above] {0.17}
} ;
\end{tikzpicture}
\item Chaque personne a 2 possibilités (1: fait sonner ou 2: ne fait pas sonner) et l'on fait passer 3 personnes ce qui correspond à une répétition identique et aléatoire. On peut donc modéliser la situation par une loi binomiale.
\[
X \sim \mathcal{B}(3; 0.76)
\]
\item Probabilité qu'une seule personne fasse sonner le portique. On voit qu'il y a 3 branches qui correspondent à cette situation dont
\[
P(X = 1) = 3 \times 0.17^1 \times 0.83^2 \approx 0.351
\]
\item
\[
P(X = 0) = 0.83^3 \approx 0.572
\]
\[
P(X \geq 2) = P(X = 2) + P(X = 3) = 3 \times 0.17^2 \times 0.83^1 + 0.17^3 \approx 0.077
\]
\item Il faut d'abord tracer le tableau résumant la loi de probabilité:
\begin{center}
\begin{tabular}{|c|*{4}{c|}}
\hline
Valeur & 0 & 1 & 2 & 3 \\
\hline
Probabilité & $0.572$ & $0.351$ & $0.072$ &$0.005$ \\
\hline
\end{tabular}
\end{center}
On peut alors calculer l'espérance
\[
E[X] = 0 \times 0.572 + 1 \times 0.351 + 2 \times 0.072 + 3 \times 0.005 = 0.51
\]
On peut donc estimer qu'il y aura en moyenne $0.51$ personnes qui feront sonner le portique sur les 3 personnes.
\end{enumerate}
\end{solution}
\begin{exercise}[subtitle={Équation puissance}]
Résoudre les équations et inéquations suivantes
\begin{multicols}{2}
\begin{enumerate}
\item $10^x = 37$
\item $6^x = 32$
\item $0.89^x \leq 36$
\item $7 \times 0.02^x = 44$
\end{enumerate}
\end{multicols}
\end{exercise}
\begin{solution}
Les solutions ci-dessous ne sont pas justifiée car l'ordinateur ne sait pas faire. Par contre, vous vous devez savoir justifier vos réponses!
\begin{enumerate}
\item $x = \log(37)$
\item $x = \frac{\log(32)}{\log(6)}$
\item Il faut faire attention quand on divise par un log car ce dernier peut être négatif ce qui est le cas ici. Il faut donc pense à changer le sens de l'inégalité.
$x \geq \frac{\log(36)}{\log(0.89)}$
\item Il faut penser à faire la division à par $7$ avant d'utiliser le log car sinon, on ne peut pas utiliser la formule $\log(a^n) = n\times \log(a)$.
$x = \frac{\log(6.29)}{\log(0.02)}$
\end{enumerate}
\end{solution}
\begin{exercise}[subtitle={Étude de fonctions}]
Soit $f(x) = - 8x^3 + 96x - 33$ une fonction définie sur $\R$.
\begin{enumerate}
\item Calculer $f'(x)$ la dérivée de $f(x)$.
\item Calculer $f'(2)$ et $f'(-2)$.
\item En déduire une forme factorisée de $f'(x)$.
\item Étudier le signe de $f'(x)$ et en déduire les variations de $f(x)$.
\item Est-ce que la fonction $f(x)$ admet un maximum ou un minimum? Si oui, calculer sa valeur.
\end{enumerate}
\end{exercise}
\begin{solution}
\begin{enumerate}
\item Dérivée de $f(x)$: $f'(x) = - 24x^2 + 96$
\item
\begin{align*}
f'(2) &= - 24 \times 2^{2} + 96\\&= - 24 \times 4 + 96\\&= - 96 + 96\\&= 0
\end{align*}
\begin{align*}
f'(-2) &= - 24 \times - 2^{2} + 96\\&= - 24 \times 4 + 96\\&= - 96 + 96\\&= 0
\end{align*}
Donc $x = 2$ et $x=-2$ sont des racines de $f'(x) = - 24x^2 + 96$.
\item On en déduit la forme factorisée suivante
\[
f'(x) = -24 (x - 2)(x--2)
\]
\item Pas de correction disponible
\item À causes des branches extérieurs, la fonction $f(x)$ n'a pas de maximum ou de minimum.
\end{enumerate}
\end{solution}
%\printsolutionstype{exercise}
\end{document}
%%% Local Variables:
%%% mode: latex
%%% TeX-master: "master"
%%% End:

View File

@ -0,0 +1,203 @@
\documentclass[a5paper,10pt]{article}
\usepackage{myXsim}
\usepackage{tasks}
% Title Page
\title{DM2 \hfill MOKHTARI Nissrine}
\tribe{TST}
\date{\hfillÀ render pour le Mercredi 24 février}
\xsimsetup{
solution/print = true
}
\begin{document}
\maketitle
\begin{exercise}[subtitle={Loi binomiale}]
Trois personnes s'apprêtent à passer le portique de sécurité. On suppose que pour chaque personne la probabilité que le portique sonne est égale à $0.1$.
Soit $X$ la variable aléatoire donnant le nombre de personnes faisant sonner le portique, parmi les 3 personnes de ce groupe.
\begin{enumerate}
\item Tracer l'arbre représentant le situation.
\item Justifier que $X$ suit une loi binomiale dont on précisera les paramètres.
\item Quelle est la probabilité qu'une seule personne fasse sonner le portique?
\item Calculer puis interpréter les probabilités suivantes
\[
P(X = 0) \qquad \qquad P(X \geq 2)
\]
\item Calculer l'espérance de $X$ et interpréter le résultat.
\end{enumerate}
\end{exercise}
\begin{solution}
\begin{enumerate}
\item
\begin{tikzpicture}[sloped]
\node {.}
child {node {$0$}
child {node {$0$}
child {node {$0$}
edge from parent
node[above] {0.9}
}
child {node {$1$}
edge from parent
node[above] {0.1}
}
edge from parent
node[above] {0.9}
}
child[missing] {}
child {node {$1$}
child {node {$0$}
edge from parent
node[above] {0.9}
}
child {node {$1$}
edge from parent
node[above] {0.1}
}
edge from parent
node[above] {0.9}
}
edge from parent
node[above] {0.9}
}
child[missing] {}
child[missing] {}
child[missing] {}
child { node {$1$}
child {node {$0$}
child {node {$0$}
edge from parent
node[above] {0.9}
}
child {node {$1$}
edge from parent
node[above] {0.1}
}
edge from parent
node[above] {0.9}
}
child[missing] {}
child {node {$1$}
child {node {$0$}
edge from parent
node[above] {0.9}
}
child {node {$1$}
edge from parent
node[above] {0.1}
}
edge from parent
node[above] {0.9}
}
edge from parent
node[above] {0.1}
} ;
\end{tikzpicture}
\item Chaque personne a 2 possibilités (1: fait sonner ou 2: ne fait pas sonner) et l'on fait passer 3 personnes ce qui correspond à une répétition identique et aléatoire. On peut donc modéliser la situation par une loi binomiale.
\[
X \sim \mathcal{B}(3; 0.76)
\]
\item Probabilité qu'une seule personne fasse sonner le portique. On voit qu'il y a 3 branches qui correspondent à cette situation dont
\[
P(X = 1) = 3 \times 0.1^1 \times 0.9^2 \approx 0.243
\]
\item
\[
P(X = 0) = 0.9^3 \approx 0.729
\]
\[
P(X \geq 2) = P(X = 2) + P(X = 3) = 3 \times 0.1^2 \times 0.9^1 + 0.1^3 \approx 0.028
\]
\item Il faut d'abord tracer le tableau résumant la loi de probabilité:
\begin{center}
\begin{tabular}{|c|*{4}{c|}}
\hline
Valeur & 0 & 1 & 2 & 3 \\
\hline
Probabilité & $0.729$ & $0.243$ & $0.027$ &$0.001$ \\
\hline
\end{tabular}
\end{center}
On peut alors calculer l'espérance
\[
E[X] = 0 \times 0.729 + 1 \times 0.243 + 2 \times 0.027 + 3 \times 0.001 = 0.3
\]
On peut donc estimer qu'il y aura en moyenne $0.3$ personnes qui feront sonner le portique sur les 3 personnes.
\end{enumerate}
\end{solution}
\begin{exercise}[subtitle={Équation puissance}]
Résoudre les équations et inéquations suivantes
\begin{multicols}{2}
\begin{enumerate}
\item $10^x = 45$
\item $2^x = 25$
\item $0.83^x \leq 40$
\item $8 \times 0.07^x = 42$
\end{enumerate}
\end{multicols}
\end{exercise}
\begin{solution}
Les solutions ci-dessous ne sont pas justifiée car l'ordinateur ne sait pas faire. Par contre, vous vous devez savoir justifier vos réponses!
\begin{enumerate}
\item $x = \log(45)$
\item $x = \frac{\log(25)}{\log(2)}$
\item Il faut faire attention quand on divise par un log car ce dernier peut être négatif ce qui est le cas ici. Il faut donc pense à changer le sens de l'inégalité.
$x \geq \frac{\log(40)}{\log(0.83)}$
\item Il faut penser à faire la division à par $8$ avant d'utiliser le log car sinon, on ne peut pas utiliser la formule $\log(a^n) = n\times \log(a)$.
$x = \frac{\log(5.25)}{\log(0.07)}$
\end{enumerate}
\end{solution}
\begin{exercise}[subtitle={Étude de fonctions}]
Soit $f(x) = - 4x^3 + 384x^2 - 9216x - 22$ une fonction définie sur $\R$.
\begin{enumerate}
\item Calculer $f'(x)$ la dérivée de $f(x)$.
\item Calculer $f'(48)$ et $f'(16)$.
\item En déduire une forme factorisée de $f'(x)$.
\item Étudier le signe de $f'(x)$ et en déduire les variations de $f(x)$.
\item Est-ce que la fonction $f(x)$ admet un maximum ou un minimum? Si oui, calculer sa valeur.
\end{enumerate}
\end{exercise}
\begin{solution}
\begin{enumerate}
\item Dérivée de $f(x)$: $f'(x) = - 12x^2 + 768x - 9216$
\item
\begin{align*}
f'(48) &= - 12 \times 48^{2} + 768 \times 48 - 9216\\&= - 12 \times 2304 + 36864 - 9216\\&= - 27648 + 27648\\&= 0
\end{align*}
\begin{align*}
f'(16) &= - 12 \times 16^{2} + 768 \times 16 - 9216\\&= - 12 \times 256 + 12288 - 9216\\&= - 3072 + 3072\\&= 0
\end{align*}
Donc $x = 48$ et $x=16$ sont des racines de $f'(x) = - 12x^2 + 768x - 9216$.
\item On en déduit la forme factorisée suivante
\[
f'(x) = -12 (x - 48)(x-16)
\]
\item Pas de correction disponible
\item À causes des branches extérieurs, la fonction $f(x)$ n'a pas de maximum ou de minimum.
\end{enumerate}
\end{solution}
%\printsolutionstype{exercise}
\end{document}
%%% Local Variables:
%%% mode: latex
%%% TeX-master: "master"
%%% End:

View File

@ -0,0 +1,203 @@
\documentclass[a5paper,10pt]{article}
\usepackage{myXsim}
\usepackage{tasks}
% Title Page
\title{DM2 \hfill MOUFAQ Amine}
\tribe{TST}
\date{\hfillÀ render pour le Mercredi 24 février}
\xsimsetup{
solution/print = true
}
\begin{document}
\maketitle
\begin{exercise}[subtitle={Loi binomiale}]
Trois personnes s'apprêtent à passer le portique de sécurité. On suppose que pour chaque personne la probabilité que le portique sonne est égale à $0.95$.
Soit $X$ la variable aléatoire donnant le nombre de personnes faisant sonner le portique, parmi les 3 personnes de ce groupe.
\begin{enumerate}
\item Tracer l'arbre représentant le situation.
\item Justifier que $X$ suit une loi binomiale dont on précisera les paramètres.
\item Quelle est la probabilité qu'une seule personne fasse sonner le portique?
\item Calculer puis interpréter les probabilités suivantes
\[
P(X = 0) \qquad \qquad P(X \geq 2)
\]
\item Calculer l'espérance de $X$ et interpréter le résultat.
\end{enumerate}
\end{exercise}
\begin{solution}
\begin{enumerate}
\item
\begin{tikzpicture}[sloped]
\node {.}
child {node {$0$}
child {node {$0$}
child {node {$0$}
edge from parent
node[above] {0.05}
}
child {node {$1$}
edge from parent
node[above] {0.95}
}
edge from parent
node[above] {0.05}
}
child[missing] {}
child {node {$1$}
child {node {$0$}
edge from parent
node[above] {0.05}
}
child {node {$1$}
edge from parent
node[above] {0.95}
}
edge from parent
node[above] {0.05}
}
edge from parent
node[above] {0.05}
}
child[missing] {}
child[missing] {}
child[missing] {}
child { node {$1$}
child {node {$0$}
child {node {$0$}
edge from parent
node[above] {0.05}
}
child {node {$1$}
edge from parent
node[above] {0.95}
}
edge from parent
node[above] {0.05}
}
child[missing] {}
child {node {$1$}
child {node {$0$}
edge from parent
node[above] {0.05}
}
child {node {$1$}
edge from parent
node[above] {0.95}
}
edge from parent
node[above] {0.05}
}
edge from parent
node[above] {0.95}
} ;
\end{tikzpicture}
\item Chaque personne a 2 possibilités (1: fait sonner ou 2: ne fait pas sonner) et l'on fait passer 3 personnes ce qui correspond à une répétition identique et aléatoire. On peut donc modéliser la situation par une loi binomiale.
\[
X \sim \mathcal{B}(3; 0.76)
\]
\item Probabilité qu'une seule personne fasse sonner le portique. On voit qu'il y a 3 branches qui correspondent à cette situation dont
\[
P(X = 1) = 3 \times 0.95^1 \times 0.05^2 \approx 0.007
\]
\item
\[
P(X = 0) = 0.05^3 \approx 0.0
\]
\[
P(X \geq 2) = P(X = 2) + P(X = 3) = 3 \times 0.95^2 \times 0.05^1 + 0.95^3 \approx 0.992
\]
\item Il faut d'abord tracer le tableau résumant la loi de probabilité:
\begin{center}
\begin{tabular}{|c|*{4}{c|}}
\hline
Valeur & 0 & 1 & 2 & 3 \\
\hline
Probabilité & $0.0$ & $0.007$ & $0.135$ &$0.857$ \\
\hline
\end{tabular}
\end{center}
On peut alors calculer l'espérance
\[
E[X] = 0 \times 0.0 + 1 \times 0.007 + 2 \times 0.135 + 3 \times 0.857 = 2.85
\]
On peut donc estimer qu'il y aura en moyenne $2.85$ personnes qui feront sonner le portique sur les 3 personnes.
\end{enumerate}
\end{solution}
\begin{exercise}[subtitle={Équation puissance}]
Résoudre les équations et inéquations suivantes
\begin{multicols}{2}
\begin{enumerate}
\item $10^x = 26$
\item $11^x = 31$
\item $0.01^x \leq 10$
\item $5 \times 0.24^x = 3$
\end{enumerate}
\end{multicols}
\end{exercise}
\begin{solution}
Les solutions ci-dessous ne sont pas justifiée car l'ordinateur ne sait pas faire. Par contre, vous vous devez savoir justifier vos réponses!
\begin{enumerate}
\item $x = \log(26)$
\item $x = \frac{\log(31)}{\log(11)}$
\item Il faut faire attention quand on divise par un log car ce dernier peut être négatif ce qui est le cas ici. Il faut donc pense à changer le sens de l'inégalité.
$x \geq \frac{\log(10)}{\log(0.01)}$
\item Il faut penser à faire la division à par $5$ avant d'utiliser le log car sinon, on ne peut pas utiliser la formule $\log(a^n) = n\times \log(a)$.
$x = \frac{\log(0.6)}{\log(0.24)}$
\end{enumerate}
\end{solution}
\begin{exercise}[subtitle={Étude de fonctions}]
Soit $f(x) = 4x^3 - 192x^2 - 9216x - 28$ une fonction définie sur $\R$.
\begin{enumerate}
\item Calculer $f'(x)$ la dérivée de $f(x)$.
\item Calculer $f'(48)$ et $f'(-16)$.
\item En déduire une forme factorisée de $f'(x)$.
\item Étudier le signe de $f'(x)$ et en déduire les variations de $f(x)$.
\item Est-ce que la fonction $f(x)$ admet un maximum ou un minimum? Si oui, calculer sa valeur.
\end{enumerate}
\end{exercise}
\begin{solution}
\begin{enumerate}
\item Dérivée de $f(x)$: $f'(x) = 12x^2 - 384x - 9216$
\item
\begin{align*}
f'(48) &= 12 \times 48^{2} - 384 \times 48 - 9216\\&= 12 \times 2304 - 18432 - 9216\\&= 27648 - 27648\\&= 0
\end{align*}
\begin{align*}
f'(-16) &= 12 \times - 16^{2} - 384(- 16) - 9216\\&= 12 \times 256 + 6144 - 9216\\&= 3072 - 3072\\&= 0
\end{align*}
Donc $x = 48$ et $x=-16$ sont des racines de $f'(x) = 12x^2 - 384x - 9216$.
\item On en déduit la forme factorisée suivante
\[
f'(x) = 12 (x - 48)(x--16)
\]
\item Pas de correction disponible
\item À causes des branches extérieurs, la fonction $f(x)$ n'a pas de maximum ou de minimum.
\end{enumerate}
\end{solution}
%\printsolutionstype{exercise}
\end{document}
%%% Local Variables:
%%% mode: latex
%%% TeX-master: "master"
%%% End:

View File

@ -0,0 +1,203 @@
\documentclass[a5paper,10pt]{article}
\usepackage{myXsim}
\usepackage{tasks}
% Title Page
\title{DM2 \hfill ONAL Yakub}
\tribe{TST}
\date{\hfillÀ render pour le Mercredi 24 février}
\xsimsetup{
solution/print = true
}
\begin{document}
\maketitle
\begin{exercise}[subtitle={Loi binomiale}]
Trois personnes s'apprêtent à passer le portique de sécurité. On suppose que pour chaque personne la probabilité que le portique sonne est égale à $0.97$.
Soit $X$ la variable aléatoire donnant le nombre de personnes faisant sonner le portique, parmi les 3 personnes de ce groupe.
\begin{enumerate}
\item Tracer l'arbre représentant le situation.
\item Justifier que $X$ suit une loi binomiale dont on précisera les paramètres.
\item Quelle est la probabilité qu'une seule personne fasse sonner le portique?
\item Calculer puis interpréter les probabilités suivantes
\[
P(X = 0) \qquad \qquad P(X \geq 2)
\]
\item Calculer l'espérance de $X$ et interpréter le résultat.
\end{enumerate}
\end{exercise}
\begin{solution}
\begin{enumerate}
\item
\begin{tikzpicture}[sloped]
\node {.}
child {node {$0$}
child {node {$0$}
child {node {$0$}
edge from parent
node[above] {0.03}
}
child {node {$1$}
edge from parent
node[above] {0.97}
}
edge from parent
node[above] {0.03}
}
child[missing] {}
child {node {$1$}
child {node {$0$}
edge from parent
node[above] {0.03}
}
child {node {$1$}
edge from parent
node[above] {0.97}
}
edge from parent
node[above] {0.03}
}
edge from parent
node[above] {0.03}
}
child[missing] {}
child[missing] {}
child[missing] {}
child { node {$1$}
child {node {$0$}
child {node {$0$}
edge from parent
node[above] {0.03}
}
child {node {$1$}
edge from parent
node[above] {0.97}
}
edge from parent
node[above] {0.03}
}
child[missing] {}
child {node {$1$}
child {node {$0$}
edge from parent
node[above] {0.03}
}
child {node {$1$}
edge from parent
node[above] {0.97}
}
edge from parent
node[above] {0.03}
}
edge from parent
node[above] {0.97}
} ;
\end{tikzpicture}
\item Chaque personne a 2 possibilités (1: fait sonner ou 2: ne fait pas sonner) et l'on fait passer 3 personnes ce qui correspond à une répétition identique et aléatoire. On peut donc modéliser la situation par une loi binomiale.
\[
X \sim \mathcal{B}(3; 0.76)
\]
\item Probabilité qu'une seule personne fasse sonner le portique. On voit qu'il y a 3 branches qui correspondent à cette situation dont
\[
P(X = 1) = 3 \times 0.97^1 \times 0.03^2 \approx 0.003
\]
\item
\[
P(X = 0) = 0.03^3 \approx 0.0
\]
\[
P(X \geq 2) = P(X = 2) + P(X = 3) = 3 \times 0.97^2 \times 0.03^1 + 0.97^3 \approx 0.998
\]
\item Il faut d'abord tracer le tableau résumant la loi de probabilité:
\begin{center}
\begin{tabular}{|c|*{4}{c|}}
\hline
Valeur & 0 & 1 & 2 & 3 \\
\hline
Probabilité & $0.0$ & $0.003$ & $0.085$ &$0.913$ \\
\hline
\end{tabular}
\end{center}
On peut alors calculer l'espérance
\[
E[X] = 0 \times 0.0 + 1 \times 0.003 + 2 \times 0.085 + 3 \times 0.913 = 2.91
\]
On peut donc estimer qu'il y aura en moyenne $2.91$ personnes qui feront sonner le portique sur les 3 personnes.
\end{enumerate}
\end{solution}
\begin{exercise}[subtitle={Équation puissance}]
Résoudre les équations et inéquations suivantes
\begin{multicols}{2}
\begin{enumerate}
\item $10^x = 10$
\item $19^x = 8$
\item $0.75^x \leq 2$
\item $5 \times 0.57^x = 24$
\end{enumerate}
\end{multicols}
\end{exercise}
\begin{solution}
Les solutions ci-dessous ne sont pas justifiée car l'ordinateur ne sait pas faire. Par contre, vous vous devez savoir justifier vos réponses!
\begin{enumerate}
\item $x = \log(10)$
\item $x = \frac{\log(8)}{\log(19)}$
\item Il faut faire attention quand on divise par un log car ce dernier peut être négatif ce qui est le cas ici. Il faut donc pense à changer le sens de l'inégalité.
$x \geq \frac{\log(2)}{\log(0.75)}$
\item Il faut penser à faire la division à par $5$ avant d'utiliser le log car sinon, on ne peut pas utiliser la formule $\log(a^n) = n\times \log(a)$.
$x = \frac{\log(4.8)}{\log(0.57)}$
\end{enumerate}
\end{solution}
\begin{exercise}[subtitle={Étude de fonctions}]
Soit $f(x) = - 5x^3 - 7.5x^2 + 1080x - 33$ une fonction définie sur $\R$.
\begin{enumerate}
\item Calculer $f'(x)$ la dérivée de $f(x)$.
\item Calculer $f'(8)$ et $f'(-9)$.
\item En déduire une forme factorisée de $f'(x)$.
\item Étudier le signe de $f'(x)$ et en déduire les variations de $f(x)$.
\item Est-ce que la fonction $f(x)$ admet un maximum ou un minimum? Si oui, calculer sa valeur.
\end{enumerate}
\end{exercise}
\begin{solution}
\begin{enumerate}
\item Dérivée de $f(x)$: $f'(x) = - 15x^2 - 15x + 1080$
\item
\begin{align*}
f'(8) &= - 15 \times 8^{2} - 15 \times 8 + 1080\\&= - 15 \times 64 - 120 + 1080\\&= - 960 + 960\\&= 0
\end{align*}
\begin{align*}
f'(-9) &= - 15 \times - 9^{2} - 15(- 9) + 1080\\&= - 15 \times 81 + 135 + 1080\\&= - 1215 + 1215\\&= 0
\end{align*}
Donc $x = 8$ et $x=-9$ sont des racines de $f'(x) = - 15x^2 - 15x + 1080$.
\item On en déduit la forme factorisée suivante
\[
f'(x) = -15 (x - 8)(x--9)
\]
\item Pas de correction disponible
\item À causes des branches extérieurs, la fonction $f(x)$ n'a pas de maximum ou de minimum.
\end{enumerate}
\end{solution}
%\printsolutionstype{exercise}
\end{document}
%%% Local Variables:
%%% mode: latex
%%% TeX-master: "master"
%%% End:

View File

@ -0,0 +1,203 @@
\documentclass[a5paper,10pt]{article}
\usepackage{myXsim}
\usepackage{tasks}
% Title Page
\title{DM2 \hfill SORIANO Laura}
\tribe{TST}
\date{\hfillÀ render pour le Mercredi 24 février}
\xsimsetup{
solution/print = true
}
\begin{document}
\maketitle
\begin{exercise}[subtitle={Loi binomiale}]
Trois personnes s'apprêtent à passer le portique de sécurité. On suppose que pour chaque personne la probabilité que le portique sonne est égale à $0.76$.
Soit $X$ la variable aléatoire donnant le nombre de personnes faisant sonner le portique, parmi les 3 personnes de ce groupe.
\begin{enumerate}
\item Tracer l'arbre représentant le situation.
\item Justifier que $X$ suit une loi binomiale dont on précisera les paramètres.
\item Quelle est la probabilité qu'une seule personne fasse sonner le portique?
\item Calculer puis interpréter les probabilités suivantes
\[
P(X = 0) \qquad \qquad P(X \geq 2)
\]
\item Calculer l'espérance de $X$ et interpréter le résultat.
\end{enumerate}
\end{exercise}
\begin{solution}
\begin{enumerate}
\item
\begin{tikzpicture}[sloped]
\node {.}
child {node {$0$}
child {node {$0$}
child {node {$0$}
edge from parent
node[above] {0.24}
}
child {node {$1$}
edge from parent
node[above] {0.76}
}
edge from parent
node[above] {0.24}
}
child[missing] {}
child {node {$1$}
child {node {$0$}
edge from parent
node[above] {0.24}
}
child {node {$1$}
edge from parent
node[above] {0.76}
}
edge from parent
node[above] {0.24}
}
edge from parent
node[above] {0.24}
}
child[missing] {}
child[missing] {}
child[missing] {}
child { node {$1$}
child {node {$0$}
child {node {$0$}
edge from parent
node[above] {0.24}
}
child {node {$1$}
edge from parent
node[above] {0.76}
}
edge from parent
node[above] {0.24}
}
child[missing] {}
child {node {$1$}
child {node {$0$}
edge from parent
node[above] {0.24}
}
child {node {$1$}
edge from parent
node[above] {0.76}
}
edge from parent
node[above] {0.24}
}
edge from parent
node[above] {0.76}
} ;
\end{tikzpicture}
\item Chaque personne a 2 possibilités (1: fait sonner ou 2: ne fait pas sonner) et l'on fait passer 3 personnes ce qui correspond à une répétition identique et aléatoire. On peut donc modéliser la situation par une loi binomiale.
\[
X \sim \mathcal{B}(3; 0.76)
\]
\item Probabilité qu'une seule personne fasse sonner le portique. On voit qu'il y a 3 branches qui correspondent à cette situation dont
\[
P(X = 1) = 3 \times 0.76^1 \times 0.24^2 \approx 0.131
\]
\item
\[
P(X = 0) = 0.24^3 \approx 0.014
\]
\[
P(X \geq 2) = P(X = 2) + P(X = 3) = 3 \times 0.76^2 \times 0.24^1 + 0.76^3 \approx 0.855
\]
\item Il faut d'abord tracer le tableau résumant la loi de probabilité:
\begin{center}
\begin{tabular}{|c|*{4}{c|}}
\hline
Valeur & 0 & 1 & 2 & 3 \\
\hline
Probabilité & $0.014$ & $0.131$ & $0.416$ &$0.439$ \\
\hline
\end{tabular}
\end{center}
On peut alors calculer l'espérance
\[
E[X] = 0 \times 0.014 + 1 \times 0.131 + 2 \times 0.416 + 3 \times 0.439 = 2.28
\]
On peut donc estimer qu'il y aura en moyenne $2.28$ personnes qui feront sonner le portique sur les 3 personnes.
\end{enumerate}
\end{solution}
\begin{exercise}[subtitle={Équation puissance}]
Résoudre les équations et inéquations suivantes
\begin{multicols}{2}
\begin{enumerate}
\item $10^x = 14$
\item $14^x = 36$
\item $0.74^x \leq 6$
\item $10 \times 0.11^x = 18$
\end{enumerate}
\end{multicols}
\end{exercise}
\begin{solution}
Les solutions ci-dessous ne sont pas justifiée car l'ordinateur ne sait pas faire. Par contre, vous vous devez savoir justifier vos réponses!
\begin{enumerate}
\item $x = \log(14)$
\item $x = \frac{\log(36)}{\log(14)}$
\item Il faut faire attention quand on divise par un log car ce dernier peut être négatif ce qui est le cas ici. Il faut donc pense à changer le sens de l'inégalité.
$x \geq \frac{\log(6)}{\log(0.74)}$
\item Il faut penser à faire la division à par $10$ avant d'utiliser le log car sinon, on ne peut pas utiliser la formule $\log(a^n) = n\times \log(a)$.
$x = \frac{\log(1.8)}{\log(0.11)}$
\end{enumerate}
\end{solution}
\begin{exercise}[subtitle={Étude de fonctions}]
Soit $f(x) = 4x^3 - 228x^2 + 1632x + 16$ une fonction définie sur $\R$.
\begin{enumerate}
\item Calculer $f'(x)$ la dérivée de $f(x)$.
\item Calculer $f'(34)$ et $f'(4)$.
\item En déduire une forme factorisée de $f'(x)$.
\item Étudier le signe de $f'(x)$ et en déduire les variations de $f(x)$.
\item Est-ce que la fonction $f(x)$ admet un maximum ou un minimum? Si oui, calculer sa valeur.
\end{enumerate}
\end{exercise}
\begin{solution}
\begin{enumerate}
\item Dérivée de $f(x)$: $f'(x) = 12x^2 - 456x + 1632$
\item
\begin{align*}
f'(34) &= 12 \times 34^{2} - 456 \times 34 + 1632\\&= 12 \times 1156 - 15504 + 1632\\&= 13872 - 13872\\&= 0
\end{align*}
\begin{align*}
f'(4) &= 12 \times 4^{2} - 456 \times 4 + 1632\\&= 12 \times 16 - 1824 + 1632\\&= 192 - 192\\&= 0
\end{align*}
Donc $x = 34$ et $x=4$ sont des racines de $f'(x) = 12x^2 - 456x + 1632$.
\item On en déduit la forme factorisée suivante
\[
f'(x) = 12 (x - 34)(x-4)
\]
\item Pas de correction disponible
\item À causes des branches extérieurs, la fonction $f(x)$ n'a pas de maximum ou de minimum.
\end{enumerate}
\end{solution}
%\printsolutionstype{exercise}
\end{document}
%%% Local Variables:
%%% mode: latex
%%% TeX-master: "master"
%%% End:

View File

@ -0,0 +1,203 @@
\documentclass[a5paper,10pt]{article}
\usepackage{myXsim}
\usepackage{tasks}
% Title Page
\title{DM2 \hfill VECCHIO Léa}
\tribe{TST}
\date{\hfillÀ render pour le Mercredi 24 février}
\xsimsetup{
solution/print = true
}
\begin{document}
\maketitle
\begin{exercise}[subtitle={Loi binomiale}]
Trois personnes s'apprêtent à passer le portique de sécurité. On suppose que pour chaque personne la probabilité que le portique sonne est égale à $0.55$.
Soit $X$ la variable aléatoire donnant le nombre de personnes faisant sonner le portique, parmi les 3 personnes de ce groupe.
\begin{enumerate}
\item Tracer l'arbre représentant le situation.
\item Justifier que $X$ suit une loi binomiale dont on précisera les paramètres.
\item Quelle est la probabilité qu'une seule personne fasse sonner le portique?
\item Calculer puis interpréter les probabilités suivantes
\[
P(X = 0) \qquad \qquad P(X \geq 2)
\]
\item Calculer l'espérance de $X$ et interpréter le résultat.
\end{enumerate}
\end{exercise}
\begin{solution}
\begin{enumerate}
\item
\begin{tikzpicture}[sloped]
\node {.}
child {node {$0$}
child {node {$0$}
child {node {$0$}
edge from parent
node[above] {0.45}
}
child {node {$1$}
edge from parent
node[above] {0.55}
}
edge from parent
node[above] {0.45}
}
child[missing] {}
child {node {$1$}
child {node {$0$}
edge from parent
node[above] {0.45}
}
child {node {$1$}
edge from parent
node[above] {0.55}
}
edge from parent
node[above] {0.45}
}
edge from parent
node[above] {0.45}
}
child[missing] {}
child[missing] {}
child[missing] {}
child { node {$1$}
child {node {$0$}
child {node {$0$}
edge from parent
node[above] {0.45}
}
child {node {$1$}
edge from parent
node[above] {0.55}
}
edge from parent
node[above] {0.45}
}
child[missing] {}
child {node {$1$}
child {node {$0$}
edge from parent
node[above] {0.45}
}
child {node {$1$}
edge from parent
node[above] {0.55}
}
edge from parent
node[above] {0.45}
}
edge from parent
node[above] {0.55}
} ;
\end{tikzpicture}
\item Chaque personne a 2 possibilités (1: fait sonner ou 2: ne fait pas sonner) et l'on fait passer 3 personnes ce qui correspond à une répétition identique et aléatoire. On peut donc modéliser la situation par une loi binomiale.
\[
X \sim \mathcal{B}(3; 0.76)
\]
\item Probabilité qu'une seule personne fasse sonner le portique. On voit qu'il y a 3 branches qui correspondent à cette situation dont
\[
P(X = 1) = 3 \times 0.55^1 \times 0.45^2 \approx 0.334
\]
\item
\[
P(X = 0) = 0.45^3 \approx 0.091
\]
\[
P(X \geq 2) = P(X = 2) + P(X = 3) = 3 \times 0.55^2 \times 0.45^1 + 0.55^3 \approx 0.574
\]
\item Il faut d'abord tracer le tableau résumant la loi de probabilité:
\begin{center}
\begin{tabular}{|c|*{4}{c|}}
\hline
Valeur & 0 & 1 & 2 & 3 \\
\hline
Probabilité & $0.091$ & $0.334$ & $0.408$ &$0.166$ \\
\hline
\end{tabular}
\end{center}
On peut alors calculer l'espérance
\[
E[X] = 0 \times 0.091 + 1 \times 0.334 + 2 \times 0.408 + 3 \times 0.166 = 1.65
\]
On peut donc estimer qu'il y aura en moyenne $1.65$ personnes qui feront sonner le portique sur les 3 personnes.
\end{enumerate}
\end{solution}
\begin{exercise}[subtitle={Équation puissance}]
Résoudre les équations et inéquations suivantes
\begin{multicols}{2}
\begin{enumerate}
\item $10^x = 14$
\item $7^x = 47$
\item $0.18^x \leq 40$
\item $3 \times 0.21^x = 25$
\end{enumerate}
\end{multicols}
\end{exercise}
\begin{solution}
Les solutions ci-dessous ne sont pas justifiée car l'ordinateur ne sait pas faire. Par contre, vous vous devez savoir justifier vos réponses!
\begin{enumerate}
\item $x = \log(14)$
\item $x = \frac{\log(47)}{\log(7)}$
\item Il faut faire attention quand on divise par un log car ce dernier peut être négatif ce qui est le cas ici. Il faut donc pense à changer le sens de l'inégalité.
$x \geq \frac{\log(40)}{\log(0.18)}$
\item Il faut penser à faire la division à par $3$ avant d'utiliser le log car sinon, on ne peut pas utiliser la formule $\log(a^n) = n\times \log(a)$.
$x = \frac{\log(8.33)}{\log(0.21)}$
\end{enumerate}
\end{solution}
\begin{exercise}[subtitle={Étude de fonctions}]
Soit $f(x) = x^3 - 64.5x^2 + 360x - 44$ une fonction définie sur $\R$.
\begin{enumerate}
\item Calculer $f'(x)$ la dérivée de $f(x)$.
\item Calculer $f'(40)$ et $f'(3)$.
\item En déduire une forme factorisée de $f'(x)$.
\item Étudier le signe de $f'(x)$ et en déduire les variations de $f(x)$.
\item Est-ce que la fonction $f(x)$ admet un maximum ou un minimum? Si oui, calculer sa valeur.
\end{enumerate}
\end{exercise}
\begin{solution}
\begin{enumerate}
\item Dérivée de $f(x)$: $f'(x) = 3x^2 - 129x + 360$
\item
\begin{align*}
f'(40) &= 3 \times 40^{2} - 129 \times 40 + 360\\&= 3 \times 1600 - 5160 + 360\\&= 4800 - 4800\\&= 0
\end{align*}
\begin{align*}
f'(3) &= 3 \times 3^{2} - 129 \times 3 + 360\\&= 3 \times 9 - 387 + 360\\&= 27 - 27\\&= 0
\end{align*}
Donc $x = 40$ et $x=3$ sont des racines de $f'(x) = 3x^2 - 129x + 360$.
\item On en déduit la forme factorisée suivante
\[
f'(x) = 3 (x - 40)(x-3)
\]
\item Pas de correction disponible
\item À causes des branches extérieurs, la fonction $f(x)$ n'a pas de maximum ou de minimum.
\end{enumerate}
\end{solution}
%\printsolutionstype{exercise}
\end{document}
%%% Local Variables:
%%% mode: latex
%%% TeX-master: "master"
%%% End:

Binary file not shown.

View File

@ -0,0 +1,226 @@
\documentclass[a5paper,10pt]{article}
\usepackage{myXsim}
\usepackage{tasks}
% Title Page
\title{DM2 \hfill \Var{Nom}}
\tribe{TST}
\date{\hfillÀ render pour le Mercredi 24 février}
\xsimsetup{
solution/print = false
}
\begin{document}
\maketitle
\begin{exercise}[subtitle={Loi binomiale}]
%- set p = round(random(), 2)
Trois personnes s'apprêtent à passer le portique de sécurité. On suppose que pour chaque personne la probabilité que le portique sonne est égale à $\Var{p}$.
Soit $X$ la variable aléatoire donnant le nombre de personnes faisant sonner le portique, parmi les 3 personnes de ce groupe.
\begin{enumerate}
\item Tracer l'arbre représentant le situation.
\item Justifier que $X$ suit une loi binomiale dont on précisera les paramètres.
\item Quelle est la probabilité qu'une seule personne fasse sonner le portique?
\item Calculer puis interpréter les probabilités suivantes
\[
P(X = 0) \qquad \qquad P(X \geq 2)
\]
\item Calculer l'espérance de $X$ et interpréter le résultat.
\end{enumerate}
\end{exercise}
\begin{solution}
%- set q = round(1-p, 2)
\begin{enumerate}
\item
\begin{tikzpicture}[sloped]
\node {.}
child {node {$0$}
child {node {$0$}
child {node {$0$}
edge from parent
node[above] {\Var{q}}
}
child {node {$1$}
edge from parent
node[above] {\Var{p}}
}
edge from parent
node[above] {\Var{q}}
}
child[missing] {}
child {node {$1$}
child {node {$0$}
edge from parent
node[above] {\Var{q}}
}
child {node {$1$}
edge from parent
node[above] {\Var{p}}
}
edge from parent
node[above] {\Var{q}}
}
edge from parent
node[above] {\Var{q}}
}
child[missing] {}
child[missing] {}
child[missing] {}
child { node {$1$}
child {node {$0$}
child {node {$0$}
edge from parent
node[above] {\Var{q}}
}
child {node {$1$}
edge from parent
node[above] {\Var{p}}
}
edge from parent
node[above] {\Var{q}}
}
child[missing] {}
child {node {$1$}
child {node {$0$}
edge from parent
node[above] {\Var{q}}
}
child {node {$1$}
edge from parent
node[above] {\Var{p}}
}
edge from parent
node[above] {\Var{q}}
}
edge from parent
node[above] {\Var{p}}
} ;
\end{tikzpicture}
\item Chaque personne a 2 possibilités (1: fait sonner ou 2: ne fait pas sonner) et l'on fait passer 3 personnes ce qui correspond à une répétition identique et aléatoire. On peut donc modéliser la situation par une loi binomiale.
\[
X \sim \mathcal{B}(3; \Var{0.76})
\]
%- set p0 = round(q**3, 3)
%- set p1 = round(3*p*q**2, 3)
%- set p2 = round(3*p**2*q, 3)
%- set p3 = round(p**3, 3)
\item Probabilité qu'une seule personne fasse sonner le portique. On voit qu'il y a 3 branches qui correspondent à cette situation dont
\[
P(X = 1) = 3 \times \Var{p}^1 \times \Var{q}^2 \approx \Var{p1}
\]
\item
\[
P(X = 0) = \Var{q}^3 \approx \Var{p0}
\]
\[
P(X \geq 2) = P(X = 2) + P(X = 3) = 3 \times \Var{p}^2 \times \Var{q}^1 + \Var{p}^3 \approx \Var{round(p2 + p3, 3)}
\]
\item Il faut d'abord tracer le tableau résumant la loi de probabilité:
\begin{center}
\begin{tabular}{|c|*{4}{c|}}
\hline
Valeur & 0 & 1 & 2 & 3 \\
\hline
Probabilité & $\Var{p0}$ & $\Var{p1}$ & $\Var{p2}$ &$\Var{p3}$ \\
\hline
\end{tabular}
\end{center}
On peut alors calculer l'espérance
%- set E = round(3*p, 3)
\[
E[X] = 0 \times \Var{p0} + 1 \times \Var{p1} + 2 \times \Var{p2} + 3 \times \Var{p3} = \Var{E}
\]
On peut donc estimer qu'il y aura en moyenne $\Var{E}$ personnes qui feront sonner le portique sur les 3 personnes.
\end{enumerate}
\end{solution}
\begin{exercise}[subtitle={Équation puissance}]
Résoudre les équations et inéquations suivantes
\begin{multicols}{2}
\begin{enumerate}
%- set a1 = randint(1, 50)
\item $10^x = \Var{a1}$
%- set a2 = randint(1, 50)
%- set b2 = randint(2, 20)
\item $\Var{b2}^x = \Var{a2}$
%- set a3 = randint(2, 50)
%- set b3 = round(random(), 2)
\item $\Var{b3}^x \leq \Var{a3}$
%- set a4 = randint(2, 50)
%- set b4 = round(random(), 2)
%- set c4 = randint(2, 10)
\item $\Var{c4} \times \Var{b4}^x = \Var{a4}$
\end{enumerate}
\end{multicols}
\end{exercise}
\begin{solution}
Les solutions ci-dessous ne sont pas justifiée car l'ordinateur ne sait pas faire. Par contre, vous vous devez savoir justifier vos réponses!
\begin{enumerate}
\item $x = \log(\Var{a1})$
\item $x = \frac{\log(\Var{a2})}{\log(\Var{b2})}$
\item Il faut faire attention quand on divise par un log car ce dernier peut être négatif ce qui est le cas ici. Il faut donc pense à changer le sens de l'inégalité.
$x \geq \frac{\log(\Var{a3})}{\log(\Var{b3})}$
\item Il faut penser à faire la division à par $\Var{c4}$ avant d'utiliser le log car sinon, on ne peut pas utiliser la formule $\log(a^n) = n\times \log(a)$.
$x = \frac{\log(\Var{round(a4/c4, 2)})}{\log(\Var{b4})}$
\end{enumerate}
\end{solution}
\begin{exercise}[subtitle={Étude de fonctions}]
%- set a_ = randint(-10, 10)
%- set a = a_*3
%- set x1 = randint(0, 50)
%- set x2 = randint(-20, 20)
%- set b = randint(-50, 50)
%- set f = Polynom.from_coefficients([b, a*x1*x2, -a*(x1+x2)/2, a_])
Soit $f(x) = \Var{f}$ une fonction définie sur $\R$.
\begin{enumerate}
\item Calculer $f'(x)$ la dérivée de $f(x)$.
\item Calculer $f'(\Var{x1})$ et $f'(\Var{x2})$.
\item En déduire une forme factorisée de $f'(x)$.
\item Étudier le signe de $f'(x)$ et en déduire les variations de $f(x)$.
\item Est-ce que la fonction $f(x)$ admet un maximum ou un minimum? Si oui, calculer sa valeur.
\end{enumerate}
\end{exercise}
\begin{solution}
\begin{enumerate}
%- set fp = f.differentiate()
\item Dérivée de $f(x)$: $f'(x) = \Var{fp}$
\item
\begin{align*}
f'(\Var{x1}) &= \Var{fp(x1).explain() |join('\\\\&= ')}
\end{align*}
\begin{align*}
f'(\Var{x2}) &= \Var{fp(x2).explain() |join('\\\\&= ')}
\end{align*}
Donc $x = \Var{x1}$ et $x=\Var{x2}$ sont des racines de $f'(x) = \Var{fp}$.
\item On en déduit la forme factorisée suivante
\[
f'(x) = \Var{a} (x - \Var{x1})(x-\Var{x2})
\]
\item Pas de correction disponible
\item À causes des branches extérieurs, la fonction $f(x)$ n'a pas de maximum ou de minimum.
\end{enumerate}
\end{solution}
%\printsolutionstype{exercise}
\end{document}
%%% Local Variables:
%%% mode: latex
%%% TeX-master: "master"
%%% End: